Sunteți pe pagina 1din 690

Introduccin al Pensamiento Cientfico

UNIVERSIDAD DE BUENOS AIRES

Rector Ruben Hallu Alberto Vicerrector Edgardo Barbieri Mara Secretaria de Asuntos Catalina Acadmicos Nosiglia Subsecretara de Marilina innovacin y Calidad Lipsman Acadmica
PROGRAMA UBA XXI

Claudia Lombardo Vicedirectora Diana Mazza Directora

Mara Coordinacin Desarrollo Alejandra Pedaggico Codazzi Coordinacin Produccin Liliana Multimedial Castillo Mara Alejandra Batista Ariadna Pou Patricia Bucich Ariel Guglielmo Coordinacin Marcela Comunicacin Integral Gamberini

GUA DE ESTUDIO

Equipo Docente Cristina Ambrosini Especialistas en Cristina contenidos Ambrosini Gastn Beraldi Introduccin al pensamiento cientfico : gua de estudio . - 1a ed. - Buenos Aires : Eudeba, 2012. - (UBA XXI; 0) E-Book. Profesora titular de IPC ISBN 978-950-23-2026-7 1. Epistemologa.

CDD 121

Eudeba Universidad de Buenos Aires 1 edicin: agosto de 2012 2012, Editorial Universitaria de Buenos Aires Sociedad de Economa Mixta Av. Rivadavia 1571/73 (1033) Ciudad de Buenos Aires Tel.: 4383-8025 / Fax: 4383-2202 www.eudeba.com.ar

UBA XXI es el Programa de Educacin a Distancia de la Universidad de Buenos Aires, dirigido a alumnos interesados en adoptar esta modalidad de enseanza para cursar algunas materias del Ciclo Bsico Comn. Es una propuesta multimedial que brinda distintos materiales y recursos para organizar el estudio de modo autnomo, sin la obligacin de asistir a clases. En la pgina web institucional http://www.uba.ar/academicos/uba21 se encuentra la informacin sobre esta propuesta de estudio y las primeras orientaciones para incorporarse al

Programa. Adems, UBA XXI cuenta con un Campus virtual, http://www.ubaxxivirtual.uba.ar, un entorno en el cual los alumnos interactan con docentes y compaeros, as como tambin acceden a actividades, materiales didcticos y recursos multimediales para acompaar el cursado de las asignaturas elegidas. Introduccin al Pensamiento Cientfico en UBA XXI Para abordar los contenidos de la materia IPC en esta modalidad, usted cuenta con los siguientes materiales y recursos didcticos: - Textos impresos de lectura obligatoria: Gua de Estudio de IPC

y Bibliografa. - Un espacio en el Campus vir tual desde el cual podr acceder al correo electrnico para consultas sobre la materia, actividades de aprendizaje como foros de discusin y de orientacin pedaggica y a otros recursos didcticos que lo acompaarn en su estudio. Se recomienda escuchar los programas grabados de radio, disponibles en la Mediateca. - Tutoras presenciales que son encuentros optativos para abordar los temas del programa, aclarar dudas sobre los contenidos y sobre las actividades de aprendizaje. - Programas de radio sobre diversos

temas de la asignatura, que se emiten por: FM Radio UBA 90.5 - Sala de lectura en la que se puede consultar la bibliografa obligatoria para el estudio de la materia, en la Sede Central de Pte. J. E. Uriburu 950, 1 piso, correo electrnico: bibliotecaubaxxi@rec.uba.ar. La Gua de Estudio de IPC es una herramienta didctica que acompaa la lectura de la bibliografa obligatoria de la asignatura. El objetivo principal es guiar a los alumnos en la comprensin de la materia. Est organizada siguiendo el desarrollo del Programa. En cada unidad se anticipan los temas y la bibliografa, se sugieren preguntas y

actividades de aprendizaje y se incluyen los "Documentos de Ctedra", que son textos elaborados por los profesores sobre algunos contenidos del programa. Este material didctico es de lectura obligatoria. Las ilustraciones correspondientes a Alicia en el pas de las maravillas han sido basadas en los originales de Sir John Tenniel. Se agradece a Ariel F. Guglielmo por los dibujos realizados. Recomendamos ingresar con frecuencia al espacio que la materia tiene en el Campus virtual para participar de los foros y otras actividades que all se

proponen.

Objetivos generales QUE LOS ALUMNOS: - comprendan las caractersticas generales del conocimiento cientfico; - conozcan los fundamentos del lenguaje lgico y del lenguaje cientfico; - comprendan las principales orientaciones de la epistemologa contempornea; - desarrollen un lenguaje riguroso y preciso; - desarrollen la capacidad de anlisis y el pensamiento crtico; - valoren al pensamiento y a la actividad cientfica como

dimensiones fundamentales de la cultura y la sociedad; - valoren el papel y el impacto social de la ciencia y la tecnologa; - valoren el compromiso social de los cientficos y del resto de los ciudadanos. Objetivos especficos QUE LOS ALUMNOS: - reconozcan las estructuras del lenguaje como paso indispensable para la comprensin de temas metodolgicos; - establezcan diferencias entre el plano formal o emprico del conocimiento cientfico; - evalen las diferentes comprensiones de las estructuras

cientficas: enfoque sincrnico (esttico: las teoras vistas como unas estructuras lgicas) o diacrnico (dinmico: el proceso histrico de gnesis, mejoramiento y cambio de teoras); - comprendan los aspectos salientes de los debates contemporneos en el terreno de la epistemologa y el impacto social de la ciencia y la tecnologa en el mundo contemporneo; - identifiquen las diferencias tericas y metodolgicas entre las ciencias formales y las ciencias fcticas. Unidad 1. Consideraciones sobre el lenguaje 1.1. Lenguaje y teoras cientficas

1.2. Lenguaje y realidad 1.3. Uso y mencin del lenguaje 1.4. La Semitica 1.5. Nombrar y clasificar. Vaguedad y ambigedad 1.6. La definicin Unidad 2. Argumentacin: el escenario formal y el escenario informal 2.1. Las leyes lgicas 2.2. Tautologas, contradicciones, contingencias 2.3. Los razonamientos. Verdad y validez 2.4. Argumento y consecuencia lgica. Reglas lgicas 2.5. La induccin y la analoga 2.6. Es formal la induccin? El intento

de Carnap 2.7. Lgica informal y falacias materiales 2.8. Toulmin, sobre los usos argumentativos Unidad 3. Las ciencias formales 3.1. La matemtica: constructos formales y realidad 3.2. Sistemas axiomticos 3.3. Propiedades de los sistemas axiomticos 3.4. Interpretacin y modelos de los sistemas axiomticos Unidad 4. La cuestin del mtodo en las ciencias fcticas 4.1. Estructura de las teoras empricas 4.2. El problema del mtodo cientfico 4.3. Estrategias metodolgicas bsicas

de las ciencias fcticas 4.4. Mtodo inductivo: inductivismo "estrecho" e inductivismo "sofisticado" 4.5. Mtodo hipottico-deductivo Unidad 5. Explicacin y prediccin en ciencias fcticas 5.1. Qu significa "explicar"? 5.2. Concepto de explicacin cientfica 5.3. Modelos de explicacin cientfica 5.4. La especificidad de las ciencias sociales: intencionalidad y explicacin por mecanismos 5.5. Dimensin explicativa y dimensin predictiva 5.6. El enfoque comprensivista en ciencias sociales Unidad 6. Corrientes epistemolgicas contemporneas

6.1. La epistemologa del siglo XX: problemas fundamentales 6.2. La concepcin epistemolgica del Positivismo lgico 6.3. Las perspectivas falsacionistas: Popper y Lakatos 6.4. La ruptura: del racionalismo de justificacin al anlisis de la historia de la ciencia 6.5. El pensamiento de Kuhn en perspectiva Unidad 7. Ciencia y sociedad 7.1. Poltica cientfica 7.1.1. Caractersticas generales 7.1.2. La planificacin de la poltica cientfica. Debates contemporneos 7.1.3. La formacin del investigador 7.1.4. Etapas de la poltica cientfica

argentina 7.1.5. Institutos de investigacin y Universidad 7.2. Ciencia, tecnologa y sociedad 7.2.1. Ciencia, tecnologa y tica: la responsabilidad social del cientfico y el tecnlogo 7.2.2. Ciencia y tica: debates en torno a la neutralidad valorativa de la ciencia 7.2.3. La tica de la investigacin cientfica 7.2.4. Ciencia bsica, ciencia aplicada y tecnologa. Cientificismo y anticientificismo

Unidad 1. Consideraciones sobre el lenguaje BIBLIOGRAFA OBLIGATORIA: Unidad 1 en la Gua de Estudio de IPC, producida por UBA XXI y editada por Eudeba, a partir de la edicin de 2010. Asti Vera, C. y Ambrosini, C. (2009). Captulo 1. En Argumentos y teoras. Aproximacin a la Epistemologa. Buenos Aires: Educando. BIBLIOGRAFA COMPLEMENTARIA: Copi, I. (1984). Captulos 2 y 4. En Introduccin a la lgica. Buenos Aires: Eudeba. Unidad 2. Argumentacin: el escenario formal y el escenario

informal BIBLIOGRAFA OBLIGATORIA: Unidad 2 en la Gua de Estudio de IPC, producida por UBA XXI y editada por Eudeba, a partir de la edicin de 2010. Asti Vera, C. y Ambrosini, C. (2009). Captulos 2 y 3. En Argumentos y teoras. Aproximacin a la Epistemologa. Buenos Aires: Educando. BIBLIOGRAFA COMPLEMENTARIA: Copi, I. (1984). Captulos 1 y 3. En Introduccin a la lgica. Buenos Aires: Eudeba. Dez, J. A. y Moulines, C. U. (1999). Captulo 2. En Fundamentos de filosofa de la ciencia. Barcelona: Ariel. Bunge, M. (1997). Captulo 3. En

Epistemologa. Mxico: Siglo XXI. Klimovsky, G. (1994). Captulo 18. En Las desventuras del conocimiento cientfico. Buenos Aires: AZ. Unidad 3. Las ciencias formales BIBLIOGRAFA OBLIGATORIA: Unidad 3 en la Gua de Estudio de IPC, producida por UBA XXI y editada por Eudeba, a partir de la edicin de 2010. Asti Vera, C. y Ambrosini, C. (2009). Captulo 4. En Argumentos y teoras. Aproximacin a la Epistemologa. Buenos Aires: Educando. BIBLIOGRAFA COMPLEMENTARIA: Dez, J. A. y Moulines, C. U. (1999). Captulos 4 y 5. En Fundamentos de filosofa de la ciencia. Barcelona: Ariel.

Klimovsky, G. (1994). Captulos 2, 3, 4 y 5. En Las desventuras del conocimiento cientfico. Buenos Aires: AZ. Unidad 4. La cuestin del mtodo en las ciencias fcticas BIBLIOGRAFA OBLIGATORIA: Unidad 4 en la Gua de Estudio de IPC, producida por UBA XXI y editada por Eudeba, a partir de la edicin de 2010. Asti Vera, C. y Ambrosini, C. (2009). Captulo 5. En Argumentos y teoras. Aproximacin a la Epistemologa. Buenos Aires: Educando. BIBLIOGRAFA OBLIGATORIA: Cohen, I. y Nagel, E. (1980). Captulos X, XI, XIV y XVI. En introduccin a la lgica y al mtodo cientfico (vol. II).

Buenos Aires: Amorrortu. Hempel, C. G. (1985). Captulos 2, 3 y 4. En Filosofa de la ciencia natural. Madrid: Alianza. Klimovsky, G. (1994). Captulos 9, 13 y 14. En Las desventuras del conocimiento cientfico. Buenos Aires: AZ. Unidad 5. Explicacin y prediccin en ciencias fcticas BIBLIOGRAFA OBLIGATORIA: Unidad 5 en la Gua de Estudio de IPC, producida por UBA XXI y editada por Eudeba, a partir de la edicin de 2010. Asti Vera, C. y Ambrosini, C. (2009). Captulo 6. En Argumentos y teoras. Aproximacin a la Epistemologa. Buenos Aires: Educando.

Beraldi, G. (2010). Documento de Ctedra: La tensin entre explicacin y comprensin. El problema de la explicacin en las ciencias sociales. En esta Gua de Estudio de IPC, Buenos Aires: Eudeba; y en el Campus virtual de UBA XXI. BIBLIOGRAFA COMPLEMENTARIA: Dez, J. A y Moulines, C. U. (1999). Captulo 7. En Fundamentos de filosofa de la ciencia. Barcelona: Ariel. Schuster, G. (1986). Captulos 2, 3, 4 y 5. En Explicacin y prediccin. Buenos Aires: Clacso. Wright, G. H. von (1979). Explicacin y comprensin. Madrid: Alianza. Unidad 6. Corrientes epistemolgicas

contemporneas BIBLIOGRAFA OBLIGATORIA: Unidad 6 en la Gua de Estudio de IPC, producida por UBA XXI y editada por Eudeba, a partir de la edicin de 2010. Asti Vera, C. y Ambrosini, C. (2009). Captulo 7. En Argumentos y teoras. Aproximacin a la Epistemologa. Buenos Aires: Educando. BIBLIOGRAFA COMPLEMENTARIA: Echeverra, J. (1999). Captulos 1, 2, 3, 4, 5 y 6. En introduccin a la metodologa de la ciencia. La filosofa de la Ciencia en el siglo XX. Madrid: Ctedra. Klimovsky, G. (1994). Captulos 21, 22 y 23. En Las desventuras del conocimiento cientfico. Buenos Aires:

AZ. Unidad 7. Ciencia y sociedad BIBLIOGRAFA OBLIGATORIA: Unidad 7 en la Gua de Estudio de IPC, producida por UBA XXI y editada po Eudeba, a partir de la edicin de 2010. Contratti, M. B. (2010). Documento de Ctedra: Poltica cientfica: problemas y perspectivas. En esta Gua de Estudio de IPC, Buenos Aires: Eudeba; y en el Campus virtual de UBA XXI. Contratti, M. B. (2010). Documento de Ctedra: tica y ciencia. En esta Gua de Estudio de IPC, Buenos Aires: Eudeba; y en el Campus virtual de UBA XXI.

A travs del curso a distancia de esta materia, introduccin al Pensamiento Cientfico, se busca poner al alcance de los estudiantes de la Universidad el conocimiento de la epistemologa, aquella regin de la filosofa que problematiza los mtodos y prcticas de la ciencia as como evala sus resultados. Esta propuesta est animada por la idea de establecer una estrecha relacin entre "construccin de la ciudadana" y "enseanza de epistemologa". Por qu, desde la perspectiva que sostenemos en esta asignatura, ensear epistemologa para posibilitar la constitucin de la ciudadana plena?

Porque, por un lado, el aprendizaje de la epistemologa permite, entre otras cuestiones, distinguir formas de argumentacin racional de otras que solamente parecen serlo, y analizar y debatir en torno de los criterios que permiten esa distincin. Por otro lado, favorece el perfeccionamiento de habilidades de pensamiento necesarias para interactuar en la vida social. Mas para que esta posibilidad se concrete, es imprescindible el acceso a determinados bienes simblicos: el uso adecuado del lenguaje; la competencia discursiva en diversas esferas de la vida social; una disponibilidad de informacin acorde, no slo con las demandas profesionales, sino que posibilite participar de

distintas interacciones en las que las personas se constituyen como ciudadanos porque estn en condiciones de tomar la palabra y sostenerla, evitando as ser vctimas de manipulaciones. Desarrollar las habilidades para pensar y actuar con flexibilidad a partir de lo que ya se sabe es apenas el comienzo de esta tarea. Implementar estrategias para la construccin del conocimiento implica establecer un puente entre el producto y el productor, porque un conocimiento slo es un producto acabado cuando el alumno logra pensar en l como un producto propio. La generacin de este conocimiento autocrtico y consciente, que conforma un instrumento importante

para la transformacin de la realidad, resulta prioritaria. En la actualidad observamos dos tipos de situaciones: el prejuicio contra la teora, que a menudo es presentada ante la opinin pblica como un intil pasatiempo para intelectuales, y una cultura grafa, basada en estmulos intensos y pasajeros (al estilo de los videoclips), en la que la capacidad crtica es cuidadosamente desactivada para ponerla al servicio de algunos prejuicios, sintetizados en unas pocas frases hechas. Arriesgarse a tomar la palabra es un gesto propio de una ciudadana activa; estar impedido de hacerlo revela al lenguaje como barrera social. Por ello creemos que, en el caso

de esta propuesta, es esencial armonizar la enseanza de habilidades de pensamiento y contenidos, donde podemos apreciar la centralidad de un uso reflexivo del lenguaje para propiciar, a su vez, la conformacin de un conocimiento pluralista, capaz de estimular en los estudiantes, desde el inicio de su formacin, el desarrollo de un pensamiento crtico y autnomo. Creemos firmemente que la enseaza de esta materia, de raigambre filosfica, no es simple transmisin de conocimientos (en este caso de lgica, nociones de metodologa e historia de la ciencia), sino que es produccin y, en tanto produccin filosfica, es produccin autocrtica y reflexiva de conocimientos.

Actualmente se menciona la "sociedad del conocimiento" para aludir al tipo de sociedades donde el conocimiento es un factor principal en la produccin de riquezas, de all que sea cada vez ms alta la demanda social de educacin superior en sociedades como la nuestra. Esta propuesta didctica, entonces, busca introducir a los alumnos en la revisin de las estructuras del lenguaje y los distintos escenarios argumentativos para que, con estas herramientas, puedan luego evaluar las distintas posiciones epistemolgicas y el mundo de debates que se genera alrededor de estas particulares y poderosas manifestaciones de la creatividad y de la inteligencia humana que llamamos

"ciencias". Por qu Introduccin al Pensamiento Cientfico en el inicio de los estudios superiores? En este curso se trata de tomar en cuenta las particularidades del conocimiento cientfico tal como los problematiza la epistemologa. Para ello, y antes de iniciar el estudio de esta disciplina, conviene recordar que nuestro concepto de ciencia es un producto histrico de ascendencia griega. Los babilonios, los egipcios, los aztecas y los incas reunieron y registraron mucha informacin sobre los fenmenos naturales, con estos conocimientos pudieron alcanzar grandes logros culturales y, sin embargo, es discutible

si este conocimiento puede considerarse "cientfico". Por otra parte, podemos admitir que la ciencia es un producto social bastante reciente si lo comparamos con otros logros culturales ms antiguos como el arte o la literatura. Dentro de esta tradicin, para que haya ciencia, es necesario que estos conocimientos formen un sistema ordenado segn criterios lgicos. En la conformacin de la idea de ciencia, el epistemlogo argentino Juan Samaja[1] reconoce dos momentos: a) Un primer momento que correspondera al nacimiento de la ciencia en el sentido ms amplio de la episteme tal como la concibe Platn (427-347 a.C.), como un conocimiento

acorde a la razn o logos, como un poder que permite controlar sus fundamentos. De all el uso del trmino logos para designar disciplinas cientficas como antropologa, psicologa, etc. En este perodo se puede ubicar el pasaje de la concepcin mitolgica del mundo a la concepcin epistemolgica y los hroes de esta conquista fueron los filsofos griegos Tales de Mileto (639-547 a.C.), Parmnides de Elea (nacido entre el 530 y el 515 a.C.), Pitgoras de Samos (aproximadamente 582-507 a.C.) y, especialmente, Aristteles (384-322 a.C.), entre otros. b) Un segundo momento que corresponde al nacimiento de las

ciencias positivas segn se concibe a partir del desarrollo de las ciencias experimentales con Galileo Galilei (1564-1642), Isaac Newton (16431727), Blaise Pascal (1623-1662), Ren Descartes (1596-1650), para nombrar slo a los primeros. Desde este punto de vista, el conocimiento cientfico es el resultado de una prctica que consiste bsicamente en "teorizar" acerca de distintas entidades, empricas o formales. "Argumentar" y "teorizar" no son sinnimos, sin embargo con estos trminos sealamos dos actividades ntimamente conectadas ya que la construccin de una teora supone utilizar argumentos que la fundamenten o

justifiquen. Realizar una actividad no implica necesariamente estar capacitado para formular las reglas de estas prcticas ni para criticar algunas de las creencias vigentes y menos para conocer su historia y las distintas interpretaciones que puedan hacerse de ellas. Est claro que se puede ser un excelente director de cine y un psimo crtico de arte o un destacado jugador de tenis y un mal comentarista deportivo. Salvando las distancias, de un modo parecido teorizar, como hablar o argumentar, es una actividad que puede realizarse correctamente sin realizar explcitamente una revisin filosfica de sus presupuestos. Los cientficos y los estudiantes de las distintas ciencias, a

menudo, suelen mirar con desconfianza a los epistemlogos y esta desconfianza en parte se justifica por lo abstracto de sus formulaciones que, a veces, parecen conducir a discusiones bizantinas completamente alejadas de las prcticas cientficas concretas y de los acuciantes dilemas morales a los que se enfrentan. Por otra parte, algunos epistemlogos sostienen que para hacer epistemologa es necesario ser cientfico o estar familiarizado con la produccin de teoras en el interior de alguna disciplina, mientras que otros defienden la autonoma y especificidad del discurso filosfico y el cientfico. Volviendo a la analoga anterior, equivale a la discusin acerca de si,

para ser crtico literario, es necesario haber escrito una novela o para discutir sobre cine, ser cineasta. Ms all de estas polmicas, podemos admitir que es deseable que un buen cientfico sea capaz de poder reflexionar crticamente acerca de su objeto de estudio y de evaluar los resultados sociales de su prctica cientfica. La epistemologa se ocupa de formular teoras filosficas sobre teoras cientficas buscando as evitar la aceptacin dogmtica y acrtica de los logros de las distintas ciencias. Desde esta concepcin, renunciamos a asentar una nica definicin de "ciencia". Por el contrario, de lo que se tratar es de presentar distintas posiciones epistemolgicas que en el

desarrollo del conocimiento occidental dieron lugar a distintas concepciones de las ciencias. Un curso de epistemologa en el inicio de la formacin universitaria de los futuros cientficos o profesionales involucrados en el tratamiento de teoras cientficas tiene el sentido de hacer explcitos los problemas que afectan a la produccin de teoras y a las particularidades de esta actividad de tan alto y controvertido impacto social. A menudo se asocia el conocimiento cientfico al progreso de la humanidad y al logro de desarrollos tecnolgicos que permiten aumentar nuestra calidad de vida, como son la elaboracin de vacunas o drogas para paliar

enfermedades antes incurables, pero tambin es cierto que puede presentarse a la ciencia como responsable de la contaminacin del planeta y de las distintas amenazas blicas. En este curso no pretendemos tomar partido por ninguna concepcin pesimista u optimista acerca de la ciencia y sus promesas o amenazas sino, por el contrario, ya antes de adoptar una posicin, esperamos que el alumno refuerce las competencias argumentativas para fundamentar alguna. En nuestro tiempo, constatamos que el viejo ideal griego del conocimiento del mundo como parte de una empresa desinteresada no da cuenta del concepto actual de ciencia, ya que de ella se

espera alguna "utilidad" para ordenar los fenmenos de un modo "eficaz". Desde este punto de vista, la aceptabilidad de las teoras se sustenta en la obtencin de xitos tecnolgicos y la verdad cientfica queda asimilada a la utilidad. La ciencia adquiere el alto grado de prestigio a partir de su utilidad social, en especial a partir de su interdependencia con el sistema productivo. Desde la concepcin positivista (que recorreremos en varios captulos de este curso) se concibe a la ciencia como un producto de la razn, que suministra el conocimiento necesario para dominar la naturaleza y controlar el orden social. Segn esta idea, la ciencia es la principal

herramienta del progreso humano. No se trata de negar este supuesto, sino de constatar su presencia y llevarlo al campo reflexivo. Bsicamente, en nuestras concepciones acerca de la ciencia creemos que el conocimiento cientfico no nos fue otorgado como un don de los dioses, sino que es el resultado de las actividades humanas que, al igual que otras adquisiciones, son productos falibles y perfectibles, que en ningn caso sus resultados son inocuos, por el contrario, que deben ser cuestionados y revisados tanto por la comunidad cientfica como por el resto de la comunidad. Esperamos que este curso brinde herramientas conceptuales a las

nuevas generaciones de cientficos y profesionales en esta tarea central para la construccin de una ciudadana activa. Un curso, una aventura llevados de la mano de Alicia Recurrir a la figura de Alicia en el pas de las maravillas[2] en un curso de epistemologa no es una originalidad. Por el contrario, los personajes presentes en el cuento son conos de nuestra cultura y su autor es objeto de culto en el mundo de los filsofos y cientficos desde la aparicin del cuento, en Inglaterra, en 1864. Podemos advertir que en esa poca haba una fuerte revisin de los conceptos centrales de la fsica y la biologa y que

esta obra es contempornea a la publicacin de El origen de las especies por medio de la seleccin natural, o la preservacin de las razas preferidas en la lucha por la vida, publicada en 1859 por otro cientfico revolucionario ingls, Charles Darwin. Para el gran pblico, especialmente el infantil, el xito editorial de Alicia fue inmediato. Tanto en el mundo de la literatura como en el de la ciencia, distintos autores reconocieron en el uso y la creacin de nuevos lenguajes la puerta de entrada a un experimento revolucionario. Si optamos por definir al lenguaje como un conjunto de smbolos regidos por reglas, la lgica que usamos es la crcel del lenguaje ya

que nos confina al uso de algunas, consagradas como las reglas "correctas". Lewis Carroll nos abre las puertas de la celda para que salgamos a "jugar", a usar otras reglas, otras leyes, otros principios, mostrando as todo tipo de situaciones paradjicas y demenciales. A juicio del escritor argentino Jorge Luis Borges (18991986), las aventuras de Alicia parecen arbitrarias e irresponsables, luego advertimos que encierran "el secreto rigor del ajedrez y de la baraja", es decir, denuncian la naturaleza convencional y arbitraria de estas reglas, lo que permite experimentar mentalmente, imaginativamente, la posibilidad de transitar distintos

rdenes. La inocente mirada de Alicia, perpleja pero irreverente, consigue un efecto desestructurador sobre nuestras convicciones profundas al poner en descubierto el absurdo de las situaciones que no encuadran perfectamente en las reglas conocidas. Bajo un disfraz de locura, el relato disimula, detrs de las actitudes candorosas de una nia, la agudeza de una inteligencia crtica capaz de ridiculizar y mostrar en toda su insignificancia y precariedad algunas de las categoras ms respetadas por las ciencias y la lgica. Acompaando a Alicia en los pasajes cruciales del cuento, encontramos la figura tutelar del

gato de Cheshire que, al modo de un alter ego, aparece y desaparece a voluntad (hasta quedar solamente su sonrisa) y es el nico que toma en broma todo lo que ocurre. En este curso aparecern varias veces alusiones a pasajes de la obra de Lewis Carroll as como fragmentos de textos de otro admirador de Alicia, Borges. Ambos consagraron gran parte de su genio creativo a la imaginacin de situaciones donde aparecen fuertemente cuestionadas las reglas y presupuestos del lenguaje. Conmover las creencias profundas acerca de nuestras habilidades lingsticas provoca perplejidad y una cierta sensacin de inseguridad, ya que con ello ponemos en

tela de juicio nuestras certezas y seguridades acerca del mundo. Revisar los mecanismos ntimos de la construccin de teoras implica superar las concepciones intuitivas para acceder (haciendo un esfuerzo contraintuitivo) a la comprensin de concepciones alternativas acerca de la relacin entre el lenguaje y la realidad. Poner una cua entre estos dos planos es uno de los logros de la epistemologa contempornea. Este pasaje es necesario para acceder al territorio de la epistemologa, especialmente para deslindar entre el plano del lenguaje (formal) y el plano de la realidad (emprico) al momento de diferenciar entre las ciencias formales y fcticas y

advertir que, a lo largo de todo el curso, estamos haciendo afirmaciones acerca de teoras y que el conocimiento de la realidad est mediado por el conocimiento de teoras cientficas, que no se da de un modo inmediato ni natural y que, por el contrario, implica la "desnaturalizacin" de nuestras certezas para transformarlas en hiptesis conjeturales y provisorias. Alicia en el pas de las maravillas: [3] resea de la obra Quin escribi un libro de cuentos dedicado a su amiga predilecta, Alice Liddell? Podemos afirmar que fue el Reverendo Charles Lutwidge Dodgson

(1832-1898), profesor de Lgica y Matemtica de la Universidad de Oxford, autor de libros tcnicos, y/o podemos inclinarnos a pensar que el seudnimo "Lewis Carroll"
[4]

esconde a otro personaje, no yuxtapuesto sino distinto y dividido del anterior. En este caso decidir por la "y" o la "o" no es un detalle. Bajo el seudnimo de Lewis Carroll encontramos, en las aventuras de Alicia, plasmadas las pesadillas de su autor, el lgico-matemtico ingls Charles Dodgson, tambin autor de tratados acadmicos de geometra y de lgica.

Las aventuras de Alicia comienzan cuando, en una soleada tarde de verano, la nia ve pasar a un conejo -con chaleco, reloj y guantes- y movida por la curiosidad lo sigue hasta una madriguera en la que desciende lentamente hasta caer en un pozo muy profundo sobre un montn de palos y hojas secas. Carroll, como matemtico, manifestaba enorme inters por las teoras de la gravitacin universal, que estaban siendo objeto de gran desarrollo en su tiempo. Se ha sealado la similitud entre la cada de Alicia en la madriguera y un ejemplo empleado por Einstein para ilustrar algunos aspectos de su teora. Podemos decir que las aventuras se inician a partir de lo que

podramos denominar un "resorte moral", un impulso que sobrepasa la capacidad reflexiva y voluntaria del personaje. Alicia se ve superada por la rigidez mecnica de la gravedad que la coloca al margen de una accin deliberada y libre puesto que queda reducida a la condicin de "cosa que cae", aunque esta cada parece responder a otras leyes distintas a las conocidas por la fsica de la poca ya que cae lentamente en una situacin que llamaramos "antigravitatoria", hasta llegar al fondo de la madriguera. Alicia comienza, entonces, una serie de metamorfosis a partir de lo que bebe o come: se achica hasta casi desaparecer

o se agranda y pierde de vista sus propios pies. Tantos cambios imprevistos provocan el llanto de la nia, la que comienza a dudar de si seguir siendo la que era antes de caer en la madriguera. La angustia de Alicia se produce frente a la posibilidad de la prdida de identidad o de unidad ante los sorpresivos cambios, que ella identifica con la prdida del nombre. Este tema es retomado, en la conversacin entre Alicia y la Oruga, la que le da consejos para sobrellevar los cambios corporales radicales, ya que ella es experta en metamorfosis profundas. El efecto "bola de nieve", de

acumulacin de situaciones, presente en toda la obra, se produce a partir de varias metamorfosis que trastocan la condicin inicial y que consigue neutralizar el asombro y la perplejidad del lector, el que se ve obligado a dejarse llevar por una dinmica vertiginosa. El encuentro de Alicia con la Duquesa y con el gato de Cheshire es uno de los captulos ms conocidos de la obra. El gato de Cheshire puede ser visto como el animal emblemtico de la pequea herona de esta historia, ya que la acompaa desde un trasfondo fantasmal donde aparece y desaparece; entre todos los animales es el nico capaz de

enfrentar a la Reina con su sonrisa sardnica. Alicia se encuentra por segunda vez con el sonriente gato de Cheshire posado sobre la rama de un rbol -la primera vez lo ve en la cocina de la Duquesa- y ste le aconseja conocer al Sombrerero loco y a la Liebre de marzo. En este episodio aparece cuestionada la idea del tiempo (recordemos que al inicio aparece cuestionado el espacio) y se presentan curiosos relojes que miden el ao o los das del mes en lugar de las horas. Luego de una conversacin disparatada, Alicia da por terminada "la merienda de locos", en la que el Tiempo se ha detenido a las seis en punto de la tarde, lo que condena a los personajes a

repetir eternamente la ceremonia del t sin darles tiempo siquiera a lavar las tazas. Nuevamente, luego de esta extraa merienda, la nia se ve arrastrada por un impulso o resorte que la lleva al Juego de Croquet de la Reina. En este captulo, Carroll ridiculiza los aspectos esenciales del espritu ingls de su poca y presenta a una Reina desptica (recordemos que estaba en el trono la Reina Victoria y que su personalidad impregn la poca dando lugar al adjetivo "victoriano" para referirse a cosas que sucedieron en su reinado), dispuesta a cortar la cabeza de todo el mundo. En el campo de Croquet de la

Reina de Corazones, las pelotas son erizos vivos y los palos unos incmodos flamencos que intentan escapar, los arcos los forman los soldados de la Reina curvando sus cuerpos de naipes. Todos juegan al mismo tiempo sin esperar su turno, peleando con los erizos que se mueven caprichosamente por todo el campo haciendo desaparecer el resultado de la jugada. Para alegra de Alicia se hace visible la cabeza del gato de Cheshire y, cuando terminan de aparecer las orejas, le cuenta entre asombrada y divertida: "No estn jugando limpiamente", empez diciendo Alicia con tono quejumbroso, "y se estn peleando todo el tiempo, de forma que no hay quin oiga nada..., y

adems, nadie hace demasiado caso a las reglas del juego; parece como si no tuviera ninguna, o, en todo caso, si las hay, nadie parece que las est siguiendo...[...] es tan seguro que va a ganar la Reina que no vale la pena que los dems sigan jugando." Un juego de esta ndole, sin reglas, donde se sabe de antemano quin va a ganarlo, donde no se distingue entre la destreza y la pura casualidad, no puede ser puesto como modelo sino como contrafigura del juego social identificado entre los ingleses como fairplay. En los tres captulos que siguen ("Historia de la Tortuga Artificial", "La cuadrilla de la

Langosta" y "Quin rob las tartas?") se multiplican los juegos de palabras, la ridiculizacin de los cuentos con moraleja y el sistema escolar ingls. Carroll explota al mximo los efectos cmicos que crea el lenguaje a travs de distintos mecanismos que, en muchos de los casos, son intraducibles. La crtica social se ve reforzada por las caracterizaciones zoomrficas de los personajes. Estos animales (reales o fantsticos) que hablan con la nia, cuestionan los parmetros de la racionalidad en un territorio donde se diluye el lmite entre la razn y la locura. En este sentido puede asociarse a Carroll con la tradicin iniciada por Esopo y continuada por La Fontaine,

hasta el mismo Walt Disney. Los animales que rodean al personaje son puestos como contrafiguras, unas veces, o como prolongacin o alter ego del protagonista en otros casos. En el ltimo episodio, Alicia se ve involucrada en un proceso judicial disparatado que presenta algunas notas en comn con el Juego de Croquet de la Reina. La acusacin recae sobre la Sota de Corazones por robar unas tartas de la Reina. Nuevamente la ausencia de reglas preexistentes junto al arbitrario y caprichoso ejercicio del poder por parte de la Reina posibilita la aparicin del absurdo: "En este momento, el Rey, que haba

estado muy ocupado escribiendo apretadamente en su libreta de notas, exclam: "Silencio!", y ley a continuacin lo que acababa de anotar: "Regla Nmero Cuarenta y Dos: Todas las personas que midan ms de una milla de altura habrn de abandonar la Sala". Todos miraron a Alicia. "pero si yo no mido una milla de altura!", dijo Alicia. "Ciertamente que s!", declar el Rey. "Casi dos millas", aadi la Reina. "pues lo que es yo, no me marchar en ningn caso", anunci Alicia; "adems, esa regla no vale porque se la acaba de inventar usted". "Es la regla ms antigua de todo el

libro", asegur el Rey. "Entonces sera la primera y no la cuarenta y dos", acus Alicia. Las actitudes contradictorias del Rey y la Reina, junto a la sorpresa de la protagonista frente a un nuevo cambio de tamao, precipitan el final cuando Alicia asume una decisin vital, transformadora, que revela la precariedad y fragilidad de las convenciones admitidas: "A callar!", vocifer la Reina ponindose morada de rabia. "pues no me callo", respondi Alicia. "Que le corten la cabeza!", chill la Reina con toda la fuerza de sus pulmones; pero nadie hizo el menor

movimiento. "Quin les va a hacer caso?", dijo Alicia (que para entonces ya haba recobrado su estatura de todos los das). "Si no son ms que un mazo de cartas!" Alicia despierta del sueo y vuelve al punto de partida. El mundo del absurdo, en el que estuvo sumergida, queda abolido. La seguridad amenazada, la tranquilidad perdida por la irrupcin de un "otro mundo" incomprensible se diluyen en una sonrisa cuando logra restablecerse el orden conocido y todo se encarrila nuevamente en la normalidad. Podemos ver, en el final de la obra, que el personaje recupera la

condicin de persona a partir de un acto de voluntad, de rebelda: la pesadilla termina en el momento en que una decisin de no-sumisin, de sublevacin aniquila la coaccin de un mundo donde estaba reducida a la condicin de ttere y le permite recuperar su libertad, su autonoma. Propuesta de un plan de estudio para los alumnos La lectura del material de estudio requiere la puesta en prctica de distintas habilidades por parte de los alumnos. Una competencia necesaria es la de deslindar los temas centrales de aquellos que los sustentan o justifican. Es decir, es necesario identificar dentro

de la bibliografa, los conceptos y nociones que vertebran los contenidos del curso y aquellos que son comentarios sobre otras posiciones, polmicas, distintos criterios de clasificacin, enfoques, etc. En este sentido, el abordaje del material de estudio supone una "estrategia de lectura", una jerarquizacin de los conceptos. En virtud de un mejor modo de organizacin para el estudio, sugerimos que: - Tome contacto con el programa de la materia, en el que encontrar los contenidos y la bibliografa, identificados por unidad. Lea la Introduccin a la materia y las presentaciones de cada unidad y luego

la bibliografa obligatoria. - Realice las actividades propuestas en esta gua de estudio, buscando la informacin que se requiera. En cada unidad, adems del libro de lectura obligatoria, se encuentra sealada una bibliografa de consulta. Tome en cuenta que en el nivel de estudios superiores es necesario recurrir a fuentes bibliogrficas reconocidas. Internet es un recurso muy accesible para encontrar informacin pero, como toda herramienta, debe ser usada con cierta destreza, en este caso, para poder determinar aquellas fuentes bien documentadas de otras poco autorizadas. La bsqueda de informacin requiere un

entrenamiento y ste puede ser el momento de adquirir esta competencia, necesaria para continuar estudios universitarios. Cuando dude acerca de la confiabilidad de la fuente encontrada puede consultar a los docentes en las distintas instancias de tutoras presenciales y virtuales. - A partir de las situaciones problemticas, propuestas por las actividades, recurra al material de estudio y realice esquemas, mapas conceptuales, resmenes y todo tipo de actividades que permitan organizar estos contenidos, unidad por unidad. En esa tnica, conviene localizar en espacio y tiempo a los autores principales que aparezcan

mencionados. Los ideales cientficos corresponden a perodos histricos determinados y se relacionan estrechamente con ideales religiosos, estticos, polticos y sociales. Conviene tomar en cuenta el contexto histrico de los autores y de las concepciones epistemolgicas que sostienen para poder comprender mejor sus planteos y el mundo de ideas que les da sentido. Para ello, se propone realizar una lnea histrica que podr ir completndose con el orden cronolgico de los autores, a medida que vaya avanzando en el curso. Advierta que la primera vez que se los nombra aparecen entre parntesis sus datos de nacimiento y

muerte. - Prepare con anticipacin las actividades y tngalas a mano para despejar sus dudas a travs de los distintos medios de comunicacin ofrecidos por UBA XXI (correo electrnico, tutoras presenciales, foros, etc.). Recuerde que el mejor resumen o esquema es el que realiza usted mismo y que el modo de aprender a hacerlo es intentndolo hasta adquirir esta destreza. Si se nos permite una analoga con la adquisicin de habilidades fsicas como "andar en bicicleta" o "nadar", la adquisicin de las competencias para estudiar es tambin cuestin de "entrenamiento".

La construccin del conocimiento es un logro social que se alcanza en el contacto con los materiales de estudio y en el dilogo con otras personas. Realizar un curso a distancia no es equivalente a estudiar en solitario. Las tutoras presenciales y el campus virtual permiten el encuentro con otros alumnos, con los docentes a cargo del dictado de la materia y con los pedagogos encargados de orientar la resolucin de problemas de distinta ndole. Recomendamos enfticamente usar estos recursos previstos para la realizacin de este curso y, en la medida de lo posible, conformar una comunidad de estudio con otros compaeros del curso. Los invitamos, adems, a escuchar los

programas de radio y los recursos sonoros disponibles en la Mediateca del Campus, en los que se abordan diversos temas de la materia. Respecto a las Actividades de aprendizaje Las actividades propuestas en cada unidad estn orientadas a la problematizacin de los contenidos del curso y no son indicativas del tipo de ejercitacin que se tomar en los parciales. Por el contrario, tienen el sentido de presentar problemticamente distintos "casos" para ilustrar diversos aspectos de las teoras abordadas y profundizar en la comprensin de los temas. Lo importante, en estos ejercicios, no es "dar con la respuesta

correcta", sino pensar y poder justificar. "Justificar", en este caso, tiene el sentido de hacer explcitos los aspectos tericos de los distintos temas (definiciones, reglas o criterios de clasificacin vinculados a los conceptos). En ningn caso esta aplicacin ser meramente mecnica ya que se tratar de interpretar estos conceptos en base a algunos criterios y cabr la posibilidad de que se privilegien algunos por encima de otros. Por ejemplo: en base a una clasificacin de distintas funciones del lenguaje preguntamos en dnde ubicara distintos casos, se debe tratar de poner en juego esta clasificacin y despus dar cuenta del criterio que se adopt en cada

eleccin. Aqu lo importante es constatar si se consider el criterio de la clasificacin. Sabemos que hay una brecha entre la definicin de un concepto y su aplicacin a casos individuales. Para usar un ejemplo conocido, digamos que, por estrictamente definida que est la definicin de "gol" en un reglamento de ftbol, es bastante comn que se generen fuertes polmicas al momento de decidir si una jugada fue un gol o no, es decir, cul es la interpretacin del reglamento. Ms que acertar con la respuesta correcta, lo que buscan estos ejercicios es problematizar los conceptos presentados en el curso y mostrar la posibilidad de aplicacin sobre casos

donde, ms all de lo anecdtico y particular de la situacin presentada, pueda llegarse a una comprensin ms profunda de la teora y no la mera asimilacin de datos. Conviene recordar que los temas presentados son de ndole filosfica y motivo de disputas tericas en las que pueden sostenerse posiciones distintas, o abordarse los problemas desde ngulos diferentes segn distintas decisiones metodolgicas.

Temas de la Unidad 1.1. Lenguaje y teoras cientficas 1.2. Lenguaje y realidad 1.3. Uso y mencin del lenguaje 1.4. La Semitica 1.5. Nombrar y clasificar. Vaguedad y ambigedad 1.6. La definicin Bibliografa obligatoria Unidad 1 en la Gua de Estudio de IPC, producida por UBA XXI y editada por Eudeba, a partir de la edicin de 2010. Asti Vera, C. y Ambrosini, C. (2009). Captulo 1. En Argumentos y teoras. Aproximacin a la Epistemologa.

Buenos Aires: Educando. BIBLIOGRAFA COMPLEMENTARIA Copi, I. (1984). Captulos 2 y 4. En Introduccin a la lgica. Buenos Aires: Eudeba. Presentacin de la Unidad

Los doce jurados estaban muy


atareados escribiendo en sus pizarras. "Questn haciendo?" susurr Alicia a Grifo; "no pueden estar nada, puesto que el juicio no ha empezado todava ". "Estn escribiendo sus nombres ", cuchiche Grifo, "no vaya a ser que los olviden antes de que acabe el juicio ". "Pero es que son imbciles?", empez a

decir Alicia, con una voz muy indignada pero se call a tiempo al ver que el Conejo Blanco gritaba "Silencio en la Sala!", y al ver que el rey se calaba las gafas y miraba severamente de un lado para otro intentando descubrir quin era el que estaba hablando. (Carroll, L., Alicia en el pas de las maravillas)

Por qu comenzamos este curso abordando el tema del lenguaje? En principio, aceptamos que toda teora

cientfica es una construccin lingstica. El lenguaje es el instrumento bsico para la construccin de teoras ya que es impensable una teora "inefable", es decir, una teora que no pudiera expresarse en algn lenguaje. La revisin de las estructuras lingsticas, presentes en las teoras cientficas, se har desde dos disciplinas emparentadas: la Semitica (estudio de los signos) y la Lgica (estudio de las estructuras del lenguaje). Estas dos disciplinas actualmente tienen desarrollos tericos autnomos aunque remiten a un origen comn en las teoras griegas. Comenzamos este recorrido con la concepcin mgica que identifica al nombre con el alma o espritu de la cosa

nombrada, antes de mencionar las ideas de Platn presentes en la obra Cratilo para caracterizar la teora esencialista. A continuacin estudiaremos la posicin revolucionaria del filsofo y telogo franciscano ingls Guillermo de Ockham (1298-1349), quien introduce, en el siglo XIV, una novedosa teora acerca del signo lingstico, en oposicin a la teora esencialista de Platn. La posicin de Ockham, caratulada como "nominalismo", nos introduce en las nociones de la Semitica al proponer un enfoque rival a la concepcin del lenguaje del esencialismo utilizando una nueva teora acerca de los signos. Advertimos en este punto que, en el texto de la bibliografa, aparecen

distintas referencias al semilogo italiano contemporneo Umberto Eco (n. 1932) quien rinde homenaje a Ockham en su novela El nombre de la rosa y problematiza all y en otros tratados de semiologa algunos de los problemas ligados al campo semntico y pragmtico del lenguaje. Es importante comprender la diferencia entre uso y mencin del lenguaje para destacar el nivel metalingstico de la Epistemologa, ya que no advertir la distincin entre las teoras, consideradas como entidades lingsticas, y la realidad, da lugar a todo tipo de confusiones y paradojas. En el terreno de la Semitica veremos, en primer lugar, la distincin del

filsofo y cientfico estadounidense Charles Peirce (1839-1914) entre signo natural, cono y smbolo a los efectos de llegar a comprender el sentido de la definicin del lenguaje como "conjunto reglado de smbolos". En segundo lugar, nos aproximaremos a los distintos temas que aluden a la divisin del estudio de los signos: la dimensin sintctica (acerca de las relaciones entre signos y las reglas que los ordenan), la semntica (acerca de las relaciones entre el signo y sus significados) y la pragmtica (acerca de la relacin entre el signo y sus intrpretes o usuarios). Al estudiar la dimensin semntica, nos detendremos en la nocin de "trmino"

usado para nombrar a una de las estructuras lgicas y con ello entramos ya en el territorio de la Lgica (para la cual los trminos son unidades de significacin). En el terreno de los trminos (signos lingsticos, nombres, smbolos) se distingue entre designacin, extensin y denotacin, considerados como partes del significado. Con la dimensin pragmtica y la distincin entre las funciones del lenguaje, se introduce el concepto de "proposicin", otra de las estructuras lgicas que permite predicar el concepto de "verdadero" o "falso" a partir de tener alguna teora acerca de la verdad proposicional. Para ello, tomaremos la

definicin de proposicin del filsofo austraco Ludwig Wittgenstein (18891951) citado en la bibliografa obligatoria: Lo que engrana con el concepto de verdad (como una rueda dentada) eso es una proposicin [...] Y lo que es una proposicin est en un sentido determinado por las reglas de formacin oracional (de la lengua castellana, por ejemplo) y en otro sentido por el uso del signo en el juego del lenguaje. (Asti Vera y Ambrosini, 2009. 26-27) El concepto semntico de verdad del lgico polaco Alfred Tarski (19011983) es presentado para destacar uno de los intentos, en la semntica contempornea, de definir el concepto de verdad

proposicional evitando la incursin en paradojas. Adems de los temas mencionados, tendr que estudiar las distintas operaciones lingsticas, fundamentales en el uso de un lenguaje, en especial en el caso de los lenguajes cientficos: nombrar, clasificar, definir. Es muy importante reconocer sus particularidades antes de entrar en el anlisis de la estructura de las teoras cientficas. La clasificacin de las ciencias que se encuentra al inicio del captulo 1 de la bibliografa, servir de parmetro o mapa conceptual til para anticipar los temas que se trabajarn en las diferentes unidades del programa. Como sostiene

Borges, toda clasificacin es arbitraria y conjetural. Esta clasificacin de las ciencias, entre formales y fcticas, no se la presenta con la intencin de legitimarla como la clasificacin correcta, sino como la que, metodolgicamente, utilizamos en este curso para ordenar los distintos temas que se presentan.

Actividades de aprendizaje
ACTIVIDAD 1. LENGUAJE Y
REALIDAD En el punto 1.2. del captulo 1, se

presentan dos posiciones antagnicas acerca del modo de justificar la relacin entre el lenguaje y la realidad: Platn y Guillermo de Ockham. Estas actividades estn orientadas a destacar la diferencia entre estas dos posiciones.

1.1. La suppositio materialis en El nombre de la rosa de Umberto Eco[5]


Lea los siguientes prrafos y luego responda las preguntas que aparecen a continuacin.
En la novela histrica El nombre de la rosa, el semilogo italiano Umberto Eco (1932) rinde homenaje a la figura de Guillermo de Ockham a travs del personaje de un monje

franciscano ingls, Guillermo de Baskerville. Para Francisco Bertelloni,

[6]
el tema central del relato es el totalitarismo de la verdad que puede mover a los hombres a matarse unos a otros. Segn Bertelloni, este tema le permite a Eco vincular una trama filosfica e histrica -las luchas doctrinales en la Edad Media- con una trama policial, una serie de asesinatos cuyas vctimas tienen un rasgo en comn: estn todos interesados en el mismo libro y mueren a causa de l. El relato comienza una maana de noviembre de 1327 en el norte de Italia y se desarrolla en siete das. La historia gira alrededor de la bsqueda del autor de los crmenes, un fantico que mata por extremo amor a la verdad pues no tolera que la cosmovisin cristiana sea sustituida por las doctrinas de un Aristteles recin descubierto, distinto al apropiado por la doctrina oficial de la iglesia. Se torna peligroso para la Teologa un Aristteles "que mira la tierra antes que el cielo" y debe ser detenido incluso mediante el crimen. Eco propone mostrar que en la Edad Media la filosofa no fue inofensiva, sino que tuvo efectos sociales relevantes. Eco apela al nominalismo empirista de Guillermo de Ockham, la nueva filosofa del siglo XIV, y la coloca como

trasfondo de los dilogos de Guillermo de Baskerville y su discpulo Adso. El trasfondo histrico se relaciona con las pretensiones del Papa Bonifacio VIII de conseguir la plenitudo potestatis, la totalidad del poder espiritual y terrenal en la Bula Unam Sanctam de 1302, promulgada 25 aos antes del inicio de la historia contada por Eco, donde define una teocracia papal: la iglesia como un cuerpo con una cabeza, el Papa. La Bula concluye que los reyes y emperadores deben subordinarse al poder espiritual para alcanzar la salvacin. Los franceses consiguen que se fije la sede del papado en Avin. La rebelin contra esta imposicin papal se concentra en la corte imperial de Luis de Baviera en Munich donde encuentran refugio Ockham, Marsilio de Padua y otros franciscanos que proponen volver a la iglesia primitiva y separar la iglesia del Estado. Al cuestionar el derecho de propiedad como parte del derecho natural, Ockham procura desmontar la estructura jerrquica de la iglesia eliminando el papel del clero como intermediario entre Dios y los hombres del mismo modo que, en el plano de la lgica, haba eliminado las entidades metafsicas como intermediarias entre Dios y los individuos, siempre singulares. En la abada, Guillermo se involucra en una trama policial (al modo de las novelas de detectives) donde se suceden los crmenes. Para resolver el problema de contestar quin es el asesino? Eco,

que parti de Ockham, va ms all del contexto medieval y hace de Guillermo de Baskerville un Sherlock Holmes con alusiones a Ludwig Wittgenstein y a la semitica contempornea sin dejar de homenajear, en la imagen del bibliotecario espaol ciego, Jorge de Burgos, a Jorge Luis Borges. Como no poda ser de otro modo, el semilogo Eco, ahora puesto en novelista, hace recaer la solucin del enigma y la resolucin del caso sobre un detalle lingstico, la suppositio materialis. Para ello, se trata de develar el sentido de la frase que permitira abrir una puerta de la biblioteca laberntica y encontrar la clave para ubicar al asesino.

La frase dice: Secretum finis Africae manus supra idolum age primum et septimum de quatuor.[7] Segn una primera interpretacin, la traduccin sera "el secreto del finis Africae consiste en que la mano sobre el dolo opera sobre el primero y el sptimo de los cuatro". Ahora el mensaje no tiene sentido ya que no existe el sptimo entre cuatro cosas.

a. Cul podra ser el sentido del mensaje? b. Dnde oprimira para abrir la

puerta? c. Cmo formulara el enigma para que fuese fcilmente comprensible? Ayuda: La puerta est oculta tras un espejo (dolo-imagen). Sobre la puerta finis Africae hay una leyenda que reproduce el versculo 4.4 del Apocalipsis de San Juan. "Super thronos viginti quatuor", y la palabra "quatuor" tiene siete letras.

1.2. La navaja de Ockham y el Dr. House


El siguiente texto presenta un episodio de la serie de televisin Dr. House. Lalo atentamente y luego responda las

preguntas que estn a continuacin:


La serie norteamericana de televisin Dr. House muestra a un mdico misgino y adicto al consumo de frmacos que, con una mente deductiva, resuelve los casos problemticos al modo de un detective o un semilogo: interpretando signos. Como en las novelas de Sherlock Holmes, el Dr. House interroga a los sntomas, como si fueran las pistas que conducen hasta hallar al culpable: la causa de la enfermedad. Las analogas entre Sherlock Holmes y el Dr. House son evidentes, incluso el Dr. Wilson representa una versin del Dr. Watson. Al igual que Holmes, House es un solitario, se relaciona de modo conflictivo con las dems personas y llega a ser incluso cortante y ofensivo con sus pacientes. Lo nico que le interesa es "resolver el caso" y todas sus emociones se concentran en ese fin. En el captulo 3 de la primera temporada de la serie, titulado originalmente "Occam's Razor", traducido al espaol como "Principio de parsimonia", se muestra el caso de Brandon, un chico de 22 aos que ha sufrido un colapso y es ingresado al hospital. En los ltimos das ha estado tosiendo y tiene un sarpullido. Ahora siente un dolor fuerte en el abdomen, nuseas,

fiebre y su presin arterial es baja. Un escner rpido y un examen no revelan nada, as que House y su equipo buscan alternativas. Coinciden en que ninguna enfermedad se corresponde con los siete sntomas que presenta el paciente. La teora de Foreman, uno de los mdicos del equipo que dirige House, es que el chico tiene una endocarditis, lo que explicara varios de los sntomas, excepto dos: la tos y el sarpullido. An as, es una posibilidad entre un milln. House examina la lista de sntomas del paciente y sugiere que dos enfermedades coinciden con los sntomas de Brandon, pero sigue siendo una posibilidad entre un milln. House apuesta 50 dlares con Foreman a que si el recuento de leucocitos del paciente aumenta, estar en lo cierto al decir que el chico tiene una infeccin. Cuando el nmero de glbulos blancos de Brandon disminuye, los dos mdicos se dan cuenta de que sus hiptesis eran errneas. House tiene una revelacin y le pregunta al doctor Wilson cul fue el primer sntoma de Brandon y sus sospechas se confirman: fue la tos. House tiene entonces otra revelacin, irrumpe en la habitacin del paciente y anuncia su diagnstico: intoxicacin por colchicina, sustancia con la que se

combate la "gota", una enfermedad que Brandon no sufre ni toma medicacin contra ella. Ahora el problema es saber: cmo ingiri la colchicina?, quin le administr esa sustancia? El enigma se resuelve cuando la madre de Brandon confiesa que le administr pastillas para la tos, sin consultar a los mdicos. Primero Foreman y luego House afirman en momentos cruciales: "Principio de parsimonia, la mejor explicacin es la ms simple".

a. En qu sentido se alude aqu a la navaja de Ockham? b. Qu enuncia, en su versin ms conocida, este principio? c. Una navaja se usa, normalmente, para cortar o rasurar la barba. Investigue a la barba de quin alude el nombre de "navaja de Ockham" y cul sera la analoga que propone la posicin nominalista y el hecho de cortar las barbas.

ACTIVIDAD 2. USO Y MENCIN


DEL LENGUAJE Lea los siguientes versos y luego conteste:
Si, (como afirma el griego en el Cratilo) El nombre es arquetipo de la cosa, En las letras de "rosa" est la rosa Y todo el Nilo en la palabra "Nilo". (Jorge Luis Borges, "El Golem")[8]

a. A qu griego se refiere Borges y cmo interpreta la frase "el nombre es arquetipo de la cosa"? b. Tomando en cuenta la distincin entre uso y mencin del lenguaje, justifique por qu las palabras "rosa" y "Nilo" aparecen en un caso escritas con comillas y en otro sin comillas? c. En este prrafo: Borges afirma que en las letras de "rosa" est la rosa y todo

el Nilo en la palabra "Nilo"? ACTIVIDAD 3. SEMITICA. ACERCA DE LOS SIGNOS. En esta actividad se trata de reconocer distintos tipos de signos segn una clasificacin cannica, para llegar al concepto de "smbolo" que es mencionado como parte de la definicin del lenguaje, tema central en esta unidad. Que el lenguaje sea definido como un "conjunto reglado de smbolos" implica, bsicamente, que no hay ningn vnculo natural ni necesario entre el nombre y el concepto u objeto nombrado, sino que este vnculo es arbitrario y convencional. Dice Charles Morris: "Los signos y la ciencia estn inextricablemente

conectados, habida cuenta de que la ciencia, simultneamente, ofrece a los hombres signos ms fiables y expresa sus resultados en sistemas de signos. La civilizacin humana depende de los signos y de los sistemas de signos, y al propio tiempo la mente humana es inseparable del funcionamiento de los signos, si es que, en verdad, la mentalidad misma no debe identificarse con ese funcionamiento." [9] Charles Morris llama "semiosis" al proceso por el cual algo funciona como signo. Es necesario que concurran tres factores para que algo funcione como signo: El vehculo sgnico, la seal, fenmeno

o cosa que acta como signo (S). El designatum, el significado del signo (D). El intrprete (I). Ejemplo: Un perro (I) responde al sonido del silbato (S) que implica la caza de ardillas (D). Le proponemos que en cada uno de los siguientes ejemplos identifique al signo (S), al designatum (D) y al intrprete (I), y coloque (S), (D) (I) en los parntesis, segn corresponda. Indique de qu clase de signo se trata. a. Al ver la sea del polica ( ), un conductor ( ) frena el auto ( ). b. Un peatn ( ) cruza la calle ( ) al ver el color de la luz del semforo ( ). c. Al ver una nube de humo negro que

sale de una ventana del primer piso ( ), los vecinos de un edificio ( ) salen de sus departamentos corriendo ( ). d. La fiebre del nene ( ) lleva al mdico ( ) al diagnstico de una hepatitis ( ). ACTIVIDAD 4. SEMITICA. ACERCA DE LOS TRMINOS. Uno de los temas centrales de esta unidad es la distincin entre designacin y denotacin. En el siguiente prrafo de Alicia en el pas de las maravillas, Lewis Carroll ilustra la idea de que los atributos no pueden andar solos (no tienen denotacin), es decir, la imposibilidad de concebir la subsistencia de un atributo luego de la desaparicin de la cosa.

Dale!", dijo el Gato, y esta vez se


desvaneci muy paulatinamente, empezando por la punta de la cola y terminando por la sonrisa, que permaneci flotando en el aire un rato despus de haber desaparecido todo el resto. "Bueno!Muchas veces he visto a un gato sin sonrisa", pens Alicia, "pero una sonrisa sin gato!... Esto es lo ms raro que he visto en toda mi vida!" (Carroll, L. 1990)

La posibilidad de poder definir trminos sin denotacin habilita, en el campo de las ciencias formales, la posibilidad de armar sistemas en los que tengamos trminos sin denotacin, es decir, sin

ninguna representacin en el mundo emprico (como la sonrisa sin gato), pero que con ellos podemos construir sistemas en la geometra, en la aritmtica o en la lgica. En el captulo segundo de la obra Symbolic Logic (1892), Carroll escribi que entre las clases de cosas estn las cosas imposibles y dio, como ejemplo, el caso de las cosas que pesan ms de una tonelada y un nio puede levantar. Advierta aqu que el autor sustancializa el lenguaje al hablar de cosas imposibles. Lo correcto sera decir "trminos que nombran cosas imposibles". Suponemos que lo hace para crear perplejidad ya que en el uso comn del lenguaje sustancializamos los

conceptos. Estos trminos que nombran cosas imposibles seran los trminos sin denotacin de las ciencias formales (nmeros, tringulos, lneas, etc.), entre otros casos. Hemos tomando como ejemplo el caso imposible de una "sonrisa sin gato". Le pedimos, ahora, que: a. Mencione al menos tres ejemplos de trminos que nombren atributos que no tienen denotacin. Justifique. b. Proponga otros tres ejemplos de trminos que tengan designacin en las ciencias formales y justifique por qu puede afirmarse que no tienen denotacin. ACTIVIDAD 5. NOMBRAR Y CLASIFICAR. VAGUEDAD Y

AMBIGEDAD. La vaguedad (significado impreciso de un trmino) y ambigedad (ms de un significado para el mismo trmino) son algunos de los temas tratados por la dimensin semntica de la semitica. Luego de leer el siguiente prrafo del libro Alicia en el pas de las maravillas, determine si el trmino "pican" es un caso de vaguedad o ambigedad.
S eguro

que estars pensando que por qu no te paso el brazo por la cintura", dijo la Duquesa tras una pausa; "la razn es que tengo mis dudas sobre el humor de ese pjaro flamenco que llevas ah. Qute parece siprobamos

el experimento?" 'A lo mejor le da unpicotazo", replic Alicia con cautela, sin ningutaagataa de intentar el experimento. "Muy cierto ", concedi la duquesa; "los flamencos y la mostaza, ambos pican; y la moraleja de esto es.... Dios los cra y ellos se juntan'".

(Carroll, L. 1990)

ACTIVIDAD 6. LA DEFINICIN
En esta actividad se busca llamar la atencin y trabajar las nociones de "definiendum" y "definiens" para advertir que un mismo trmino puede ser parte de un definiendum o de un definiens. Elabore definiciones para los siguientes trminos, haciendo corresponder al definiendum un gnero y una diferencia apropiados. Definiendum Definiens Gnero Diferencia a.- soltero b.- banquete a.- vstago a.- hembra b.- caballo b.- macho

c.- muchacho c.- hombre c.- casado d.- hermano d.- comida d.- no casado e.- nio f.- potro f.- oveja g.- hija h.- oveja i.- padre j.- gigante k.- muchacha l.- marido g.hermano h.- mujer f.- muy pequeo g.- joven e.e.- muy progenitor grande

ll.- cordero m.- yegua n.- enanito .- madre o.- pony p.- carnero q.- hermana r.- porcin s.- hijo t.- soltero

ACTIVIDAD 7. DEFINICIN. DESIGNACIN Y EXTENSIN.


Entre designacin y extensin hay una relacin inversa: a mayor designacin (notas definitorias), menor extensin (cantidad de ejemplares). "Hombre" tiene ms designacin que "animal" pero hay menos ejemplares de hombres que de animales. Dicho de otro modo: la especie tiene mayor designacin que el gnero y menor extensin. Le proponemos que ordene los siguientes trminos segn la mayor cantidad de ejemplares (extensin): a. Argentino, hombre, americano,

misionero. b. Fsico, hombre, cientfico, ser vivo. c. Hombre, futbolista, deportista, Martn Palermo.

Temas de la Unidad 2.1. Las leyes lgicas 2.2. Tautologas, contradicciones, contingencias 2.3. Los razonamientos. Verdad y validez 2.4. Argumento y consecuencia lgica. Reglas lgicas 2.5. La induccin y la analoga 2.6. Es formal la induccin? El intento de Carnap 2.7. Lgica informal y falacias materiales 2.8. Toulmin, sobre los usos

argumentativos Bibliografa obligatoria Unidad 2 en la Gua de Estudio de IPC, producida por UBA XXI y editada por Eudeba, a partir de la edicin de 2010. Asti Vera, C. y Ambrosini, C. (2009). Captulos 2 y 3. En Argumentos y teoras. Aproximacin a la Epistemologa. Buenos Aires: Educando. BIBLIOGRAFA COMPLEMENTARIA Copi, I. (1984). Captulos 1 y 3. En Introduccin a la lgica. Buenos Aires: Eudeba. Dez, J. A. y Moulines, C. U. (1999). Captulo 2. En Fundamentos de filosofa de la ciencia. Barcelona: Ariel.

Bunge, M. (1997). Captulo 3. En Epistemologa. Mxico: Siglo XXI. Klimovsky, G. (1994). Captulo 18. En Las desventuras del conocimientocientfico. Buenos Aires: AZ. Presentacin de la Unidad " Pero es que a m me gusta estar entre locos", observ Alicia. "Eso s que no lo puedes evitar", repuso el Gato; "todos estamos locos por aqu. Yo estoy loco; t tambin lo ests". "Y cmo sabes t si yo estoy loca?", le pregunt Alicia. "Has de estarlo a la fuerza", le contest el Gato; "de lo contrario no

habras venido aqu". Alicia pens que eso no probaba nada; pero continu de todas formas: "Y cmo sabes que t ests loco?" "Para empezar", repuso el Gato, "los perros no estn locos, de acuerdo?'' "Supongo que no", dijo Alicia. "Bueno, pues entonces", continu diciendo el Gato, "vers que los perros gruen cuando algo no les gusta, y mueven la cola cuando estn contentos. En cambio, yo gruo cuando estoy contento y muevo la cola cuando me enojo; luego estoy loco." (Carroll, L. Alicia en el pas de las maravillas)

Esta unidad est reservada a una revisin de la tercera estructura lgica: el "razonamiento". En la primera unidad ya hemos visto la primera estructura

lgica: los "trminos" y la segunda: las "proposiciones". Aqu estudiaremos el escenario formal y el escenario informal. En el primero, encontraremos los principios lgicos como paso necesario para introducir los conceptos de tautologas, contradicciones y contingencias que aluden a tipos distintos de proposiciones. Esta distincin entre proposiciones apunta a deslindar firmemente entre ciencias formales y fcticas dado el distinto tipo de enunciados que involucran sus teoras. Es importante reconocer la pertinencia del uso de los conceptos de verdad y validez que tambin apuntan a mostrar una distincin entre proposiciones y razonamientos.

Es muy importante reconocer las reglas lgicas y el concepto de razonamiento deductivo. A continuacin, estudiaremos la propuesta del filsofo alemn Rudolf Carnap (1891-1970) y su esfuerzo por dotar a la induccin (una forma de razonamiento invlido desde el punto de vista deductivo) de la rigurosidad propia de los procedimientos formales para establecer una "lgica inductiva". Tambin veremos el razonamiento analgico como un tipo de razonamiento no deductivo. Este tipo de razonamiento es un recurso argumentativo muy usado en el campo de la ciencia. En el escenario informal veremos diferentes manifestaciones de lo que se

llama "lgica informal" o "teoras de la argumentacin". Para ello estudiaremos, en principio, distintos tipos de falacias materiales, donde se distingue entre dos grupos con caractersticas particulares: a) falacias de inatinencia (o de inatingencia) y b) falacias de ambigedad. Es importante la revisin de este tipo de argumentaciones falaces ya que es muy provechoso reconocerlas en el lenguaje social, para no ser vctimas de manipulaciones retricas como as tambin para detectarlas en las argumentaciones cientficas. Esta unidad se completa con uno de los aportes relevantes en el terreno de las teoras de la argumentacin, surgidas a partir de la segunda mitad del siglo XX:

la teora de los usos argumentativos del filsofo estadounidense Stephen Toulmin (1922-2009). Aqu es importante que comprenda la distancia que toma Toulmin del planteo de la Lgica formal cuando parte de una analoga jurdica: los argumentos son comparables a las demandas judiciales y esta lgica que apunta a la "prctica" argumentativa sera una suerte de "jurisprudencia generalizada". El esquema argumentativo bsico distingue entre D (datos), C (conclusiones) y G (garantas). En el ejemplo del libro de la bibliografa, se muestra el esquema segn el cual la afirmacin "Juan Carlos S. naci en Salta, que es una provincia argentina" (D), permite inferir la

conclusin (C) "Juan Carlos S. es ciudadano argentino", a partir de la garanta (G) "Si una persona nace en una provincia de la Argentina, es argentina". En un esquema posterior, se agregan elementos al anlisis del argumento con los conceptos de "modalizador" (M), de la fuerza de la conclusin (presuntamente, probablemente) y las condiciones de excepcin o refutacin (E) donde pueden presentarse casos de excepcin ("a menos que haya sido naturalizado espaol"). Una nocin central que debe ser tomada en cuenta para comprender la propuesta de Toulmin es la de "campo argumentativo" ya que esta nocin permite deslindar entre "argumentos analticos", llamados

tambin "tericos", que seran independientes del contexto y asimilables a los razonamientos deductivos de la lgica formal, y los "argumentos sustanciales", tambin llamados "prcticos", que seran dependientes del contexto y de importante aplicacin en el mundo prctico. Debemos advertir que la presentacin del escenario informal se completa, en el texto de la bibliografa, con otras dos propuestas destacadas como son la del filsofo del derecho belga Cham Perelman (1912-1984), y la de los pensadores contemporneos holandeses Frans Van Eemeren y Rob Grootendorst que en este programa no se incluyen

como lectura obligatoria ni forman parte del material de lectura obligatoria pero que, de todos modos, y ms all de las exigencias temticas del curso, conviene leer y confrontar con la propuesta de Toulmin para completar la comprensin de las diferencias entre el escenario formal y el escenario informal.

Actividades de aprendizaje
ACTIVIDAD 1. ACERCA DE LA
ESTRUCTURA DE LOS RAZONAMIENTOS El razonamiento es la estructura lgica

que se compone de premisas y conclusin en las que, dada una serie de enunciados que actan como premisas, se infiere una conclusin. Los razonamientos pueden ser "vlidos" o "invlidos". En el libro Alicia en el pas de las maravillas, el gato de Cheshire irrumpe en las escenas de modo inesperado. En el prrafo citado en la presentacin de esta unidad, que aqu transcribimos, se argumenta acerca de la inevitabilidad de la locura en el pas donde se encuentra Alicia y para ello recurre a un extrao razonamiento. A. Lea atentamente el razonamiento. B. Identifique las premisas y la conclusin. C. Determine si este razonamiento es

vlido o invlido y justifique.

pero es que a m no me gusta estar


entre locos", observ Alicia. "Eso s que no lo puedes evitar", repuso el Gato; "todos estamos locos por aqu. Yo estoy loco; t tambin lo ests". "Y cmo sabes t si yo estoy loca?", le pregunt Alicia. "Has de estarlo a la fuerza", le contest el Gato; "de lo contrario no habras venido aqu". Alicia pens que eso no probaba nada; pero continu de todas formas: "Y cmo sabes que t ests loco?" "Para empezar", repuso el Gato, "losperros no estn locos, de

acuerdo?" "Supongo que no", dijo Alicia. "Bueno, pues entonces", continu diciendo el Gato, "vers que los perros gruen cuando algo no les gusta, y mueven la cola cuando estn contentos. En cambio,yo gruo cuando estoy contento y muevo la cola cuando me enojo; luego estoy loco. " (Carroll, L., 1990)

ACTIVIDAD 2. ACERCA DE LOS


CONCEPTOS DE VERDAD Y VALIDEZ Como afirmamos en la bibliografa de esta unidad, predicamos la verdad o

falsedad de las proposiciones y la validez o invalidez de los razonamientos. A menudo, en el lenguaje comn, usamos estos conceptos de un modo impreciso y mezclado. Es necesario reconocer la pertinencia del uso de estos dos conceptos para reconocer dos estructuras lgicas distintas: las proposiciones y los razonamientos. Complete las siguientes expresiones de modo que se conviertan en enunciados verdaderos: a. Si un enunciado tiene premisas falsas y conclusin verdadera, el razonamiento puede ser .................................................................. b. Si un razonamiento es vlido y tiene

premisas falsas, su conclusin puede ser .................................................................. c. Si un razonamiento tiene premisas verdaderas y conclusin verdadera, su forma puede ser .................................................................. d. Si un razonamiento tiene premisas falsas y conclusin falsa, su forma puede ser .................................................................. e. Si un razonamiento tiene premisas verdaderas y conclusin falsa, su forma es .................................................................. f. Para obtener una conclusin verdadera se requiere que el razonamiento sea....................................... y las premisas......................................

ACTIVIDAD 3. REGLAS LGICAS Y


FALACIAS 3.1. En los cuatro razonamientos siguientes, determine qu formas son deductivas (poner el nombre a la regla) y cules son falacias. a. Si sumerjo un cubito de hielo en un vaso de agua caliente, entonces el cubito se derrite. Sumerg el cubito, por lo tanto, se derriti. b. Si sumerjo un cubito de hielo en un vaso de agua caliente, entonces el cubito se derrite. No lo sumerg, por lo tanto, no se derrite. c. Si sumerjo un cubito de hielo en un vaso de agua caliente, entonces el cubito se derrite. El cubito no se derrite, por lo tanto, no lo sumerg.

d. Si sumerjo un cubito de hielo en un vaso de agua caliente, entonces el cubito se derrite. El cubito se derrite, por lo tanto, lo sumerg. 3.2. En los dos razonamientos presentados a continuacin hay uno deductivo y otro invlido, a pesar de que ambos concluyen en el mismo enunciado. Identifique cada caso y explique la diferencia entre ambos. a. Si Juan no repite el ao, entonces se va a Bariloche. Juan se va a Bariloche. Juan no repiti el ao.

b. Si Juan repite el ao, entonces no va a Bariloche. Juan fue a Bariloche (no es cierto que no va). Juan no repiti el ao. 3.3. Reglas lgicas y falacias formales Empleando el enunciado: "Si me anoto en el Ciclo Bsico Comn, entonces curso el primer ao de mi carrera", qu conclusin se obtiene construyendo: a. un Modus ponens, b. un Modus tollens, c. una falacia de afirmacin del consecuente y

d. una falacia de negacin del antecedente. 3.4. Conclusin correcta de decir "Si p entonces q" a. Si alguien promete a Juan: "Si termins el CBC este ao, te llevo de viaje a Europa" y, luego, lo lleva a Europa a fin de ao, aunque no haya terminado el CBC en un ao, habr mentido? Explique por qu. b. Si no termina el CBC en un ao y no lo lleva a Europa, habr mentido? Explique por qu. c. Si termina el CBC en un ao y no lo lleva a Europa, habr mentido? Explique por qu. 3.5. Diferencias entre decir "si p entonces q" a decir "si y slo si p

entonces q" Qu consecuencias distintas podran tener las siguientes afirmaciones? a. "Si me recibo de contador este ao, me caso". b. "Si y slo si me recibo de contador este ao, me caso". ACTIVIDAD 4. LA INDUCCIN 4.1. Suponiendo que las siguientes proposiciones son las conclusiones de distintos razonamientos inductivos, en cada caso determine cules podran ser las premisas. a. "Los planetas se mueven alrededor del sol". b. "Todas las modelos se casan con jugadores de ftbol". c. "Todos los mamferos tienen pelos".

d. "Todas las aves tienen plumas". e. "Los argentinos toman mate". 4.2. Dados los siguientes enunciados, consideradas las premisas de algn razonamiento inductivo, proponga la conclusin. a. "Las peras, manzanas, bananas y frutillas sirven para hacer un postre". b. "Los nmeros 2, 4, 10 y 22 son divisibles por dos". c. "El mejilln, las almejas, las ostras, los pulpos, los calamares y las sepias viven en el mar". d. "El Aedes aegypty y el Anopheles transmiten enfermedades". e. "Las ratas, los conejos y los ratones son vivparos". f. "Los peruanos, los bolivianos y los

coreanos venden ropa en la feria de La Salada". 4.3. Luego de leer los tres textos de Aristteles citados, determine de qu tipo de argumento se trata en cada caso. Deca Aristteles: a. "Si el mejor de los pilotos es el ms diestro y el mejor de los aurigas es tambin el ms diestro, podemos afirmar, en general, que en cada profesin el mejor es el ms diestro". (Tpicos, I, 12) b. "Los animales sin hiel son de larga vida; el hombre, el caballo, el mulo, son todos animales sin hiel; por lo tanto, todos los animales sin hiel son de larga vida". (Primeros Analticos, II, 23)

c. "Todo ocurre segn una finalidad, todo movimiento tiende a un fin, a veces ese fin se consigue y otras veces no. En el arte, el escriba, a veces, comete faltas en la escritura y el mdico, a veces, administra equivocadamente un remedio. As ocurre tambin que la naturaleza, cuando yerra la generacin, en vez de reproducir el tipo del engendrador, se desva de l y nace un monstruo (trata). El monstruo es un ser inacabado, informe, mal logrado (anaperon). Cuando esto ocurre, la simiente traiciona la forma desprendida del esperma del engendrador, que no logr dominar perfectamente la materia suministrada

por la hembra." (Fsica, II, 8) ACTIVIDAD 5. LA ANALOGA La analoga es un modo de razonar que, a partir de la comparacin entre dos o ms casos que tienen notas comunes, permite concluir para alguno de esos casos, algo admitido nicamente en alguno de los otros casos. Al igual que la induccin, es una forma de razonamiento invlido aunque en muchos casos puede resultar de inters pragmtico. Dados los siguientes casos, reconstruya un razonamiento por analoga determinando cul es la conclusin y cules son las premisas. a. "Ludwig Wittgenstein (Viena, Austria, 1889 - Cambridge, Reino

Unido, 1951) sola comparar el pensar con la natacin. As como en sta nuestros cuerpos tienen una tendencia natural a flotar sobre la superficie del agua, de modo que se necesita un gran esfuerzo fsico para sumergirse en el fondo, de igual modo en el pensar se necesita un gran esfuerzo mental para alejar a nuestras mentes de lo superficial." [10] b. "Uno de los atributos ms naturales de la mujer es el cuidado de los nios. De hecho, es correcto afirmar que los grupos en los que los hombres, y no las mujeres, cran a los hijos pequeos son totalmente excepcionales. Puesto que el enfermo y el discapacitado se asemejan en muchos aspectos a los

nios, pues no solamente son fsicamente dbiles y desvalidos, sino tambin psicolgicamente dependientes y narcissticamente regresivos, era bastante fcil suponer que las mujeres estn especialmente capacitadas tambin para el cuidado del enfermo." [11] ACTIVIDAD 6. LGICA INFORMAL Y FALACIAS MATERIALES En el diario La Vanguardia de Barcelona, el 4 marzo de 2010 aparece la siguiente noticia. Histricamente la comunidad de Madrid y la de Barcelona son antagonistas en muchos sentidos. En Catalunya, el Parlament est tratando la prohibicin de la corrida de toros a partir de los reclamos de las sociedades

defensoras de los derechos de los animales, mientras que en Madrid hay una gran resistencia ya que representa un punto de inters tambin turstico. Veamos el argumento de la presidenta de la comunidad de Madrid en defensa de las corridas de toros. A. Lea atentamente la noticia. B. Identifique el argumento de Esperanza Aguirre. C. Determine si incurre en alguna falacia. Justifique. Los toros, a debate

AGUIRRE:

"LOS TOROS SON UN ARTE Y MERECEN ESTAR

BIEN PROTEGIDOS" La presidenta de la comunidad de Madrid justifica as la intencin de su gobierno de declarar los toros Bien de Inters Cultural, que coincide con el debate de prohibicin que se est haciendo en el Parlament de Catalunya El consejero de Cultura de la Comunidad, Ignacio Gonzlez, reconoce que "es evidente que tiene mucha relacin" con el debate que se est produciendo en Catalunya. Madrid (EFE).- La presidenta de la Comunidad de Madrid, Esperanza

Aguirre, ha defendido que "los toros son un arte" y que "merece la pena declararlos Bien de Inters Cultural para que estn bien protegidos". Aguirre ha hecho estas declaraciones en los pasillos de la Asamblea, despus de que su Gobierno haya acordado iniciar el expediente para declarar la Fiesta de los toros Bien de Inters Cultural (BIC). "A nadie le puede extraar, porque la cultura taurina es algo que desde tiempo inmemorial est en la cultura espaola y en la cultura mediterrnea", ha afirmado la presidenta madrilea. A su juicio, "no hay ms que ver que, desde Goya, Picasso o Garca Lorca

o, fuera de nuestras fronteras, Hemingway y Orson Welles, todos se han ocupado de los toros como arte". "Qu duda cabe que es un bien cultural a proteger y, como dice nuestra ley, no solamente los monumentos o museos deben protegerse, tambin los bienes inmateriales", ha argumentado. [...] Aguirre: "Los toros son un arte y merecen estar bien protegidos". (2010, marzo 4). La Vanguardia. Disponible en: <http://www.lavanguardia.es/ciudadan 53896639502/aguirre-los-toros-sonun-arte-y-merecen-estar-bienprotegidos-madrid-esperanzaaguirre-ignacio-gonza.html>

ACTIVIDAD 7. EL CRCULO DE
VIENA Rudolf Carnap fue uno de los miembros destacados del Crculo de Viena (Wiener Kreis) y director de la Revista Erkenntnis donde se publicaron los principales aportes de los integrantes de este crculo de cientficos y epistemlogos destacados. Dada la siguiente lista de nombres de personalidades del siglo XX, investigue acerca de sus trayectorias y luego determine qu rol cumplieron en relacin con el Crculo de Viena. Moritz Schlick, Ludwig Wittgenstein, Otto Neurath, Otto Weininger, Sigmund Freud, Hans Reichenbach, Adolf Hitler, Kurt Gdel, Bertrand Russell, Carl

Hempel, Alfred Einstein, Alfred Tarski, Karl Popper, A. J. Ayer. ACTIVIDAD 8. FALACIAS MATERIALES. EL ESCENARIO INFORMAL Identifique la falacia en la que se incurre en los siguientes argumentos: a. "Te dije el ao pasado que si te casabas con tu novio, al ao estaras separada. Ahora ests separada y es por lo que te dije." b. "Las mujeres no tienen alma. Los chinos, desde la ms remota antigedad, han negado que la mujer tenga alma."[12] c. "Usted no tiene derecho a tomar la palabra porque es funcionario de un gobierno corrupto." d. "No puede hacerme la boleta por

exceso de velocidad porque, si tengo que pagar la boleta, pierdo la ganancia del da y no tengo para darles de comer a mis hijos." e. "Todo abogado tiene derecho a la libertad de consultar libros cuando busca fundamentar mejor una defensa y todo mdico tiene derecho a consultar sus libros cuando quiere curar mejor a un enfermo. Del mismo modo, debe permitirse a los alumnos ejercer la libertad de consultar sus libros para contestar mejor en un parcial." f. "Las esposas de los hombres exitosos usan ropa cara, de modo que la mejor manera que tiene una mujer de ayudar a su marido en los negocios es gastar mucha plata en ropa."

g. "Seor Gerente, mi marido merece que le aumente el sueldo porque lo que gana apenas alcanza para alimentar a los nios." h. "En los ltimos 20 aos ha aumentado la asistencia de adolescentes en la Universidad de Buenos Aires y tambin ha aumentado la delincuencia juvenil. Por lo tanto, para combatir el delito, es necesario limitar el ingreso de jvenes a la Universidad de Buenos Aires." i. "Est comprobado que los seres extraterrestres ayudaron a construir las pirmides de Egipto porque hasta ahora nadie ha podido demostrar que ellos no intervinieron." j. "Si un automvil funciona, entonces tiene nafta en el tanque. Pero ahora el

auto no funciona, eso quiere decir que no tiene nafta en el tanque." k. "Si Marilyn Monroe fue asesinada, entonces est muerta y, efectivamente, est muerta. Eso quiere decir que fue asesinada." l. "Si el amor es ciego y Dios es amor, entonces, Dios es ciego." ACTIVIDAD 9. TOULMIN. MAPAS ARGUMENTATIVOS Stephen Toulmin afirm que las argumentaciones cotidianas no siguen el clsico modelo riguroso del silogismo. Para l, en una argumentacin directa, un "sujeto argumentador" presenta explcitamente una "tesis" u opinin y expone una serie de argumentos o "razones lgicas" que deben desembocar en una "conclusin" que confirma la

tesis propuesta.

Una exclamacin como "Tienes una infeccin y por tanto debes tomar penicilina" es un argumento que parte de una dato ("tienes una infeccin") y postula una conclusin ("debes tomar penicilina"), pero su credibilidad depende de una garanta (la penicilina cura infecciones, por ejemplo) que a su vez se basa en un respaldo (pruebas de laboratorio, experiencia mdica, etc.), condicin que hace posible el uso del

modalizador ("por lo tanto"). Y el argumento puede tener tambin una refutacin ("a menos que seas alrgico" podra ser una). Todos los argumentos parten de un esquema lgico de este tipo, en el que la capacidad persuasiva depende de las premisas en que se funda y de su capacidad de vincularse a las creencias previas del pblico. Por ello, una misma conclusin puede ser defendida o refutada desde diversos puntos de partida. El diagrama de Toulmin puede ayudarnos a esquematizar esta actividad en el momento de organizar la comunicacin. Por ejemplo, yo puedo exhortar a alguien a adquirir un libro con distintos argumentos como:

"Este libro es barato, adquirelo." (Basado en una premisa cuantitativa) "Este libro est muy bien escrito, adquirelo." (Basado en una premisa cualitativa) "Este libro no volvers a verlo, adquirelo." (Basado en una premisa que remite al valor de lo nico) "Si no compras el libro, te reprueban." (Basado en una premisa que remite al criterio del mal menor)
[13]

9.1. Tomando lo estudiado en la unidad, conteste: a. Segn Toulmin, en qu se diferencian los argumentos deductivos de la lgica formal y los argumentos prcticos de la

vida social? b. Por qu la lgica prctica que propone Toulmin incluye el concepto de campos argumentativos? Qu crticas recibe su teora? 9.2. Seale con una (X) cules de las siguientes caractersticas corresponden a los argumentos prcticos de la vida social segn Toulmin: a. Son analticos. b. Son independientes del contexto. c. Dependen del contexto. d. Fundamentan la conclusin de un modo absoluto. e. Fundamentan la conclusin de un modo probabilstico.

Temas de la Unidad 3.1. La matemtica: constructos formales y realidad 3.2. Sistemas axiomticos 3.3. Propiedades de los sistemas axiomticos 3.4. Interpretacin y modelos de los sistemas axiomticos Bibliografa obligatoria Unidad 3 en la Gua de Estudio de IPC, producida por UBA XXI y editada por Eudeba, a partir de la edicin de 2010. Asti Vera, C. y Ambrosini, C. (2009). Captulo 4. En Argumentos y teoras. Aproximacin a la Epistemologa.

Buenos Aires: Educando. BIBLIOGRAFA COMPLEMENTARIA Dez, J. A. y Moulines, C. U. (1999). Captulos 4 y 5. En Fundamentos de filosofa de la ciencia. Barcelona: Ariel. Klimovsky, G. (1994). Captulos 2, 3, 4 y 5. En Las desventuras del conocimiento cientfico. Buenos Aires: AZ. Presentacin de la Unidad

Te apetece unpoco de vino? ", insinu


meliflua la Liebre de Marzo. Alicia mir por toda la mesa sin ver ms que t, por lo qt observ: "No veo ese vino por ninguna parte. " "No lo hay ", replic enseguida la

Liebre de Marzo. "Entonces, no ha sido nada amable elofrecrmelo ", dijo Alicia enojada "Tampoco lo ha sido el sentarse a esta mesa sin haber sido invitada " repuso la Liebre. (Carroll, L. Alicia en el pas de las maravillas)

De acuerdo con el cuadro de clasificacin de las ciencias presentado en la Unidad 1, esta unidad est destinada a completar las caractersticas sealadas all como propias de las ciencias formales. Es decir, estudiaremos el "mtodo axiomtico" y los conceptos que incluye su

formulacin. En este punto conviene detenerse y prestar atencin a la distincin entre la concepcin clsica de las ciencias formales, identificada con Aristteles y la geometra de Euclides en el siglo III a.C. y la ruptura que se produjo en este terreno a partir del cuestionamiento del V Postulado de Euclides, conocido como el Postulado de las paralelas, a mediados del siglo XIX de nuestra era. Para comprender mejor esta ruptura y cambio radical de teoras en el campo de las ciencias formales, conviene que considere los principales aportes a las geometras no euclidianas, sealadas en este captulo. Para ejemplificar los componentes de un sistema axiomtico, donde como en todo

lenguaje, desde el punto de vista sintctico, encontramos trminos (definidos y no definidos), proposiciones (demostrables: teoremas y no demostrables: axiomas) y razonamientos (reglas deductivas), deber detenerse en la presentacin de estos conceptos a partir de la Aritmtica de Peano (matemtico y filsofo italiano, 1858-1932). En la revisin de las propiedades de los sistemas axiomticos, es necesario advertir la importancia del aporte del lgico, matemtico y filsofo estadounidense Kurt Gdel (1906-1978) y su cuestionamiento al requisito de completitud para algunas regiones de las ciencias formales. Finalmente,

revisaremos las nociones de interpretacin y modelo en las ciencias formales. Estas nociones sern retomadas en los captulos siguientes ya que una importante corriente epistemolgica considera a las teoras de las ciencias fcticas como "sistemas axiomticos interpretados" y encuentra elementos comunes en el anlisis lgico de las teoras fcticas y de las teoras formales.

Actividades de aprendizaje
ACTIVIDAD 1. CONSTRUCTOS

FORMALES Y REALIDAD La relacin entre ciencias formales y realidad reintroduce, en parte, las disputas o debates en torno a la relacin entre lenguaje y realidad, ya presentados en la primera unidad puesto que las ciencias formales son de muy productiva aplicacin en la realidad (pensemos que no podramos siquiera pagar el boleto del colectivo si no tuvisemos un dominio de las operaciones matemticas elementales; mucho menos podran construirse puentes o edificios de gran altura ni tampoco planificar actividades econmicas, para citar algunos ejemplos). Pero estas ciencias justifican el valor de verdad de sus enunciados por medio de mtodos totalmente

formales, sin recurrir a la contrastacin emprica ni esperando su validacin o justificacin ms que en razonamientos deductivos. En el fragmento que se presenta a continuacin, se trata de destacar la diferencia entre un conocimiento formal (tautolgico) y un conocimiento emprico (contingente) y se propone volver al cuadro de clasificacin de las ciencias presentado en el libro de la bibliografa (cap. 1) que, como dijimos, sirve como un "mapa" para ir completando a lo largo de todo el curso. A. Lea el siguiente fragmento:
Definir una estructura es lo mismo que formular su teora. Hay que especificar cules son los conceptores, qu combinaciones de conceptores son los axiomas y qu lgica determina la relacin

de consecuencia entre axiomas y teoremas. [...] todo esto es independiente de la realidad emprica del mundo, todo esto es mera matemtica. En efecto, la matemtica suele definirse como la ciencia de las estructuras. En este sentido, todas las teoras son matemticas. [...] Lo que nos interesa es, en primer lugar, el abigarrado yjugoso mundo perceptual que nos rodea, y en segundo lugar, el mundo que simblicamente captamos con nuestro lenguaje y con nuestros conceptos, en resumen, la historia. La teora es un mero instrumento para iluminar la historia. Pero la historia es siempre hipottica e insegura. Slo los fros y vacos teoremas de la teora son seguros, pero no dicen nada acerca del mundo. [...] En definitiva, poseemos un saber perfecto y seguro sobre lo irreal, vaco y formal (las estructuras, objeto de las teoras), pero slo un saber imperfecto e inseguro sobre lo real, lo vivo y lo material (los sistemas objeto de la historia). Somos como las araas, y las teoras son como las redes o telas de araa con que tratamos de captar o capturar el mundo. No hay que confundir estas

redes o telas de araa con el mundo real pero, sin ellas, cunto ms alejados estaramos de poder captarlo y, en ltimo trmino, gozarlo![14]

B. Ahora le proponemos que conteste las siguientes preguntas: a. Teniendo en cuenta las caractersticas de las ciencias formales, cmo interpreta la frase "la matemtica suele definirse como la ciencia de las estructuras"? b. Mostern asocia el conocimiento perfecto y seguro a lo formal y el imperfecto e inseguro a lo real. Explique esta distincin tomando en cuenta el cuadro de clasificacin de las ciencias de la bibliografa (cap. 1). c. Este autor realiza una analoga entre los cientficos y las araas. Explique esta analoga.

ACTIVIDAD 2. SISTEMAS AXIOMTICOS. ACERCA DE LA


INTENCIONALIDAD DE TODO ORDENAMIENTO LGICO Y DE LA NECESIDAD DE TENER UN MTODO. Una de las caractersticas destacadas del conocimiento cientfico es la utilizacin de mtodos o procedimientos que nos permitan llegar a la afirmacin de teoras. En esta unidad se presentan las caractersticas del "mtodo axiomtico" en tanto es el adecuado para la construccin de teoras en ciencias formales. Recordemos que la palabra mtodo, derivada del griego, etimolgicamente significa camino. En el pasaje que se cita a continuacin,

Lewis Carroll nos indica que es necesario definir adnde queremos llegar antes de saber qu mtodo o camino seguir ya que, por definicin, todo camino nos conduce a algn lugar. A. Lea el siguiente dilogo entre Alicia y el gato de Cheshire. B. Luego, explique por qu Alicia afirma que la proposicin: "Si slo caminas bastante, puedes estar segura de llegar a algn lado", es irrefutable.

"Gatito de Cheshire" empez con un


poco de temor, porque no Y V estaba muy segura de que a lle gustara el nombre. Sin embargo, el Gato sonri algo ms ampliamente. "Vamos, hasta ahora le gusta", pens Alicia y

continu: "Querra decirme, por favor, qu camino debo tomar para irme de aqu?" "Eso depende mucho del lugar adndequieras llegar"dijo el Gato. "Me da lo mismo ellugar[...]"dijo Alicia. "Entonces no importa qu camino tomes"dijo el Gato. "[...]siempre y cuando llegue a algn lado agreg Alicia a modo de explicacin."

"Oh, puedes estar segura de llegar a algn lado "dijo el Gato, "si slo caminas bastante." Alicia comprendi que esto era irrefutable, de modo que prob con otra pregunta [...] (Carroll, L. 1990)

ACTIVIDAD 3. ACERCA DE LAS


PROPIEDADES DE UN SISTEMA AXIOMTICO Dado el siguiente sistema axiomtico formal (SAF): Trminos primitivos: M, I, U. Axiomas: MI Reglas:

RI: Si se tiene una cadena cuya ltima letra sea I, se le puede agregar una U al final. RII: Supongamos que se tenga Mx. En tal caso, puede agregarse Mxx a la coleccin. ("x" es cualquier cadena, por ejemplo: I, o IU, etc. As podemos pasar de MI a MII siendo I "x" e II "xx", o pasar de MIU a MIUIU siendo IU "x" e IUIU "xx"). Las cadenas nunca pueden contener "x". RIII: Si en una de las cadenas de la coleccin aparece la secuencia I I I, puede elaborarse una nueva cadena sustituyendo I I I por U. RIV: Si aparece UU en el interior de una de las cadenas, est permitida su eliminacin.[15]

Puede usted producir la palabra MU como teorema? El desafo consiste en probar o bien que la palabra MU es un teorema en el sistema MIU, o bien que no lo es. Si MU es un teorema, entonces puede ser generado a partir de su nico axioma y las cuatro reglas de transformacin de frmulas, y en tal caso habr que mostrar su derivacin. Si MU no es un teorema de MIU, entonces habr que probar que bajo ninguna circunstancia puede ser generado por el axioma y las cuatro reglas de MIU. Esta prueba no es una derivacin en MIU, sino que tendr la forma de una demostracin en el metalenguaje. se es el desafo. MATERIALES DE LECTURA I

ACERCA DE LAS CARACTERSTICAS


DEL CONOCIMIENTO DE LAS CIENCIAS FORMALES Y SU DIFERENCIA CON EL CONOCIMIENTO DE LA REALIDAD TAL COMO LO PRESENTA BERTRAND RUSSELL En el siguiente texto, de modo algo irnico, Bertrand Russell destaca la diferencia entre un mundo "ucrnico" (no afectado por el paso del tiempo) y el mundo real donde todo es perecedero y el tiempo es una magnitud primordial. Con esta nota seala la diferencia entre el conocimiento formal y el conocimiento fctico. Aqu Russell alude al mundo de las ciencias formales como un "mundo ucrnico", intemporal,

es decir, donde el paso del tiempo no cambia nada y donde no hay progreso, en el sentido en que podramos decir que no hay progreso entre adoptar como un juego al ftbol o al tenis, sencillamente son dos juegos distintos. Como veremos en las siguientes unidades, se debate acerca del progreso de las ciencias en el campo de las ciencias fcticas. De todos modos, la aplicacin de la nocin de progreso a las ciencias formales es motivo de debates epistemolgicos, por ejemplo, si representa un progreso cientfico la aparicin de las geometras no euclidianas. Bertrand Arthur William Russell,

tercer conde Russell, naci el 18 de mayo de 1872 y muri, a los 97 aos, el 2 de febrero de 1970. Durante casi un siglo vivi una vida asombrosamente variada. Bertrand Russell fue el segundo hijo del Vizconde de Amberley, un ttulo creado en 1861 para su abuelo, Lord John Russell, el primer ministro liberal que luch por la Ley de Reforma de 1831 y fue partidario decidido de las doctrinas de John Stuart Mill. Su madre muri cuando tena tres aos y fue educado por su padre en el agnosticismo, pero al ao siguiente tambin l muri. Su abuela le imparti una educacin estrictamente victoriana en franca contravencin a la voluntad de sus padres muertos. Su abuelo, John

Russell, haba sido primer ministro de la Reina Victoria de 1846 a 1852 y de nuevo de 1865 a 1866. De adulto, Russell recordaba cuando se sentaba en las rodillas regias de Victoria durante las visitas de sta a la mansin de su abuelo. Segn su propio relato, fue un joven solitario y reprimido. Aludiendo a s mismo confes: "Yo no nac feliz[...] Durante mi adolescencia aborrec la vida y estuve continuamente al borde del suicidio, del cual me apartaba debido a mi deseo de aprender ms matemticas". Llegado el momento, march al Trinity College, en Cambridge, la misma institucin que acogi al joven Isaac Newton, ms de dos siglos antes y que luego recibira a

otros genios de la talla de John Maynard Keynes y Ludwig Wittgenstein, con quienes entabl una relacin de profunda colaboracin intelectual. Inmediatamente luego de entrar a la vida universitaria, Russell se entreg a las matemticas, algo impersonal que, en sus propias palabras, poda amar sin ser amado en reciprocidad. Para Russell, las matemticas ofrecan una nica va para la certeza y perfeccin. "Me desagradaba el mundo real, y busqu refugio en el mundo ucrnico, sin cambio ni corrupcin ni el fuego fatuo del progreso."

MATERIALES DE LECTURA II ACERCA DE BORGES Y LOS

INFINITOS El primer prrafo del cuento de Borges "El libro de arena",[16] dice: La lnea consta de un nmero infinito de puntos; el plano, de un nmero infinito de lneas; el volumen, de un nmero infinito de planos; el hipervolumen, de un nmero infinito de volmenes... No, decididamente no es ste, more geomtrico, el mejor modo de iniciar mi relato. Afirmar que es verdico es ahora una convencin de todo relato fantstico; el mo, sin embargo, es verdico. En este cuento Borges problematiza la idea de infinito. Segn el relato, el

protagonista compra a un vendedor que toca a su puerta un curioso libro, muy pesado. La particularidad de este libro es que una vez abierta una hoja ya nunca se la volver a encontrar. En este curioso libro entre la tapa y la primera pgina hay infinitas pginas de modo que se presenta la situacin anmala de que nunca podramos acceder a la primera pgina porque siempre aparecer otra. "Ninguna es la primera, ninguna es la ltima", afirma el vendedor. El protagonista lo compra pero luego de un tiempo se deshace del libro que considera tan monstruoso como l mismo. As nos lo cuenta Borges:

Declinaba el verano, y comprend que el libro era monstruoso. De nada me sirvi considerar que no menos monstruoso era yo, que lo perciba con ojos y lo palpaba con diez dedos con uas. Sent que era un objeto de pesadilla, una cosa obscena que infamaba y corrompa la realidad. Pens en el fuego, pero tem que la combustin de un libro infinito fuera parejamente infinita y sofocara de humo al planeta. Record haber ledo que el mejor lugar para ocultar una hoja es un bosque. Antes de jubilarme trabajaba en la Biblioteca Nacional, que guarda novecientos mil libros; s que a mano

derecha del vestbulo una escalera curva se hunde en el stano, donde estn los peridicos y los mapas. Aprovech un descuido de los empleados para perder el Libro de Arena en uno de los hmedos anaqueles. Trat de no fijarme a qu altura ni a qu distancia de la puerta. Siento un poco de alivio, pero no quiero ni pasar por la calle Mxico. En el siguiente texto, el escritor y matemtico Guillermo Martnez profundiza sobre este tema en su libro Borges y la matemtica:[17] Dice Guillermo Martnez (Argentina, 1962): "Hay elementos de matemtica muy variados a lo largo de la obra de

Borges. Los cuentos 'El disco', 'El libro de arena', 'La biblioteca de Babel', 'La lotera de Babilonia', 'Del rigor en la ciencia', 'Examen de la obra de Herbert Quain', Argumentum ornithologicum'; los ensayos La perpetua carrera de Aquiles y la tortuga' junto con 'Avatares de la tortuga', 'El idioma analtico de John Wilkins', 'La doctrina de los ciclos', 'Pascal' junto con 'La esfera de Pascal', etc. Hay textos que son incluso pequeas lecciones de matemtica. Uno de los cuentos sealados es 'El libro de arena'. Qu ocurrir cuando consideramos los nmeros fraccionarios? Los nmeros fraccionarios son muy importantes en el pensamiento de Borges. Por qu? Recordemos que los nmeros

fraccionarios, que tambin se llaman quebrados, o nmeros racionales, son los que se obtienen al dividir nmeros enteros, los podemos pensar como pares de enteros: un nmero entero en el numerador y un nmero entero (distinto de cero) en el denominador. 3/5, 5/4, 7/6, 7/16... Cul es la propiedad que tienen estos nmeros, la propiedad que usa Borges en sus relatos? Entre dos nmeros fraccionarios cualesquiera siempre hay uno en el medio. Entre 0 y 1 est 1/2, entre 0 y 1/2 est 1/4, entre 0 y 1/4 est 1/8, etc. Digamos, siempre se puede dividir por 2.

De modo que cuando yo quiero saltar del 0 al primer nmero fraccionario, nunca puedo encontrar ese primer nmero en el orden usual, porque siempre hay uno en el medio. sta es exactamente la propiedad que toma prestada Borges en 'El libro de Arena'. Recordarn que hay un momento en este cuento en que al personaje de Borges lo desafan a abrir por la primera hoja el Libro de Arena. 'Me dijo que su libro se

llamaba el Libro de Arena porque ni el libro ni la arena tienen principio ni fin. Me pidi que buscara la primera hoja. Apoy la mano izquierda sobre la portada y abr con el dedo pulgar casi pegado al ndice. Todo fue intil: siempre se interponan varias hojas entre la portada y la mano. Era como si brotaran del libro'."

MATERIALES DE LECTURA III ACERCA DE KURT GDEL Y JOHN NASH. MENTES QUE
BRILLAN Es frecuente y parte del imaginario

popular la asociacin entre genios matemticos y esquizofrenia. La pelcula norteamericana A Beautiful Mind, traducida como una mente brillante, alude a los trastornos psiquitricos del matemtico norteamericano John Nash, Premio Nobel de Economa de 1994, por sus aportes a la teora de juegos y los procesos de negociacin, junto con Reinhard Selten y John Harsanyi. Existe una vasta literatura tanto cientfica como no cientfica que asimila al genio, es decir, a una persona con talentos intelectuales ms desarrollados o un coeficiente intelectual superior, con un carcter extravagante, insociable o patolgico.

En esta actividad presentamos algunas de las circunstancias de la vida de Gdel, no tanto para detenernos en detalles biogrficos, sino para mostrar el ambiente intelectual de la produccin de teoras revolucionarias en ciencias formales en las primeras cinco dcadas del siglo XX, visto desde los avatares personales de algunos de sus principales protagonistas que padecieron las consecuencias de dos guerras mundiales, la persecucin racial o poltica, el exilio y, en muchos casos, la incomprensin de sus contemporneos. Kurt Gdel naci el 28 de abril de 1906 en Brnn, Moravia, en la poca

parte del Imperio Austro-hngaro. Ingres en la Universidad de Viena en 1924 planeando estudiar Fsica Terica. Hacia 1926 su atencin volvi a las matemticas y se produjo su unin a lo que ms tarde fue conocido como el Crculo de Viena. Gdel estuvo asociado con este grupo durante muchos aos. Gdel se fue interesando progresivamente en Teora de Nmeros y, despus, en Lgica Matemtica durante estos aos. En 1930, Gdel se doctor en matemticas dirigido por H. Hahn, un notable matemtico miembro del Crculo de Viena. A partir de aqu Gdel comienza a trabajar en sus ms

importantes teoras sobre la completitud de sistemas formales. Viaj a los Estados Unidos dando un ciclo de conferencias y se encontr por primera vez con Albert Einstein en 1933. Gdel pas el ao acadmico 1933-34 en Princeton, en el recin fundado Instituto de Estudios Avanzados, donde disert sobre sus resultados de incompletitud. Fue invitado a volver al ao siguiente, pero al poco de regresar a Viena sufri una grave crisis mental. Se recuper a tiempo para retornar a Princeton en el otoo de 1935; al mes de su llegada sufri una recada, y no volvi a impartir enseanza hasta la primavera de

1937, en Viena. Dedic alguno de los aos siguientes al estudio de problemas de Fsica y de Psicologa. Durante esta poca tuvo que ser ingresado varias veces en hospitales por problemas de salud. Por ser confidencial el historial mdico de Gdel, la diagnosis de su mal sigue siendo desconocida. Sus problemas parecen haber comenzado con hipocondra: estaba obsesionado por su dieta y por sus hbitos intestinales. Durante veinte aos llev un registro diario de su temperatura corporal y de su consumo de leche de magnesia. Tema sufrir un envenenamiento accidental; con los aos, le aterraba ser objeto de una

intoxicacin deliberada. Esta fobia le llev a no querer tomar alimentos, con la consiguiente desnutricin. Lo que no le impeda ingerir pldoras de diversa condicin para un imaginario problema cardaco. Gdel se cas con Adele Porkert en 1938 y decidieron trasladarse definitivamente a los Estados Unidos en 1940. Ms de una vez Adele, sirvindole de catadora de alimentos, contribuy a paliar los temores de Gdel, cada vez ms fuertes, de que buscaban envenenarlo. Se asentaron en Princeton, New Jersey, donde residieron hasta el final de sus vidas. Lleg a ser un gran amigo de Einstein, y trabajaron juntos los aspectos

filosficos y matemticos de la Teora General de la Relatividad. Gdel incluso trabaj con xito en las ecuaciones del campo gravitatorio, encontrando soluciones sorprendentes. Tambin dedic gran parte de esa poca al estudio del concepto de tiempo, publicando varios artculos y dando varias conferencias sobre el tema. El ltimo de sus artculos publicados en vida apareci en 1958. Despus, se sumi en la introversin, cada vez ms demacrado, paranoide e hipocondraco. Su ltima aparicin pblica aconteci en 1972, al recibir un doctorado honorfico por la Universidad Rockefeller. Tres aos

despus le fue otorgada la Medalla Nacional de Ciencias, pero Gdel disculp su asistencia por razones de salud. El 1 de julio de 1976, alcanzados los 70 aos, edad de jubilacin obligatoria, Gdel se convirti en profesor emrito. Sus responsabilidades empero no disminuyeron, porque su esposa, que durante tantos aos lo haba alimentado y protegido, haba sufrido pocos meses antes un ataque cardaco que la dej invlida. Ahora le corresponda a l cuidarla. Y as lo hizo, con devocin, hasta julio de 1977, cuando ella hubo de someterse a una operacin de urgencia y permaneci hospitalizada durante

casi seis meses. Por aquellas fechas, Morgenstern, el amigo que haba contribuido a cuidar de Gdel tras fallecer Einstein en 1955, muri de cncer. Gdel tuvo entonces que luchar por s solo contra su cada vez ms acusada paranoia. Solo frente a ella, su declive se aceler. Temeroso de ser envenenado, dej de comer y acab muriendo por desnutricin el 14 de enero de 1978.

MATERIALES DE LECTURA IV GDEL PARA TODOS


En la Feria del Libro de la ciudad de Buenos Aires realizada en abril de 2009 se present el libro de Guillermo Martnez y Gustavo Pieiro, Gdel para

todos.[18] En una nota de la poca, se hace un comentario del libro en el que encontramos una breve resea de los principales hitos en la historia de las ciencias formales. Gdel para todos En la evolucin de cada disciplina existen hitos, momentos de ruptura. En el caso de la matemtica, el Teorema de Incompletitud de Gdel marca un antes y un despus. El mtodo axiomtico-deductivo, fuente de toda verdad desde que fuera introducido por Platn y Aristteles y plasmado por Euclides, mostrar importantes limitaciones: existen

verdades no demostrables. En Gdel para todos, Guillermo Martnez y Gustavo Pieiro se plantean el inmenso desafo de desarrollar la demostracin completa del Teorema de Gdel con rigor absoluto y accesible a todos. Los egipcios y los babilonios comienzan a construir la matemtica en torno de problemas ligados a la medicin de la Tierra. Los griegos toman el relevo y constatan que, para mejorar su eficacia, la matemtica debe revisar sus fundamentos. As, en los Segundos Analticos, Aristteles seala: "La ciencia demostrativa debe partir de premisas verdaderas, primeras, inmediatas, ms

familiares, anteriores, causas de la conclusin". En este contexto surgen los elementos de Euclides (325-265 a.C.), que organizan por primera vez los principales resultados de la Geometra a partir de diez verdades primeras. Euclides distingue entre postulados y nociones comunes o axiomas. Las nociones comunes son verdaderas en todos los campos del pensamiento e incluyen afirmaciones tales como: "Las cosas iguales a una misma cosa son iguales entre s", y los postulados se aplican a temas especficos de la geometra como: "Dos puntos determinan una recta". Desde Euclides, para validar sus resultados, la matemtica recurre a un

mtodo especial, la demostracin deductiva a partir de estas verdades primeras. En la formulacin de los postulados de Euclides hubo cierta incomodidad. Las sospechas caan sobre el Postulado de las paralelas (quinto postulado): "Dados una recta y un punto exterior a ella puede trazarse una nica recta paralela a la recta dada que pase por ese punto". Era necesario considerarlo un postulado o se podra deducir de los otros, en cuyo caso era en realidad un teorema? En 1799, Karl Friederich Gauss escribe: "He hecho algunos progresos en mi trabajo que no conducen en absoluto a la meta que buscamos (la deduccin del quinto

postulado); ms bien parecen obligarme a dudar de la geometra misma". A partir de 1831, Gauss desarrolla una geometra basada en la afirmacin de todos los postulados y la negacin del quinto, y concluye que su trabajo no conlleva contradiccin. Muy asombrado afirma: " Los teoremas de esta geometra parecen paradjicos y, para los no iniciados, absurdos, pero una reflexin tranquila y sosegada revela que no contiene en absoluto nada imposible". Los trabajos de Nicoli Lobachevski, Jnos Bolyai y Georg Riemann profundizan las ideas de Gauss, mostrando que es posible negar el postulado de las paralelas

sin generar contradicciones y desarrollar las consecuencias del nuevo conjunto de axiomas. Dan fundamento al descubrimiento de nuevas geometras -no euclidianas- e instalan la idea de que los axiomas y postulados de Euclides son verdades empricas ms que verdades evidentes por s mismas. En particular, deja de tener sentido distinguir entre postulados (verdades de la matemtica) y nociones comunes (verdades fuera de la matemtica). Las verdades primeras pasan a considerarse todas axiomas, independientes de la intuicin y la experiencia. En la segunda mitad del siglo XIX, los resultados sobre las

geometras no euclidianas hacen que los matemticos pongan en duda los fundamentos de otras zonas de la disciplina. Esto lleva a un trabajo de revisin y a una preocupacin por elaborar una fundamentacin slida eliminando los conceptos vagos y las contradicciones, que resultan en axiomas. Como ejemplo de conceptos que se busca precisar, podemos citar la ambigedad de Euclides al hablar de puntos que estn entre otros puntos sin aclarar la nocin de "estar entre", y al definir una recta como "aquella que yace por igual respecto de los puntos que estn en ella", sin aclarar qu entiende por "yacer por igual". David Hilbert, principal impulsor de

la matemtica de la segunda mitad del siglo XIX y primera del XX, propone no definir los conceptos bsicos y explcita: "Las nociones de puntos, rectas y planos pueden reemplazarse por mesas, sillas y jarros de cerveza". Siguiendo a Hilbert, basta que los axiomas hagan afirmaciones acerca de conceptos no definidos. Si el punto o la recta son conceptos no definidos, el axioma que dice que dos puntos determinan una sola recta proporciona una afirmacin que puede ser utilizada para obtener posteriores resultados acerca de puntos y rectas. En trminos coloquiales, no importa quin eres, basta saber con quin andas. La

intencin de Hilbert era refundar la matemtica sobre bases slidas a partir de sistemas axiomticos (o teoras), entendidos como conjuntos de axiomas con reglas lgicas que permiten desarrollar demostraciones. El desafo era encontrar un sistema axiomtico consistente (libre de contradicciones) y completo, es decir, que permita obtener a travs de demostraciones todos los resultados verdaderos de la matemtica. En 1929, el jovencsimo Kurt Gdel (1906-1978) presenta su tesis doctoral, en la que demuestra su Teorema de Completitud, que refiere a la lgica de predicados, esto es, a las afirmaciones, vlidas en todo

contexto, que sustentan el razonamiento matemtico. Es posible dar axiomas que permiten demostrar todas las afirmaciones de esta clase. En 1930, casi todos los matemticos estaban convencidos de que en todos los sistemas axiomticos sera posible encontrar teoremas de completitud similares; elegidos adecuadamente los axiomas, toda afirmacin verdadera en la teora sera deducible. Sin embargo, Gdel demuestra que no es as en su famoso primer Teorema de Incompletitud (1931), en el que prueba que la aritmtica elemental es incompleta: no es posible dar axiomas que permitan demostrar todas las

verdades de esta teora. [...] Se lee en la Introduccin: "A diferencia de la teora de la relatividad, en que por la sofisticacin de las ecuaciones, los mejores intentos de divulgacin parecen condenados a ejemplos con relojes y personas que no envejecen en viajes por el espacio, en el caso del Teorema de incompletitud hay una buena noticia, y es que puede darse una exposicin a la vez rigurosa y accesible que no requiere ninguna formacin matemtica, ms que el recuerdo de la suma y la multiplicacin tal como se ensean en la escuela primaria". Extrado de: Gdel para todos. (2009, junio 6). Revista . [en lnea].

[consulta: 26 de julio de 2010]. <http://www.revistaenie.clarin.com/no 01932842.htm> Actividad de integracin de los temas de la Unidad Determine si las siguientes afirmaciones son verdaderas o falsas (V F). Justifique en cada caso. a. En una regla de inferencia deductiva nunca se da el caso de que de una falsedad se obtenga una verdad. b. En una regla de inferencia deductiva nunca se da el caso de que de una verdad se obtenga una falsedad. c. En un sistema axiomtico, todos los enunciados se demuestran. d. En un sistema axiomtico, todos los

trminos se definen. e. Las geometras no euclidianas surgieron al cuestionarse la independencia del V postulado de Euclides. f. Los teoremas son relativos a su sistema de pertenencia; esto es, en otro sistema pueden funcionar como axiomas. g. Las geometras no euclidianas representan un avance cientfico que resta inters a la geometra de Euclides. h. La independencia de los axiomas permite que unos se puedan deducir de otros. i. Si todos los teoremas se demostraran, los sistemas axiomticos seran viciosos o infinitos.

Temas de la Unidad 4.1. Estructura de las teoras empricas 4.2. El problema del mtodo cientfico 4.3. Estrategias metodolgicas bsicas de las ciencias fcticas 4.4. Mtodo inductivo: inductivismo "estrecho" e inductivismo "sofisticado" 4.5. Mtodo hipottico-deductivo Bibliografa obligatoria Unidad 4 en la Gua de Estudio de ipc, producida por UBA XXI y editada por Eudeba, a partir de la edicin de 2010. Asti Vera, C. y Ambrosini, C. (2009). Captulo 5. En Argumentos y teoras.

Aproximacin a la Epistemologa. Buenos Aires: Educando. BIBLIOGRAFA COMPLEMENTARIA Cohen, I. y Nagel, E. (1980). Captulos X, XI, XIV y XVI. En Introduccin a la lgica y al mtodo cientfico (vol. II). Buenos Aires: Amorrortu. Hempel, C. G. (1985). Captulos 2, 3 y 4. En Filosofa de la ciencia natural. Madrid: Alianza. Klimovsky, G. (1994). Captulos 9, 13 y 14. En Las desventuras del conocimiento cientfico. Buenos Aires: AZ. Presentacin de la Unidad Quin soy yo? Ah!Eso s que es un misterio!" Y con esto se puso a en

pensar en todas las nias de su edad que conoca, para ver si se haba transformado en una de ellas. [...] "Voy a ver si al menos s las cosas que antes saba. Veamos: cuatro por cinco son doce, cuatro por seis son trece, y cuatro por siete... Ay, Dios mo! As no llegar nunca a veinte! Bueno, de todas formas la tabla de multiplicar no me vale; probemos con la Geografa. Londres es la capital de Pars, y Pars es la capital de Roma, y Roma es[...]" (Carroll, L. Alicia en el pas de las maravillas)

La distincin entre ciencias formales y fcticas reconoce diferencias entre ambos tipos de saberes, lo que no significa necesariamente que no existen condiciones constructivas comunes. Segn algunos autores, una teora

emprica puede considerarse como un conjunto de hiptesis de partida y sus consecuencias lgicas (Popper, Bunge, Klimovsky). De all que esta unidad comienza con la exposicin del lenguaje y la estructura de una teora fctica, realizando luego una presentacin de la cuestin del mtodo en las ciencias fcticas, tema que es motivo de importantes debates en el mbito de la epistemologa contempornea. En su desarrollo examinaremos las estrategias bsicas de las ciencias empricas, tanto como las discusiones ms importantes en el contexto de las metodologas rectoras en el siglo XX. Entre otras cuestiones anticipan las controversias, an no canceladas, en torno al monismo

metodolgico y al pluralismo metodolgico que estudiaremos de manera ms especfica en las unidades siguientes y que afectan especialmente a las ciencias sociales, a su constitucin, a la legitimacin de sus mtodos y a su proyeccin histrica. En esta unidad veremos distintas versiones acerca del modo de interpretar la relacin teoraobservacin. Estas diferencias se presentan a partir de la distincin propuesta por el filsofo alemn Hans Reichenbach (1891-1953) entre contexto de descubrimiento y contexto de justificacin, a los que el matemtico y filsofo argentino Gregorio Klimovsky (1922-2009) agrega el contexto de aplicacin de las ciencias antes de

presentar la posicin del filsofo, socilogo y terico de la ciencia austraco Karl Popper (1902-1942), quien confina la metodologa al contexto de justificacin. Esta diferenciacin entre contextos y la reclusin de la Epistemologa a alguno de estos contextos con exclusin de otros es motivo de fuertes polmicas, en especial entre las posiciones de Popper y del epistemlogo estadounidense Thomas Kuhn (1922-1996), y creemos que es importante advertir la presencia de dichas distinciones en esta unidad ya que luego esas diferencias sern recuperadas en las unidades siguientes. A continuacin conviene que se detenga a estudiar con especial inters, en el

captulo 5 de la bibliografa, las distintas estrategias metodolgicas bsicas en las ciencias fcticas donde encontrar sealada la especificidad de las ciencias sociales en la postulacin de mtodos distintos a los de las ciencias naturales. Este punto es conveniente que lo relacione con el Documento de Ctedra de Gastn Beraldi de la Unidad 5. En la Unidad 2, desde distintos enfoques, hemos visto la induccin como forma de razonamiento y los esfuerzos de Carnap por legitimar este tipo de inferencia como un procedimiento formal. En esta unidad, enfocaremos el problema del inductivismo considerado como una

posicin metodolgica, es decir, la posicin epistemolgica que considera a la induccin como mtodo de investigacin cientfica. El llamado por sus crticos "inductivismo ingenuo" defiende la induccin por enumeracin simple, cree que la ciencia empieza con la observacin y sostiene que la observacin ofrece una base segura a partir de la cual se puede derivar el conocimiento. En su versin refinada, el inductivismo tiene mtodos ms sofisticados que la enumeracin simple, pero sigue reconociendo a la observacin como primera fuente de conocimiento. En referencia al llamado "inductivismo ingenuo", debe detenerse en la lectura y comprensin de los

llamados "mtodos de Mill", que toman el nombre de su creador, el filsofo ingls John Stuart Mill (1806-1873). Se conocen estos cnones o reglas de investigacin como "mtodo de la concordancia", "mtodo de la diferencia", "mtodo conjunto de la concordancia y la diferencia", "mtodo de los residuos" y "mtodo de la variacin concomitante". Estos mtodos buscan descubrir conexiones causales entre fenmenos. La insistencia de Mill en la utilidad de estos mtodos para descubrir conexiones causales lo llev a largas polmicas con sus contemporneos y a reafirmar sus convicciones en defensa de la lgica inductiva. Dada la importancia central

de esta posicin en la historia de la epistemologa de los siglos XIX y XX, en esta unidad encontrar expuestas distintas versiones del inductivismo, comenzando con la presentacin de los mtodos de Mill y las crticas del filsofo alemn Carl Hempel (19051997) cuando lo caratula como inductivismo ingenuo antes de sealar las versiones ms elaboradas de Carnap y de Reichenbach. El estudio de este tramo del captulo 5 de la bibliografa es necesario para luego contextualizar las distintas posiciones epistemolgicas presentadas en la Unidad 6. Ya avanzando en la historia del siglo XX, se presenta en este captulo al "mtodo hipottico-deductivo" en las

versiones confirmacionistas de Carnap y Hempel, representantes del llamado "inductivismo sofisticado". Se presenta luego la versin refutacionista de Popper y el llamado "refutacionismo sofisticado" del matemtico y filsofo de la ciencia hngaro Imre Lakatos (1922-1974). En la presentacin de la posicin refutacionista, adjudicada a Popper, conviene que advierta la importancia del Modus Tollens como un argumento deductivo que permite la refutacin de una hiptesis y ligada a la importancia de este tema, acceder a la comprensin del concepto de "falsabilidad" como criterio de demarcacin para considerar cientfica a una hiptesis. En el desarrollo de esta

parte de la unidad, conviene que lea atentamente los pasos del mtodo hipottico-deductivo y los distintos tipos de hiptesis involucradas en la puesta a prueba de las teoras cientficas (hiptesis fundamental, hiptesis auxiliar e hiptesis ad hoc). Estas distinciones son importantes ya que sern consideradas como temas centrales en la presentacin de la posicin de Lakatos en la Unidad 6. Dentro de la metodologa de las ciencias fcticas es importante la utilizacin de mtodos estadsticos en ciencias naturales y en ciencias sociales, de all que ste sea el tema que cierra esta unidad. Si bien la utilizacin de los mtodos estadsticos ha tenido un gran

desarrollo en el campo de las ciencias, su justificacin no deja de ser problemtica y de dar lugar a la incursin en diversos tipos de argumentos falaces. Al comienzo de este tema se realiza una breve resea histrica de los estudios sobre clculos de probabilidad aludiendo a los matemticos franceses Blas Pascal y Pierre de Fermat (1608-1665). Se presenta el concepto de "promedio estadstico" visto como elemento importante para luego revisar las limitaciones de estos mtodos segn han sido sealadas por los filsofos estadounidenses Morris R. Cohen (1880-1947) y Ernest Nagel (19011985). En este punto es importante

reconocer la conexin de este tema con el uso de procedimientos inductivos y los problemas que se presentan cuando se trata de justificar este tipo de inferencias.

Actividades de aprendizaje
ACTIVIDAD 1. INDUCTIVISMO
El inductivismo es la posicin epistemolgica que postula la induccin como nico mtodo de investigacin cientfica. Ya se ha visto que la induccin es una forma de razonamiento que no tiene validez lgica; sin embargo,

para conocer el mundo emprico y ampliar nuestro conocimiento acerca de l, con todas sus falencias, la induccin sigue siendo imprescindible. El inductivismo ingenuo defiende la induccin por enumeracin simple, cree que la ciencia empieza con la observacin y sostiene que la observacin ofrece una base segura a partir de la cual se puede derivar el conocimiento. En su versin refinada el inductivismo tiene mtodos ms sofisticados que la enumeracin simple pero sigue reconociendo a la observacin como primera fuente de conocimiento. Para los inductivistas ingenuos la investigacin tiene los siguientes pasos:

1) Observacin neutral de la realidad. 2) Captacin de una propiedad en diversos individuos. 3) Generalizacin de dicha propiedad a todos los individuos. 4) Formulacin de una ley o generalizacin emprica. El supuesto que permite ir del paso 2 al paso 3 es el llamado Principio de Regularidad de los Fenmenos, principio por el cual se cree que la naturaleza es constante, de modo que se hace valer para todos los casos lo observado slo en algunos. Lea el siguiente texto e identifique los pasos de la investigacin inductiva apuntados ms arriba:

Beber alarga la vida Un estudio sobre los hbitos de ingestin de alcohol de los norteamericanos ha arrojado un resultado sorprendente. Los hombres que toman entre 0,8 litros y 2,4 litros de cerveza a la semana tienen ms oportunidades de vivir durante ms tiempo. Segn el profesor David Williams, de la Universidad de Gales, en Cardiff (Reino Unido) existen pruebas fehacientes de que las personas que beben cerveza moderadamente tienen una expectativa de vida cinco aos mayor que los abstemios.

ACTIVIDAD 2. INDICTIVISMO. LOS MTODOS DE JOHN STUART MILL


En el texto de la bibliografa, se mencionan los que se han considerado, en su formulacin clsica, "mtodos de Mill" de la inferencia inductiva. Mill ha insistido en la utilidad de estos mtodos para descubrir conexiones causales y reafirmado sus convicciones en defensa de la lgica inductiva: "La misin de la lgica inductiva es proveer de reglas y modelos (como el silogismo y sus reglas son modelos para el raciocinio) que, si los razonamientos inductivos se adecuan a ellos, son concluyentes, pero en caso contrario no lo son." Segn Mill, sus mtodos permitiran

descubrir y probar conexiones causales. Es indudable que esta pretensin no se ha realizado, afirma Copi, puesto que la utilizacin de mtodos mecnicos no alcanz, en muchos casos, para descubrir la causa de algunos fenmenos ya que no hay ningn recurso simple o mtodo mecnico que por s mismo haya permitido conquistar el conocimiento cientfico. "Para hacer un anlisis correcto en la aplicacin de estos mtodos se necesita el conocimiento previo de teoras", concluye Copi. (1984. 452-453) 2.1. El bebedor cientfico Irving Copi en su libro introduccin a la lgica (1984) cita el caso de un bebedor que todas las noches se

emborracha. Estaba arruinando su carrera y su salud. Uno de los pocos amigos que le quedaba le aconsej abandonar la bebida. Resolvi llevar a cabo un cuidadoso experimento para descubrir la causa exacta de las borracheras al tomar distintas mezclas de bebidas. Durante cinco noches seguidas tom gin con soda, whisky con soda, aguardiente con soda, cognac con soda y ron con soda. Usando el mtodo de la concordancia de Mill lleg a la conclusin: "Nunca ms tomo soda". Responda: El mtodo de la concordancia de Mill avalara esta conclusin? Dnde radica la falacia? 2.2. Pasteur y el experimento con la vacuna contra el carbunco

En la primavera de 1881, Pasteur puso a prueba su hiptesis de que la vacuna de carbunco produce inmunidad a la enfermedad, hiptesis ridiculizada por los veterinarios de la poca. En una granja de Poully-leFort se administr la vacuna a 24 ovejas, una cabra y varias vacas. Se tom como muestra de control otras veinticuatro ovejas, una cabra y varias vacas que quedaron sin vacunar. Dos meses despus las 48 ovejas, las dos cabras y todas las vacas fueron inoculadas con una dosis letal de virulentos bacilos de carbunco. Tres das despus los animales del primer grupo brincaban y coman normalmente como si nunca

hubieran estado en contacto con el bacilo, mientras que los animales no vacunados estaban muertos o en una irreversible agona. As mostrado, este experimento parece responder al mtodo conjunto de la concordancia y la diferencia. Identifique los pasos de la investigacin en el texto. Este experimento prueba que es verdadero el enunciado "la vacuna inmuniza contra la enfermedad"? Por qu? 2.3. El piojo y el tifus Se supuso durante mucho tiempo que el virus del tifus es inoculado por el piojo al chupar la sangre, pero no es

as. La infeccin no est en la saliva, como en el caso del mosquito, sino en las heces. Al ser picado, el animal se rasca y la infeccin entra en contacto con la sangre de tal modo que "picadura" e "infeccin" son fenmenos inseparables. En 1922 dos bilogos pusieron piojos infectados sobre un mono, tomando la precaucin de que las heces no entraran en contacto con la piel del mono. A pesar de las picaduras, el mono no se infect. Responda: Cul de los mtodos de Mill puede advertirse en este experimento? Los resultados del experimento son concluyentes para afirmar que "las heces

del piojo contagian la enfermedad"? ACTIVIDAD 3. ACERCA DEL MTODO HIPOTTICO-DEDUCTIVO El amor es una droga dura es el ttulo de una novela de la escritora uruguaya Cristina Peri Rossi,[19] residente en Barcelona. La historia trata sobre un fotgrafo de 50 aos, felizmente casado y asentado en su profesin, que super las secuelas de una vida de excesos y adicciones al alcohol y las drogas pesadas pero que sucumbe a la ltima tentacin: Nora. Del mismo lugar, el diario La vanguardia (Barcelona), el 2 de junio de 2005, publica una nota donde da cuenta de investigaciones cientficas acerca del funcionamiento del cerebro humano, titulada "El amor

como adiccin". Lea atentamente el texto y, luego, conteste las preguntas. La Vanguardia Josep Corbella Barcelona

EL AMOR COMO ADICCIN El cerebro humano experimenta el enamoramiento igual que una adiccin, segn investigadores de Estados Unidos que han observado por resonancia magntica qu ocurre

en el cerebro de personas que han iniciado hace poco una relacin de pareja. La investigacin responde a viejos debates sobre la relacin entre amor y sexo. Segn resultados publicados el martes en The Journal of Neurophysiology, la atraccin fsica y el enamoramiento son procesos diferentes que activan regiones distintas del cerebro. Esto explicara, segn los autores de la investigacin, por qu una persona puede encontrar atractivas a mltiples parejas potenciales, pero difcilmente enamorarse de ms de una a la vez: la regin que procesa el enamoramiento tiende a la monogamia, mientras que la que procesa la atraccin fsica

tiende a la poligamia. Los investigadores -de la Universidad del Estado de Nueva York, la Universidad Rutgers de Nueva Jersey y la Escuela de Medicina Albert Einstein de Nueva York- han analizado a 10 mujeres y 7 hombres de 18 a 26 aos que declararon llevar entre 1 y 17 meses "intensamente enamorados". Las resonancias magnticas indican que las regiones del cerebro que se activan cambian a medida que una relacin de pareja madura. En las parejas que llevan entre uno y siete meses juntas se observa una hiperactividad en reas involucradas en las adicciones, sobre todo el rea

tegmental ventral y el ncleo caudado. Pero en parejas que llevan ms tiempo juntas empieza a activarse el plido ventral, que parece vital para establecer relaciones duraderas. El estudio muestra que, a medida que pasan los meses y unas reas del cerebro se activan, otras se desactivan. Esto explicara por qu una relacin duradera y gratificante no impide que una pareja pueda verse sorprendida por un enamoramiento imprevisto: mientras el rea del cerebro que garantiza la relacin estable permanece activa, la habitacin de la pasin, en el rea tegmental ventral, se encuentra disponible.

Pero el resultado que los autores de la investigacin consideran ms relevante es que el enamoramiento no es una emocin sino ms bien una adiccin. "No hemos encontrado ningn patrn emocional consistente", declara en un comunicado Arthur Aron, codirector de la investigacin de la Universidad del Estado de Nueva York,. Las emociones del enamoramiento, advierte Aron, pueden oscilar de manera catica entre la euforia, la ansiedad, el enfado, la tristeza o la alegra. Por el contrario, "todos nuestros voluntarios mostraron una actividad intensa en las regiones de motivacin y recompensa del cerebro". Estas regiones son las

mismas que se activan en las adicciones. Igual que en una adiccin, los investigadores han observado que el enamoramiento se asocia a intensas descargas de dopamina en el centro del cerebro. Y otro punto en comn con las adicciones: cuando una persona que se encuentra en esta fase efervescente es rechazada por su pareja, presenta un patrn de actividad cerebral similar al de un sndrome de abstinencia, segn un nuevo estudio del mismo equipo de investigacin difundido por The New York Times. Este sndrome de abstinencia explicara por qu muchas rupturas, ms que inhibir el deseo de estar con la otra persona, lo

acrecientan. Ignasi Morgado, catedrtico de Psicobiologa de la Universitat Autnoma de Barcelona (UAB), record ayer que los estudios de Samir Zeki y Andreas Bartels en el Colegio Universitario de Londres han mostrado una inhibicin de la actividad del crtex prefrontal -la principal sede de la racionalidad en el cerebro- en personas que declaran estar muy enamoradas. "Por lo tanto, parece hacer falta una cierta irracionalidad para el enamoramiento", seal. Las principales reas del cerebro involucradas en el enamoramiento trabajan a nivel inconsciente y son comunes a todos los mamferos. Los

autores del estudio recuerdan adems que hay una especie de ratones de las praderas que establecen relaciones mongamas duraderas gracias a la activacin del mismo rea del cerebro que las personas. Todo ello apunta a que "el enamoramiento es posiblemente un tipo de impulso bsico de los mamferos para optimizar el proceso de apareamiento", afirma Helen Fisher, codirectora del estudio. Lo cual, sin embargo, no significa que la experiencia del enamoramiento sea igual en la especie humana que en otros mamferos, ya que el cerebro humano tiene una capacidad de ser consciente del proceso, superior al de

cualquier otra especie. Y tampoco significa, advierten los investigadores, que el enamoramiento pueda reducirse a los procesos fisiolgicos que se observan en las resonancias magnticas, ya que es un fenmeno complejo condicionado por influencias culturales. Josep Corbella. (2005, junio 2). El amor como adiccin. La vanguardia. A. Segn el mtodo hipotticodeductivo en su versin falsacionista: a. Cul es el problema? b. Cul o cules son las hiptesis de partida de los neurlogos? c. Qu consecuencias observacionales pondran a prueba estas hiptesis?

d. Considera suficiente la base observacional ofrecida por los neurlogos mencionados? Justifique su respuesta. e. Se ha demostrado la verdad de alguna hiptesis, segn este informe? Justifique su respuesta. B. Encuentra falsables los siguientes enunciados? Justifique sus respuestas en cada caso. a. "El enamoramiento es un fenmeno complejo condicionado por influencias culturales". b. "El enamoramiento es posiblemente un tipo de impulso bsico de los mamferos para optimizar el proceso de apareamiento." ACTIVIDAD 4. MTODO

HIPOTTICO-DEDUCTIVO. ACERCA DE LOS TIPOS DE HIPTESIS En el libro de la bibliografa encontrar la distincin entre distintos tipos de hiptesis que pueden participar en la puesta a prueba de una teora: hiptesis fundamental, hiptesis auxiliar e hiptesis ad hoc. Cuando hay que derivar consecuencias observacionales de las hiptesis principales, a menudo hay que hacer suposiciones adicionales ya que estas hiptesis principales, por s solas, no permiten derivar directamente consecuencias observacionales. A estas suposiciones adicionales se las llama "hiptesis auxiliares". Se supone que estas hiptesis han sido contrastadas con anterioridad, de forma independiente,

con la hiptesis principal y pueden o no formar parte de la disciplina a la que pertenece la hiptesis principal. Una hiptesis auxiliar se considera ad hoc cuando no puede ser contrastada de forma independiente de la principal y es usada para salvar a la principal de ser refutada. Lea el siguiente texto y, luego, responda las preguntas. Martes 6 de enero de 2009

LA ESFERIDAD DE LA TIERRA 2: DE LA ANTIGEDAD A COLN

La idea de una Tierra esfrica es muy antigua y se remonta al menos a los tiempos de la filosofa griega clsica y posiblemente tambin a la filosofa india antigua. El concepto de una Tierra esfrica desplaz a antiguas creencias sobre una Tierra plana. En el pensamiento mesopotmico se consideraba que el mundo era un disco plano flotando en el ocano y este retrato de la Tierra fue plasmado en los primeros mapas de la Tierra realizados por Anaximandro y Hecateo de Mileto. [...] Los primeros filsofos griegos se refieren a una Tierra esfrica aunque un tanto ambiguamente. Pitgoras (nacido en 570 a.C.) basaba la

creencia de que la Tierra y los dems planetas eran esfricos porque, para l, el slido geomtrico ms armnico era la esfera. [... ] Eratstenes (276-194 a.C.) no slo crea que la Tierra era redonda, sino que adems estim sus dimensiones. No posea computadoras, ni satlites de reconocimiento, tan slo una mente brillante y conocimientos de trigonometra.

Eratstenes escuch que al sur en Siena durante el solsticio de verano el Sol permaneca en el cnit de forma que los objetos no arrojaban sombra alguna. Se dice que

Eratstenes contrat a otra persona para medir la distancia entre Siena y Alejandra mediante la cuenta de sus pasos. Al llegar el solsticio de verano se hicieron dos observaciones simultneas. Mientras que en Siena una estaca no arrojaba sombra ninguna a medioda, en Alejandra exista una pequea sombra. Eratstenes midi el ngulo de la sombra que result ser de unos 7,2, como conoca la distancia a Siena, por un sencillo clculo trigonomtrico calcul la circunferencia de la Tierra en 250.000 estadios (una medida griega de longitud) traducido a valores actuales nos damos cuenta de que

Eratstenes se equivoc en apenas un 3% en su estimacin, sin duda un logro impresionante. [...]


[20]

a. Cul era la hiptesis principal de Eratstenes? b. Qu consecuencias observacionales se derivaban de ella? c. Qu pudo probarse, finalmente? d. Encuentra alguna hiptesis auxiliar implcita en este experimento, entre las que se mencionan a continuacin? Justifique su eleccin. i) La forma en que caan los rayos del sol. ii) La distancia de Siena a Alejandra.

iii) La existencia de dos obeliscos. e. Reflexione acerca de la importancia de las hiptesis auxiliares. Puede una hiptesis auxiliar falsa malograr la aceptacin de la hiptesis principal? Por qu?

Temas de la Unidad 5.1. Qu significa "explicar"? 5.2. Concepto de explicacin cientfica 5.3. Modelos de explicacin cientfica 5.4. La especificidad de las ciencias sociales: intencionalidad y explicacin por mecanismos 5.5. Dimensin explicativa y dimensin predictiva 5.6. El enfoque comprensivista en ciencias sociales Bibliografa obligatoria Unidad 5 en la Gua de Estudio de IPC, producida por UBA XXI y editada por Eudeba, a partir de la edicin de 2010.

Asti Vera, C. y Ambrosini, C. (2009). Captulo 6. En Argumentos y teoras. Aproximacin a la Epistemologa. Buenos Aires: Educando. Beraldi, G. (2010). Documento de ctedra: La tensin entre explicacin y comprensin. El problema de la explicacin en las ciencias sociales. En esta Gua de Estudio de IPC, Buenos Aires: Eudeba; y en el Campus virtual de UBA XXI. BIBLIOGRAFA COMPLEMENTARIA Dez, J. A y Moulines, C. U. (1999). Captulo 7. En Fundamentos de filosofa de la ciencia. Barcelona: Ariel. Schuster, G. (1986). Captulos 2, 3, 4 y 5. En Explicacin y prediccin. Buenos

Aires: Clacso. Wright, G. H. von (1979). Explicacin y comprensin. Madrid: Alianza. Presentacin de la Unidad

Sabes por qu son blancas las


pescadillas?" "Nunca me lo he preguntado", respondi Alicia. "Porqu?" "Pues porque sirven para darles brillo a los zapatos y a las botas", explic el Grifo con gran solemnidad, "por lo blancas que son ". Alicia se qued de una pieza. "Para sacar brillo!", repeta, sin saber cmo explicrselo. "Pues claro! A ver! Cmo se

limpian los zapatos?", le pregunt el Grifo. "Quiero decir, cmo se les saca brillo? Alicia se mir los pies y reflexion un poco antes de dar una contestacin: "Con negro de betn me parece". "Pues bajo el mar a las botas y a los zapatos se les da con blanco de pescadilla", interpuso el Grifo con voz pretenciosa: "Ya lo sabes." (Carroll, L. Alicia en el pas de las maravillas)

En el captulo 6 de la bibliografa, se examinan los modelos de explicacin cientfica ms transitados por la Epistemologa contempornea as como los debates que se generan en torno a las modalidades explicativas en ciencias sociales. Adems de los modelos de explicacin nomolgico-deductivo y estadstico-inductivo, en esta parte se debe prestar atencin a las caractersticas de la explicacin gentica y de la explicacin teleolgica.

En este texto resulta de especial inters la revisin de la particular posicin del filsofo noruego Jon Elster (n. 1940) quien, a partir de sus estudios sobre distintos modelos de racionalidad, considera que el criterio de clasificacin de las ciencias debe tomar en cuenta las distintas estrategias explicativas. Reconoce tres modalidades bsicas de explicacin: causal, funcional e intencional. En este orden, los campos de investigacin a los que se aplica cada una de stas son: ciencias fsicas, ciencias biolgicas y ciencias sociales. La explicacin causal, que es el modelo nico y cannico de explicacin en el campo de la fsica, tambin se emplea frecuentemente en los

otros dos campos de investigacin. Elster critica el uso de explicaciones funcionales en ciencias sociales y reserva para estas ciencias el uso de las explicaciones por mecanismos, de all la presentacin de este tema que cierra el captulo dedicado a dicho tema en la bibliografa. El material de lectura de esta unidad se completa con el Documento de Ctedra escrito por Gastn Beraldi, La tensin entre explicacin y comprensin. El problema de la explicacin en las ciencias sociales, en el que aparece el cuestionamiento a las versiones explicacionistas en las ciencias sociales y la bsqueda de mtodos propios. En este artculo se presentan a los

principales representantes del "comprensivismo", siguiendo el anlisis de los filsofos Wilhelm Dilthey (18331911, alemn) y Georg H. von Wright (1916-2003, finlands), identificados en oposicin al Positivismo como antipositivistas aunque no representan un grupo homogneo ni permiten reducir sus posiciones a unos pocos postulados comunes. El rasgo comn del enfoque positivista es el monismo metodolgico y una posicin "explicacionista" para el caso de las ciencias sociales. Aqu es importante advertir la oposicin de estos autores al monismo metodolgico y la bsqueda de estrategias explicativas propias para las ciencias sociales alrededor del concepto de

"comprensin".

Actividades de aprendizaje
MATERIALES DE LECTURA I LA EXPLICACIN CIENTFICA
Mientras que describir es responder a la pregunta acerca de cmo es algo, explicar es responder a la pregunta de por qu ese algo es o sucede. O sea que explicar es dar cuenta de la produccin y/o existencia de algn fenmeno. Toda teora cientfica puede ser vista como una gran

explicacin acerca de algn fenmeno, evento, suceso o proceso. Mas no toda explicacin cientfica constituye por s sola una teora completa. Sin embargo, no hay teora sin explicacin, de all la importancia que reviste este tema. Empleamos el trmino "explicar" en distintos sentidos: Explicar qua narrar: "Ana me explic su viaje a China". Explicar qua elucidar: "Ana me explica las reglas de la gramtica". Explicar qua respuesta a una pregunta por qu": "Ana me explic por qu decidi viajar a China". Partes. Toda explicacin tiene dos partes: el explanandum y el explanans. El enunciado que contiene

el hecho a explicar se denomina explanandum y el conjunto de enunciados que conforman la explicacin propiamente dicha, se denomina explanans. El explanandum puede describir un hecho puntual, por ejemplo "se inunda el patio de mi casa", o una regularidad, "el corcho flota en el agua". En ambos casos el explanandum encierra el enigma de por qu ocurren tales cosas, enigma que el explanans va a resolver. Requisitos. Para que una explicacin sea cientfica debe ser atinente y contrastable. La atinencia o relevancia explicativa es la pertinencia del explanans respecto al explanandum y la contrastabilidad es

la condicin por la cual todo los enunciados de la explicacin tienen que poder ser puestos a prueba. Tipos. Hay varios tipos de explicacin: genticas, teleolgicas o funcionales, intencionales, nomolgico-deductivas y las estadstico-inductivas. Explicaciones nomolgico-deductivas y estadstico-inductivas. En ambos tipos de explicacin el explanans est formado por leyes y condiciones iniciales. Las leyes son los enunciados ms generales a los que los enunciados observacionales se subsumen y las condiciones iniciales son los enunciados que describen las situaciones de hecho que

acompaaron la produccin del fenmeno que el explanandum describe. Modelo nomolgicodeductivo L1,L2,L3..................Ln C1,C2,C3................Ln

Explanans

Explanandum

La diferencia entre los dos tipos de explicacin deriva de la diferencia

que hay entre las leyes que estn en el explanans de uno u otro modelo. En efecto, las leyes pueden ser de probabilidad mxima, o bien de alta probabilidad. Las leyes de probabilidad mxima son aquellas tales como la Ley de Gravedad, la Ley de Boyle y Mariot, el Principio de Arqumedes, etc., todos ellos enunciados nomolgicos que describen regularidades de la naturaleza. En cambio, las leyes de alta probabilidad son enunciados estadsticos que describen regularidades probables, con un considerable margen de excepciones. Se encuentran en este ltimo grupo la mayor parte de las leyes estadsticas

de las ciencias sociales, por ejemplo las que vinculan enfermedades virsicas y contagio (Salud), o pobreza e ndice de mortalidad infantil (Sociologa), ausencia de imagen paterna y anorexia u homosexualidad (Psicologa), etc. Esta diferencia en las leyes es muy importante porque en las explicaciones nomolgicodeductivas, dndose por verdaderas las leyes de probabilidad mxima que cubren la totalidad de los casos, puede inferirse el explanandum del explanans como una conclusin deductiva. En cambio, como en el explanans del modelo estadsticoinductivo, las leyes probabilsticas no

cubren la totalidad de los casos -hay un cono de sombra en donde estn los que no siguen la ley-, aun siendo este explanans verdadero, no hay garanta de verdad para el explanandum. Esta situacin es la que se indica con la doble raya que antecede al explanandum en el esquema de las explicaciones estadstico-inductivas (EI).

ACTIVIDAD 1. ACERCA DE LOS


DISTINTOS MODELOS DE EXPLICACIN En los siguientes ejemplos, determine qu modelo de explicacin podra aplicarse en cada caso y justifique su eleccin:

a. El enfermo se cur porque lo trataron con penicilina. b. Juan contrajo hepatitis por usar jeringas contaminadas. c. Mara compr un auto para dormir dos horas ms a la maana. d. Ernesto choc con el auto porque manejaba alcoholizado. e. Los huesos sirven para que el cuerpo se mantenga erguido. f. Las golondrinas emigran en otoo porque necesitan vivir en lugares clidos. g. El Papa convoc a un Concilio porque quiere reformar el ritual de la misa. h. La babosa vive en lugares oscuros y tiene hbitos nocturnos porque no tiene

caparazn. i. El supermercado X vende la gaseosa por debajo del precio de costo porque quiere eliminar a la competencia. j. El corazn late porque la mdula determina su ritmo y fuerza. k. Juan se enferm de los pulmones porque fumaba mucho. l. Mi vecino perdi su fortuna porque es adicto a las carreras de caballos. ACTIVIDAD 2. MODELOS DE EXPLICACIN CIENTFICA. ACERCA DE LA EXPLICACIN NOMOLGICO-DEDUCTIVA Hemos visto que la explicacin nomolgico-deductiva se ajusta mejor a los casos de la Fsica. En el artculo que se encuentra a continuacin, se presenta

el especial comportamiento del agua en relacin con la Ley general de expansin trmica (la materia se dilata cuando se calienta y se contrae cuando se enfra) que presentara un comportamiento distinto. Luego de leer atentamente el artculo: a. Seale cul sera el explanandum y el explanans en el caso presentado del comportamiento del agua a distintas temperaturas. b. Determine si el explanandum menciona una regularidad o un caso particular. Por qu el agua no obedece a la Ley general de expansin trmica? Con pocas excepciones, las formas de

la materia -slidos, lquidos, gases y plasmas- se dilatan cuando se calientan y se contraen cuando se enfran. En los slidos, por lo general, esos cambios de volumen no son muy notables, pero existen: las lneas telefnicas, por ejemplo, se estiran y cuelgan ms en un da clido que en uno de invierno; los lquidos, por su parte, se dilatan en forma aprecia-ble al aumentar su temperatura; pero el agua tiene comportamientos notables: a temperaturas cercanas a su punto de congelacin, a menos de 0 grados cuando el agua ya es hielo macizo, su volumen es bastante mayor y su densidad es menor (por esto el hielo

flota en el agua). Al calentarse el agua, hasta que su temperatura llega a 4 grados, contina contrayndose. Slo a temperaturas ms altas se comienza a dilatar y la expansin contina hasta llegar al punto de ebullicin. Revista Muy interesante. N 290 (2009, diciembre).

ACTIVIDAD 3. LA ESPECIFICIDAD DE LAS CIENCIAS SOCIALES. LA


MODALIDAD EXPLICATIVA COMO CRITERIO DE CLASIFICACIN DE LAS CIENCIAS Elster plantea que hay diferentes modalidades de explicacin: la causal,

la funcional y la intencional. En su libro El cambio tecnolgico[21] propone el siguiente esquema que solamente agregamos a los fines de visualizar la distincin entre conducta intencional y conducta racional.

En el texto, a continuacin, Elster dice que puede haber intencionalidad sin racionalidad. A. Lea el texto.

B. Explique con sus palabras por qu puede haber intencionalidad sin racionalidad. C. Elabore tres ejemplos de conducta intencional-racional. Intencionalidad y racionalidad segn Jon Elster Puede haber intencionalidad sin racionalidad? O racionalidad sin intencionalidad? [...] Evidentemente todo depende de cmo definimos el concepto de racionalidad. Si solamente queremos decir "adaptacin en el sentido de maximizacin local", hemos visto que puede haber racionalidad no intencional. Pero cualquiera sea el

modo en que definimos la racionalidad, creo que debera reservarse para los casos en que tiene poder explicativo. Es decir que nunca habra que caracterizar una creencia, una accin o un modelo de conducta como racional a no ser que se est dispuesto a afirmar que la racionalidad explica que lo que se dice es racional. [...] Para los propsitos presentes es suficiente observar que la racionalidad mnimamente implica consistencia de metas y creencias. Para calzar una cua entre intencionalidad y racionalidad, debemos demostrar que puede haber deseos inconsistentes y creencias inconsistentes. Con

respecto a las creencias inconsistentes, demostrar su posibilidad mediante una historia sobre Niels Bohr, que cierta vez tena una herradura sobre la puerta. Cuando se le pregunt si la haba colocado all porque crea que le traera suerte, contest: "No, pero me dijeron que traen suerte incluso a quienes no creen en ellas". Arreglando un poco la historia, resulta lo siguiente: 1) Niels Bohr cree "La herradura no me traer suerte". 2) Niels Bohr cree "Las herraduras le traen suerte a quienes no creen que les traer suerte". Aunque las creencias entre comillas

son consistentes entre s, ambas no pueden ser ciertas y ser credas (por Bohr). Pero un sistema de creencias es consistente solamente si existe un mundo posible en el que son todas ciertas y credas. Si a favor del argumento, suponemos que Bohr no estaba haciendo una broma y que en realidad coloc una herradura sobre la puerta porque quera suerte y crea aunque inconsistentemente que le traera suerte, tenemos el caso de una accin claramente irracional y, sin embargo, explicada intencionalmente.
[22]

ACTIVIDAD 4. DIMENSIN
EXPLICATIVA Y DIMENSIN

PREDICTIVA. EL HOMBRE NO LOGR CUMPLIR SU GRAN ILUSIN: PREDECIR EL FUTURO Despus de leer el siguiente texto, realice la actividad propuesta a continuacin: Eduardo Flichman (1932-2005), argentino, fue un fsico y filsofo de la ciencia, dedicado a la investigacin y docencia en el mbito de la epistemologa y la enseanza de las ciencias. En una nota periodstica publicada en el diario Clarn, Flichman dice: "la inquietud por predecir la conducta humana todava tiene ms interrogantes que certezas". Para l, si pudisemos

predecir el futuro, si pudisemos saber con antelacin lo que nos va a pasar, dejara de tener sentido el concepto de libertad y los hombres estaran condenados a seguir un destino. Y proporciona un ejemplo: "se pueden estudiar las constantes en las variaciones del trnsito y determinar que a las 7 de la tarde se va a ir ms rpido por una calle lateral que por la autopista porque sta ltima est colapsada". Pero cuando se dan a conocer los resultados de la investigacin, ya ese conocimiento altera los comportamientos de los conductores y, si todos eligen ir por las calles laterales, entonces resultar falsa la

prediccin y all se ir mucho ms lento y ms rpido por la autopista. En ciencias sociales, expresa Flichman, "el conocimiento modifica las conductas". Dada esta situacin: "Frente a la prediccin de una epidemia de gripe A durante el mes de julio de 2009, en Argentina se tomaron medidas preventivas. La epidemia no se produjo en los niveles previstos y eso llev a algunos a pensar que se trat de una falsa alarma destinada a sembrar miedo en la poblacin y a aumentar la venta de medicamentos". Responda: El hecho de que no se haya cumplido la epidemia en el grado

previsto, indica que los bilogos se equivocaron en la prediccin? ACTIVIDAD 5. ENFOQUE COMPRENSIVISTA DE LAS CIENCIAS SOCIALES 5.1 Teniendo en cuenta lo estudiado en el enfoque comprensivista de las ciencias sociales, ubique los trminos en las columnas segn corresponda. Trminos Explicacin Comp Por qu? Hempel Pluralistas Popper Dilthey Cmo?

Positivistas Descripcin Busca explicar Teleolgico Monistas Antipositivistas Aristteles Rechazo de las ciencias naturales como patrn metdico Simmel Intencionalidad

Galileo Comte Historia Platn Unidad de mtodo Hermenutica Busca comprender Droysen Reduccin del mtodo de las ciencias sociales a las naturales Causalidad

Geisteswissenschaften Empata Reduccionismo Sentido Vertehen Weber Tiempo Situacin Prediccin Accin Dray 5.2. Responda el siguiente cuestionario: a. Qu pregunta se hacen los partidarios de la comprensin en las ciencias sociales y cul los de la explicacin?

b. Realice un cuadro comparativo entre las posiciones de Hempel, Popper, Dilthey, Droysen y von Wright. c. Cul es la diferencia entre monismo metodolgico y pluralismo metodolgico? d. Von Wright sostiene que el problema de la explicacin y la comprensin en las ciencias se remonta a dos tradiciones cientficas. Cules son esas dos tradiciones y a qu problema se orienta cada una? Indique, asimismo, con qu corrientes epistemolgicas contemporneas puede trazar una correspondencia. e. Por qu motivo los partidarios del enfoque comprensivista rechazan el monismo metodolgico? Qu razones

brinda Dilthey al respecto? f. Quin introdujo la dicotoma metodolgica entre "explicacin" y "comprensin"? g. Describa brevemente cul es la tarea de la comprensin (hermenutica) como mtodo cientfico a partir de lo expuesto por Simmel, Dilthey, von Wright, Weber y Echeverra. h. Cul es la posicin de von Wright en esta tensin entre explicacin y comprensin? i. Mencione cul sera el objetivo de la ciencia para cada una de estas posiciones. 5.3. Dadas las siguientes situaciones: A. Supongamos que un alumno se copia en un examen y es descubierto por el

profesor. De acuerdo con el modelo comprensivista de von Wright, responda: a. Qu deberamos preguntarnos? b. Cmo daramos cuenta de ese hecho temporal y causalmente? c. Cmo puede relacionar este hecho con los conceptos de intencin, accin, libertad y responsabilidad? d. Se podra brindar una explicacin que siempre fuese la misma para ese hecho? B. Supongamos ahora que una piedra cae desde una montaa y que muere una persona al ser golpeada por esa piedra. Conteste las mismas preguntas anteriores.

5.4. Dada la siguiente situacin: Supongamos el caso de un conocido hecho: el incendio en el centro comercial de Paraguay. All mueren ms de cien personas. Se hace la investigacin judicial y se solicita la opinin de distintos expertos para comprender por qu o cmo murieron esas personas. A. De acuerdo con el modelo de van Fraassen, qu respondera... a. ... el perito mdico b. ... el fiscal que investig el caso c. ... un ingeniero d. ... un urbanista e. ... la polica f. ... el abogado defensor g. ... el resto del pblico asistente al

centro comercial, pero que no fue alcanzado por el incendio h. ... el dueo del centro comercial B. Segn van Fraassen, todas esas respuestas son legtimas?, de qu depende su legitimidad? 5.5. Dada la siguiente situacin: Imagine que usted debe realizar una investigacin sobre un autor ya fallecido (filsofo, epistemlogo, socilogo, jurista, poeta, arquitecto, economista, etc.) y desea "comprender" su pensamiento y "explicar" el mismo mediante un escrito monogrfico para que el pblico tambin lo conozca, o lo conozca mejor. De acuerdo con el modelo de la comprensin (hermenutica) brindado

por los aportes de Simmel, Dilthey, Schltz, Gadamer, Habermas, Vattimo, Ricoeur, responda: a. Qu debera hacer usted? b. Qu datos debera investigar y de qu manera? c. Por qu debera indagar ciertos datos que, en apariencia, no seran tiles? d. Se llegara al mismo resultado de su investigacin si no recabase toda esa informacin? Justifique.

Introduccin

En Argumentos y teoras. Aproximacin a la epistemologa de Carlos Asti Vera y Cristina Ambrosini, encontramos que:
"[...] el criterio que distingue entre ciencias naturales y sociales es menos firme que el que diferencia a las ciencias formales y a las fcticas, ya que, a menudo no hay acuerdo acerca de dnde ubicar la lnea divisoria [entre un tipo de ciencia y otro], lo que tambin afecta a las consideraciones sobre los mtodos a emplear." (2009, 189)

Esta cuestin de la metodologa de las ciencias fcticas comienza a presentarse como problemtica hacia mediados y fines del siglo XIX y principios del XX con las consideraciones realizadas por Johann Gustav Droysen (historiador alemn, 1808-1884) y, fundamentalmente, por Wilhelm Dilthey (filsofo alemn, 1833-1911). Hasta ese

entonces se entenda que el mtodo de justificacin, modelo en las ciencias fcticas, era el correspondiente a las ciencias naturales (el inductivista y posteriormente, ya en el siglo XX, el hipottico-deductivo), y en ese sentido, las ciencias sociales se subsuman bajo dicho modelo. Esta concepcin, en lneas generales, se la conoce bajo la denominacin de monismo metodolgico.[23] La problemtica de la distincin (pluralismo metodolgico)[24] o no (monismo metodolgico) entre los mtodos de la ciencias fcticas trae aparejada la cuestin de si estas ciencias (tanto naturales como sociales) explican -causalmente-o describen -para

comprender. Y aqu nuevamente nos encontramos con estas dos posiciones epistemolgicas: monismo y pluralismo. Al inicio del captulo 6 de Asti Vera y Ambrosini (2009. 214), se indica que en las ciencias fcticas hay una diferencia entre explicar y describir, dando cuenta de que la primera responde a la pregunta del "porqu" y la segunda a la pregunta del "cmo". As, si nos preguntramos cul sera el objetivo de la ciencia: explicar o describir?, la posicin mayoritaria dentro de la comunidad cientfica se inclinara por la afirmacin que la ciencia debe explicar y no describir, incluso frente a las diferencias existentes entre ciencias naturales y

ciencias sociales o humanas o del espritu. Por otra parte, el campo del conocimiento social estuvo dominado por el empirismo lgico hasta la aparicin de un conjunto de teoras epistemolgicas que en los ltimos treinta aos cambiaron el panorama rechazando la idea de que pudiera haber observaciones tericamente neutrales. Este distanciamiento con el criterio de neutralidad en el campo de la ciencia natural permiti que emergieran con renovada vitalidad tradiciones ya existentes como la fenomenologa y la hermenutica; esta ltima tal como la desarrollaron Hans-Georg Gadamer (filsofo alemn, 1900-2002) y Paul

Ricoeur (filsofo francs, 1913-2005) siguiendo y recreando a Dilthey. Y por otra parte, adquiri cierta relevancia la teora crtica representada por Jrgen Habermas (filsofo alemn, n. 1929). Lo ms importante de la situacin de la epistemolga de los ltimos aos es la consideracin de la ciencia como una actividad interpretativa y, entonces, los problemas de significado y comunicacin adquieren relevancia en la epistemologa.

1. El problema de la "explicacin" en las ciencias fcticas

Particularmente respecto del problema de la explicacin en las ciencias sociales, Carl Hempel (filsofo, epistemlogo y lgico germanoestadounidense, 1905-1997) en La explicacin en la ciencia y en la historia (1981), sostiene que los dos tipos bsicos de explicacin cientfica, el modelo nomolgico-deductivo y el modelo estadstico-inductivo, sirven de soporte al modelo de explicacin caractersticamente histrico. Y defiende que la explicacin histrica aspira, adems, a demostrar que los hechos no ocurren por azar, sino que poda esperarse su ocurrencia en vista de ciertos antecedentes o condiciones simultneas (von Wright, 1979. 44) y si

no podemos formular leyes generales en las explicaciones histricas, eso se debe a la excesiva complejidad de tales leyes y a la insuficiente imprecisin con que las conocemos. De esta manera, las explicaciones histricas constituyen esbozos de explicacin (von Wright, 1979. 45). Algo similar sostiene Karl Popper (socilogo y espistemlogo austrobritnico, 1902-1994), representante tambin del modelo de cobertura legal (nomolgico-deductivo), quien afirma que la razn de que las leyes no sean formuladas en las explicaciones histricas es que ellas son demasiado triviales para merecer una mencin explcita, y que las damos

implcitamente por supuestas (von Wright, 1979. 45). Sin embargo, la cuestin acerca de si la ciencia, y particularmente las ciencias sociales, deben explicar o describir no est zanjada. Respecto de esta cuestin, Georg Henrick von Wright (filsofo finlands, 1916-2003), en su obra Explicacin y Comprensin (1979), emprende el tratamiento de la explicacin en la historia y en las ciencias sociales, y analiza las diferencias en los mtodos explicativos entre las ciencias humanas y las ciencias naturales. Desde el punto de vista metodolgico, se suelen sealar dos actitudes con relacin al problema del mtodo, lo que se llama monismo

metodolgico (hay un solo modelo tanto para las ciencias naturales como para las sociales), y lo que se denomina, sensu contrario, pluralismo metodolgico. En este sentido, en la base de la discusin entre explicacin y comprensin (descripcin) est la cuestin de dnde reside el origen de la controversia. Hempel indica que los factores que han estimulado la investigacin cientfica son fundamentalmente dos: "[...] Una es el deseo persistente del hombre por mejorar su posicin estratgica en el mundo por medio de mtodos confiables para la prediccin y, cuando sea posible, el control de los

acontecimientos. [...] Pero adems, una segunda motivacin es la insaciable curiosidad intelectual del hombre [...]" (Hempel, 1981). 1.1. ORIGEN DE LA CONTROVERSIA Recordemos que la dicotoma existente entre explicacin y comprensin hunde sus races en la historia de las ideas y se relaciona con el problema de si la construccin terica es intrnsecamente un mismo gnero de empresa tanto en las ciencias naturales como en las ciencias sociales, humanas o del espritu. von Wright sostiene que el problema parte de dos tradiciones cientficas en la historia de las ideas. Una es la

aristotlica y la otra la galileana que se remonta a Platn. La primera se vincula a los esfuerzos del hombre por "comprender" las cosas "teleolgicamente", es decir, comprender las cosas como tendientes a un fin, a un objetivo; y la segunda por "explicarlas causalmente" (von Wright, 1979. 18). Y considera que cuando el filsofo se cuestiona el tipo de conocimiento, se encuentra con estas dos tradiciones que difieren en el planteamiento de las condiciones a satisfacer por una explicacin cientfica. En el mismo sentido, en la historia de la ideas, y particularmente de las ciencias, nos encontramos frente a dos corrientes epistemolgicas fuertes, la positivista y

la antipositivista. Y segn von Wright, la positivista, cuyos representantes principales son Auguste Comte (socilogo francs, 1798-1857) y John Stuart Mill (filsofo y economista ingls, 1806-1873), queda vinculada a la tradicin galileana a travs de su monismo metodolgico, y por otro lado, la antipositivista, cuyas figuras ms significativas son Dilthey y Max Weber (filsofo, economista y socilogo alemn, 1864-1920), entre varios otros, representa una tendencia mucho ms diversificada y heterognea que el Positivismo, rechazando el monismo metodolgico y rehusndose a establecer como patrn metodolgico a las ciencias naturales exactas como

ideal regulador nico y supremo de la comprensin racional de la realidad. Para caracterizar a esta ltima corriente, von Wright utiliza el nombre de hermenutica -cuestin que trataremos en detalle en los prximos pargrafos- y la vincula con la tradicin aristotlica (von Wright, 1979. 23-24). 1.2. EL PROBLEMA DEL MTODO Y EL REDUCCIONISMO Antes de incursionar en las concepciones de la corriente "comprensivista" y en su enfoque hermenutico, hay que tener en cuenta que, por un lado, desde el Positivismo, se sostiene la unidad de mtodo

tendiendo a subsumir el mtodo de las ciencias sociales al de las naturales; establecindose como ideal metodolgico las ciencias fsicomatemticas; ponindose nfasis en la explicacin y prediccin; dando cuenta de que la explicacin es causal; y rechazando a las explicaciones finalistas o teleolgicas como acientficas. Y mientras, por otro lado, el antipositivismo, la otra posicin, tiende a sostener un contraste entre, por una parte, las ciencias que al modo de la fsica, la qumica o la fisiologa, aspiran a generalizaciones sobre fenmenos reproducibles, y por otro parte, las ciencias que, como la historia, buscan "comprender" las peculiaridades

individuales y nicas de sus objetos, impugnando el enfoque positivista de la explicacin. Las ciencias sociales o humanas no pueden, sostiene Dilthey, pretender la comprensin de la vida a travs de categoras externas a ella -como lo pretende el Positivismo-, sino a travs de categoras intrnsecas, derivadas de ella misma, por tal motivo no deben explicar, sino comprender y no lo pueden hacer mediante los mtodos de las ciencias naturales sino mediante los suyos propios. Es decir, para "explicar" (entre comillas porque Dilthey utiliza "comprender") la vida, para comprender las estructuras del sentido de la vida, no podemos usar categoras, conceptos de

las ciencias naturales como leyes lgicas o razonamientos deductivos o inductivos, porque todo esto son construcciones formales realizadas por el hombre, y no son "la vida", es decir, no pertenecen a la categora de la vida esas categoras cientficas, ya que son slo objetivaciones que realiza el hombre para explicarla, pero no son categoras propias de ella. Las categoras propias de la vida tienen que ver con lo que Dilthey llama "vivencias" y que son, a grandes rasgos, lo que nos ocurre, y lo que nos ocurre histricamente. Son los acontecimientos de la vida, el querer, el sentir y el representar. Y por eso, para Dilthey, la comprensin en las "ciencias del

espritu" se da porque la vida se exterioriza en textos, que dan cuenta de las vivencias del que lo escribi y, entonces, nos permiten comprender, adems del texto, a su autor.

2. La "comprensin" en las ciencias sociales


La corriente antipositivista, por su parte y de diversas maneras, sostiene que el objetivo de las ciencias naturales es "explicar", y el de la historia y las ciencias sociales, es "comprender" los fenmenos que ocurren en su mbito; y es el mismo Droysen el que introdujo tal dicotoma metodolgica y acu los

nombres de explicacin y comprensin. Estas ideas metodolgicas fueron luego desarrolladas con mayor profundidad y sistematizadas por Dilthey, quien rechaza la tendencia de fundar un conocimiento humano siguiendo los procedimientos de las ciencias naturales. Es el mismo Dilthey quien para designar todo el dominio de aplicacin del mtodo de comprensin se sirvi del trmino "ciencias del espritu" (Geisteswissenschaften) que conocemos tambin por ciencias humanas o ciencias sociales, y en el habla inglesa por Moral science (von Wright, 1979. 24). El uso cotidiano del lenguaje no hace una distincin aguda entre los trminos

"explicar" y "comprender", y as se podra decir que, cualquier tipo de explicacin proporciona una comprensin de las cosas. Pero "comprensin" cuenta adems con una connotacin psicolgica de la que carece el trmino "explicacin". Se considera que la comprensin, como mtodo caracterstico de las ciencias del espritu, es segn Simmel (filsofo y socilogo alemn, 1858-1918) una forma de empata o recreacin en la mente del estudioso de la atmsfera espiritual, pensamientos, sentimientos y motivos, de sus objetos de estudio (von Wright, 1979. 24). Sin embargo, no es esta la nica caracterstica que distingue a la

comprensin de la explicacin. Von Wright sostiene que la comprensin se encuentra ligada adems con la "intencionalidad" de una manera en que la explicacin no lo est. Se comprenden los objetivos y propsitos de un agente, el significado de un signo o de un smbolo, el sentido de una institucin social, de un rito religioso. Esta dimensin intencional, sostiene el mismo von Wright, ha llegado a jugar un papel relevante en la discusin metodolgica ms reciente a travs de la hermenutica (von Wright, 1979. 24-28). El gran objetivo de Dilthey consisti en desarrollar una metodologa apropiada para el "entendimiento" (Verstehen) de las obras humanas, que eluda el

reduccionismo y mecanicismo de las ciencias naturales, ya que la vida debe ser entendida a partir de la propia experiencia de vida. En tal sentido es central para Dilthey esta distincin entre ciencias naturales y ciencias del espritu. Las primeras descansan en el concepto de "fuerza" propuesto por la fsica y en las matemticas; las ciencias humanas, en cambio, se apoyan en el concepto de "sentido" y en la historia. Y as, el concepto clave de las ciencias del espritu es el de "entendimiento" o "comprensin" (Verstehen). Si en sociologa (ciencia social), Comte era el principal representante del positivismo, Weber se inclinaba hacia el

lado opuesto y de una manera ms slida que Comte. Uno de los elementos que aparecen en la concepcin weberiana del mtodo cientfico es el de "comprensin" (Verstehen). Esta comprensin no significaba una identificacin completa con el sujeto investigado, sino que, para comprender, por ejemplo las obras de Shakespeare, yo no necesito ser Shakespeare, pero s tengo que poder de alguna manera concebir qu puede estar pensando Shakespeare en una determinada situacin. As, comprender es la capacidad que tiene cualquier ser humano de entender a otro ser humano en una situacin, aunque l no haya vivido exactamente esa situacin. Es,

aunque no en el ms completo sentido, lo que comnmente conocemos por "ponerse en el lugar del otro" para comprender una actitud, una accin, etc. El sentido propio de la comprensin (o entendimiento) siempre se halla en un contexto de horizonte que se extiende hacia el pasado y el futuro. La historicidad y la temporalidad son dimensiones inherentes e inevitables de toda comprensin. Por ejemplo, si quisiramos comprender a Scrates y su pensamiento, lo que deberamos hacer es tender un puente con el pasado y con el presente, es decir, no podramos comprenderlo plenamente si slo lo analizramos desde la actualidad, pero tampoco lo podramos comprender

plenamente si slo lo analizramos en su contexto, con lo cual hay que tender un puente entre pasado y presente, un puente que nos permita abrir las posibilidades del pensamiento de Scrates. De esta manera, el todo recibe su sentido de las partes y las partes slo pueden comprenderse en relacin con el todo, conformando como dira Gadamer un crculo hermenutico. Como sucede con una obra literaria, no podemos comprenderla leyendo slo un captulo (una parte), pero tampoco podemos comprenderla completa si cada captulo no le da un sentido unificador a la obra. El sentido es histrico, dinmico y contextual, es siempre parte de una determinada situacin. La interpretacin

siempre remite a la situacin en la cual se halla el intrprete, pero esto no significa que el sentido sea subjetivo y nos lleve al relativismo; el sentido es una percepcin de una relacin real dentro de un nexo anterior a la separacin sujeto-objeto en el pensamiento (Echeverra, 1993. 203205). La tradicin antipositivista procura sustituir las nociones cientficas de explicacin, prediccin y control por las interpretativas de comprensin, significado y accin. Por su parte, von Wright sostiene que la metodologa de las ciencias humanas comenz gradualmente a atraer la atencin de los filsofos analticos[25]

(positivistas y neopositivistas vinculados por la pretensin de excluir por completo a la metafsica de las ciencias), y que la filosofa analtica de la ciencia se vio inmersa en el campo de batalla de la metodologa positivista y antipositivista (von Wright, 1979. 29). Es as que hasta aproximadamente los aos 60 rein una coincidencia general en cuanto a que el modelo explicativo causal suministraba el marco de referencia idneo para el estudio de los fenmenos sociales, pero luego, y hasta cerca de 1980, la alternativa interpretativa, el modelo de la comprensin, comenz a ganar adeptos en los pases de habla inglesa. Los trabajos de varios filsofos de

raigambre analtica, positivista y neopositivista, reflejan el creciente inters, en el seno de la filosofa analtica, por el concepto de accin que est vinculado con la intencionalidad. Tal es as que, William Dray (canadiense, filsofo de la historia, n. 1921), en Leyes y Explicacin en Historia, expresa que las explicaciones de la historia son cientficas y racionales, a diferencia de lo que sostena Hempel, aunque no son ciencia de leyes generales. Dray sostiene que la historia explica y lo hace en el sentido de mostrar que una accin fue el proceder adecuado teniendo en cuenta los antecedentes. As aborda la pregunta de cmo fue posible? y no la pregunta

de por qu? Von Wright indicar, entonces, que este giro del "porqu" al "cmo" lo sita cerca de la tradicin hermenutica (von Wright, 1979. 45-52). De esta manera, en virtud del inters de algunos epistemlogos positivistas por la explicacin en las ciencias sociales, von Wright trata de tender un puente entre "explicacin" y "comprensin" demostrando que, tanto como sostenan Hempel, por un lado, o Dilthey, por el otro, no son incompatibles, sino complementarias. Influenciado por las consideraciones filosficas del segundo Ludwig Wittgenstein (filsofo y lingista austro-britnico, 1889-1951) y la filosofa neowittgensteniana, y tambin por la hermenutica, von Wright

postula que la filosofa analtica es afn a la hermenutica a partir de los conceptos de significado, intencionalidad, interpretacin y comprensin, y por compartir, adems, la preocupacin por la metodologa y la filosofa de la ciencia. Sostiene en este sentido que la comprensin en el lenguaje de la filosofa hermenutica debera diferenciarse de la empata, afirmando que se trata de una categora antes semntica que psicolgica (von Wright, 1979. 53). As von Wright indica que las explicaciones causales son tpicamente cuasi-teleolgicas: explicaciones susceptibles de formulacin en trminos teleolgicos pero cuya validez depende

de la efectividad de conexiones nmicas (leyes). Las explicaciones de este gnero suelen responder a las preguntas sobre cmo es? o cmo llega a ser posible algo? Y estas "explicaciones" tienen un lugar predominante en las ciencias humanas, permitiendo "comprender" lo que algo es o cul es la razn de que algo ocurra (von Wright, 1979.108-109). Sealando de este modo que los acontecimientos que resultan de acciones bsicas tienen lugar cuando los investimos de intencionalidad. 2.1. EL MODELO COMPRENSIVISTA DE VON WRIGHT De acuerdo con las diversas posiciones

epistemolgicas y metodolgicas que antes tratamos, podemos decir que la nocin de causalidad vale con ms facilidad para el mbito de los fenmenos naturales que en el de los humanos. Sin embargo, von Wright sostiene que si rastreamos el concepto de causalidad, veremos que se origina en nuestro conocimiento inmediato, ntimo, de la accin humana. Es decir, este concepto proviene de una proyeccin de nuestras propias acciones sobre el mundo natural. Nosotros, cultural o histricamente, experimentamos la accin, y la nocin de causa surge por una especie de proyeccin de nuestras propias acciones al mundo natural. As, si nosotros no

acturamos, no tendramos el concepto de causa. De esta manera, en lugar de pensar que yo soy la causa por la cual, supongamos, "x" muri por mi accin o mi accin fue la causa de la muerte de "x"; la causalidad en la naturaleza deriva del "saber que yo puedo" matar a una persona. O sea, invierte el orden con respecto a la idea que previamente uno poda tener. Todos los comentarios de von Wright estn dirigidos al problema de mostrar que la conducta humana es algo completamente distinto de los hechos que nos encontramos en el mundo natural, y hasta formula hiptesis de cmo en el mbito humano puede haber

procesos causales de tipo retroactivo, en los que el efecto es anterior a la causa. Por ejemplo, se piensa que un cierto proceso cerebral tiene que ser de alguna manera anterior a un movimiento muscular, porque se supone que el cerebro ordena al brazo, por ejemplo, a extenderse, pero von Wright piensa que es al revs. Es el levantar el brazo, el extenderlo, lo que produce el fenmeno cerebral, pero lo produce antes. Es decir, no es que la causa de que apret el dedo haya sido el fenmeno cerebral, sino que el fenmeno cerebral se produce porque "voy a" apretar el dedo, o "a" extender el brazo. Y sostiene que el brazo se extiende, o el dedo se aprieta porque "yo decid hacerlo". Yo lo que

decido es extender el brazo, es una decisin que tomo, decido extender el brazo y lo extiendo. Con lo cual, en el momento que decid hacerlo, y lo hago efectivamente, ese hecho, para von Wright, es primario causalmente, no temporalmente, y no lo es temporalmente porque l reconoce que no hubiera extendido el brazo si no hubiese producido un cierto estado cerebral antes. As, causalmente, primero est la decisin, segundo, el proceso cerebral y tercero, la accin. Pero temporalmente, primero est el proceso cerebral y segundo la accin. Esta cuestin se emparenta directamente con el tema de la "intencionalidad" y la "responsabilidad". Un ser humano

directamente levanta el brazo, aprieta el botn, etc., y al hacer esto, l es responsable de su accin, y slo podr ser responsable si la decisin la toma l. Y esto es lo que le lleva a decir que el fenmeno A (proceso cerebral) se da antes que B (apretar el dedo), pero B (apretar el dedo) es la causa de A (proceso cerebral). Un concepto fundamental en von Wright es el concepto de accin. La caracterizacin de este concepto consiste en la produccin de cambios que no hubieran tenido lugar de no haber mediado nuestra intervencin. Por ejemplo, en el caso de un crimen, acusamos a una persona de haberlo cometido porque no habra tenido lugar

la muerte de la otra persona de no haber mediado la accin de quien, supongamos, accion el gatillo. Pero una accin es tanto la accin propiamente dicha como la omisin, que es una accin negativa. As, la "explicacin" en el caso de las acciones humanas es completamente distinta de la explicacin de los fenmenos de tipo natural, porque en el caso de las acciones no voy a buscar leyes, y, en el caso de los fenmenos naturales, no puedo adjudicarles ni intencin, ni decisin, ni responsabilidad. von Wright sostiene entonces que lo ms adecuado para "comprender" las acciones humanas no es el esquema de

Hempel (leyes), sino el de Aristteles (interpretacin), que es el esquema que se conoce como silogismo prctico, esquema sobre las explicaciones intencionales que ya se ha visto en la bibliografa.[26] De esta manera, es posible afirmar que, en el modelo de "explicacin comprensivista" de von Wright decir que el concepto de causalidad supone el de libertad es correcto en el sentido de que nicamente a travs de la idea de hacer cosas se llega a "comprender" las ideas de causa y efecto (von Wright, 1979. 106). 2.2. EL MODELO PRAGMTICO DE VAN FRAASSEN

Ya se haba anticipado que los modelos de explicacin en ciencias sociales se vinculaban fundamentalmente con el concepto de comprensin de origen hermenutico, y habamos indicado que en la comprensin eran inescindibles nociones tales como intencionalidad y contexto, como tambin las de temporalidad, significado e interpretacin. Precedentemente se ha tratado la cuestin del "modelo de comprensin" vinculado a la intencionalidad, lo que se denomin el modelo comprensivista de von Wright. Se iniciar ahora el recorrido al modelo pragmtico de Bas van Fraassen (filsofo holands, n. 1941), vinculado fundamentalmente con

el contexto, y aunque este epistemlogo no se vincule directamente con el comprensivismo, podemos observar ciertas semejanzas con esa posicin metodolgica. Van Fraassen considera los problemas epistemolgicos ms all, no exclusivamente desde el punto de vista del anlisis de las relaciones lgicas y relaciones conceptuales (dimensin sintctica y semntica), sino tomando en cuenta lo que se denomina la dimensin pragmtica. Recordemos que la dimensin pragmtica toma en cuenta los usuarios o las circunstancias en las cuales se usan las expresiones. Con Dilthey decamos que el sentido era contextual, y la interpretacin se remite

siempre a la situacin en la cual se halla el intrprete. La circularidad del entendimiento (crculo hermenutico) tiene otra importante consecuencia, no existe realmente un punto de partida verdadero para el entendimiento. Ello significa que no es posible concebir un entendimiento carente de presupuestos. Todo acto de entendimiento, de comprensin, tiene lugar en el interior de un determinado contexto u horizonte. Ello es igualmente vlido para las explicaciones cientficas. Estas siempre requieren de un marco de referencia. As, comprendemos slo por referencia a nuestra experiencia. Van Fraassen piensa que es intil hacer una caracterizacin de las explicaciones

que no tenga en cuenta las situaciones en las cuales las explicaciones se producen, es decir, su contexto. As, si yo pregunto por qu (algo que Hempel dira que genera una explicacin) Adn comi la manzana?, por lo pronto, el tipo de respuesta que voy a admitir como adecuada tiene que ver con determinado contexto. As, una explicacin que podra ser perfectamente vlida para un contexto, podra estar fuera de lugar en otro. Lo que sostiene van Fraassen es que una pregunta, en primer lugar, siempre se refiere a un tpico, y el tpico ser el explanandum. Y as, la propia suposicin de que el explanandum es verdadero depende de una cuestin

contextual. Por otro lado, van Fraassen sostiene que puede haber respuestas distintas a la misma pregunta de "por qu?" e incluso puede haber respuestas completamente distintas, pero todas vlidas, cada una en relacin con un contexto. Por ejemplo, en un accidente automovilstico muri Juan X. Se hace una investigacin judicial y se solicita la opinin de distintos expertos para "comprender" cmo o por qu muri Juan X. Las respuestas variarn dependiendo de quien las responda, as el mdico podr responder "por un paro cardaco", el mecnico "porque los frenos no estaban en buenas condiciones y fallaron", y un urbanista responder "porque los

rboles tapaban el semforo". Todas estas respuestas son legtimas segn van Fraassen, pero eso no quiere decir que cualquier respuesta en cualquier contexto sea legtima. Con lo cual, esto significa slo que puede haber explicaciones vlidas alternativas. Van Fraassen indica que puedo explicar un hecho A a partir de un hecho B, o puedo hacer al revs, explicar B a partir de A. Y en esto parece contradecir lo que sostena Hempel respecto a que la explicacin era unidireccional, es decir, que se va de las leyes a las consecuencias y no al revs. Pero, en este caso, si uno acepta un determinado contexto, es perfectamente legtima una explicacin y no lo es la otra.

Una de las consecuencias que extrae van Fraassen es que es completamente equivocado pensar que las explicaciones constituyen uno de los objetivos esenciales de las ciencias, ya que una explicacin no puede ser juzgada como correcta o no sobre la base de las caractersticas que Hempel haba tomado en cuenta, relativizando de esta manera el concepto de explicacin.

A modo de conclusin
Finalmente podemos decir que el punto crtico se encuentra entonces en los conceptos de mtodo y objetividad propios de la ciencia moderna. Actualmente, desde la epistemologa

comprensivista podemos destacar los aportes de: a) Alfred Schtz (socilogo y filsofo austraco, 1899-1959), para quien la comprensin es la reconstruccin de las perspectivas que los individuos tienen de la sociedad. La accin social es entendida como una vivencia guiada por un plan o proyecto. La interpretacin surge entonces de los motivos del actor (Schtz, 1932), pero el lmite es la imposibilidad de generalizar y predecir. b) Hans-Georg Gadamer aplica el mtodo al anlisis de textos histricos, y sostiene la naturaleza

histrica de la comprensin misma. Sostiene que hay una tensin entre la objetividad del texto y la subjetividad del intrprete, y la forma de resolver esta tensin es mediante la fusin de horizontes,
[27]

es decir, lograr un acuerdo de ajuste de nuestros prejuicios (Gadamer, 1981). Segn Gadamer, la etimologa del trmino "hermenutica" significa explicacin, y se trata de explicar unos enunciados analizndolos mediante otros enunciados. Es, pues, el arte o la teora de la interpretacin (Gadamer, 1998).

Para el filsofo alemn, "[...] la hermenutica no constituye un mtodo determinado que pudiera caracterizar por ejemplo a un grupo de disciplinas cientficas frente a las ciencias naturales. La hermenutica se refiere ms bien a todo el mbito de comunicacin infrahumana [...]" (Gadamer, 1998. 85). c) Jrgen Habermas, por su parte, destaca la capacidad de persuasin como estrategia para la dominacin a partir de la comprensin de las significaciones dadas en la accin comunicativa. La comprensin es entendida como experiencia

comunicativa que implica una actitud reflexiva y la posibilidad no slo de intercambios de mensajes, sino tambin para cambiar y formar actitudes. As, la hermenutica se vincula con la retrica, con la argumentacin de tipo informal. En el mismo sentido, Gadamer sostiene que "el gran legado de la retrica sigue influyendo en puntos decisivos en relacin con la nueva labor de interpretacin de los textos". De manera que retrica y hermenutica son profundamente afines, y particularmente, por ejemplo, las clases de retrica de Melanchthon
[28]

(telogo alemn, 14971560) ejercieron una influencia determinante en la configuracin del sistema escolar (Gadamer, 1998. 271-272). d) Paul Ricoeur, en Del texto a la accin (1969), tambin problematizar el intento de resolver la apora central de la hermenutica, es decir, la alternativa entre explicacin y comprensin, apora que a su juicio es infundada. l buscar la complementariedad de las dos actitudes, dado que la objetivacin del discurso en la obra y el

carcter estructural de la composicin, a lo cual se agregar el distanciamiento mediante la escritura, lo obliga a poner en tela de juicio la oposicin recibida de Dilthey entre comprender y explicar. Ahora bien, sera ciertamente ilusorio, sostiene von Wright, creer que la verdad reside inequvocamente en una de estas dos posiciones opuestas (explicacionista-comprensivista), sino que existe una confrontacin de base, y sta se funda en la eleccin de conceptos primitivos bsicos para la argumentacin en conjunto. Podra calificarse esta eleccin de "existencial" consistiendo en la opcin por un punto

de vista no susceptible de ulterior fundamento (von Wright, 1979. 57). Sin embargo, se puede considerar desde Dilthey, que las ciencias del espritu y, por ende, el mtodo de comprensin, son epistemolgicamente anteriores a las de la naturaleza, a las que por lo dems abarcan, pues toda ciencia natural es tambin un producto histrico. Pero como ya hemos dicho anteriormente, la cuestin an no est zanjada, y sigue en pie precisamente por ser un problema filosfico. Bibliografa Asti Vera, C. y Ambrosini, C. (2009). Argumentos y teoras. Aproximacin a la Epistemologa. Buenos Aires: Educando.

Braithwaite, R. B. (1965). La explicacin cientfica. Madrid: Tecnos. Comesaa, J. M. (1998). Lgica informal, falacias y argumentos filosficos. Buenos Aires: Eudeba. Dilthey, W. F. (2003). La esencia de la filosofa. Buenos Aires: Losada. Dray, W. (1957) Ley y Explicacin en Historia. Londres: Oxford University Press. Echeverra, R. (1993). El Bho de Minerva. Santiago de Chile: Dolen Ediciones. Fernndez, C. (1967). Dilthey. Buenos Aires: Centro Editor de Amrica Latina. Gadamer, H. G. (1981). Hermenutica como filosofa prctica. En La razn en

la poca de la ciencia. Madrid: Alfa. Gadamer, H. G. (1998). El giro hermenutico. Madrid: Ctedra. Gadamer, H. G. (1992-3). Verdad y Mtodo I y II. Salamanca: Sgueme. Giddens, A. y Turner, J. (1995). La teora social hoy. Buenos Aires: Alianza. Hempel, C. G. (1981). La explicacin en la ciencia y en la historia. En Teora de la historia. Mxico: Terra Nova. Hempel, C. G. (1983). Filosofa de la ciencia natural. Madrid: Alianza. Ricoeur, P. (2003). Del texto a la accin. Ensayos de hermenutica ii. Buenos Aires: Fondo de Cultura Econmica. Schtz, A. (1973). El problema de la

realidad social. Buenos Aires: Amorrortu. Windelband, W. (1943). Historia general de la filosofa. Mxico: Ateneo. Wrigth, G. H. von (1979). Explicacin y comprensin. Madrid: Alianza.

Temas de la Unidad 6.1. La Epistemologa del siglo XX: problemas fundamentales 6.2. La concepcin epistemolgica del Positivismo lgico 6.3. Las perspectivas falsacionistas: Popper y Lakatos 6.4. La ruptura: del racionalismo de justificacin al anlisis de la historia de la ciencia 6.5. El pensamiento de Kuhn en perspectiva Bibliografa obligatoria Unidad 6 en la Gua de Estudio de IPC,

producida por UBA XXI y editada por Eudeba, a partir de la edicin de 2010. Asti Vera, C. y Ambrosini, C. (2009). Captulo 7. En Argumentos y teoras. Aproximacin a la Epistemologa. Buenos Aires: Educando. BIBLIOGRAFA COMPLEMENTARIA Echeverra, J. (1999). Captulos 1, 2, 3, 4, 5 y 6. En Introduccin a la metodologa de la ciencia. La filosofa de la Ciencia en el siglo XX. Madrid: Ctedra. Klimovsky, G. (1994). Captulos 21, 22 y 23. En Las desventuras del conocimiento cientfico. Buenos Aires: AZ. presentacin de la Unidad

Creo que podran ustedes hacer algo ms til para matar el tiempo que malgastarlo con adivinanzas que no tienen solucin". "Ay! Si conocieras al Tiempo tan bien como lo conozco yo", exclam el Sombrerero, no hablaras de malgastarlo y menos de matarlo! Se trata de un tipo de mucho cuidado y no de una cosa cualquiera." "Me parece que sigo sin comprenderle", dijo Alicia. "Naturalmente que no me comprendes!", dijo el Sombrerero

elevando orgullosamente la nariz. "Con toda seguridad; ni siquiera habrs hablado con el Tiempo!" "Puede que no", contest Alicia con cautela. "Pero s s", aadi esperanzada, "que en las lecciones de msica marco el tiempo a palmadas". "Ah!Ah!Eso lo explica todo!", afirm el Sombrerero. "El Tiempo no tolera que le den de palmadas [...]" (Carroll, L. Alicia en el pas de las maravillas)

Esta Unidad 6 corresponde a la lectura del captulo 7 de la bibliografa y supone un enfoque metaterico ya que aqu se sealan algunos de los aspectos centrales de la Epistemologa

contempornea, y reitera algunos de los problemas abordados en las unidades anteriores desde un ngulo diferente. La expresin "la concepcin heredada", mencionada en este captulo, para referirse bsicamente a la epistemologa del Crculo de Viena, se adjudica al filsofo estadounidense Hilary Putnam (n. 1926) y se identifica con las crticas a esta epistemologa caratulada tambin como "cientificista". Encontrar en el captulo, una mencin a los aportes del psiclogo suizo Jean Piaget (1896-1980) identificando su posicin con la llamada Epistemologa gentica y algunos representantes de la filosofa francesa que acentan el importante papel de la historia de la

ciencia en el anlisis epistemolgico. Para hacer explcitos los postulados filosficos del llamado Positivismo lgico, se centra la atencin en los postulados del Crculo de Viena y la extendida influencia que ejercieron sus miembros sobre otros grupos de cientficos y epistemlogos de la poca. Un punto central dentro de los temas tratados, y sobre el que ya se presentaron varios aspectos, es la posicin falsacionista a travs de las figuras de Popper y de Imre Lakatos, tomando en cuenta sus ideas acerca de la produccin de teoras cientficas aludiendo a la posicin del primero como "realismo crtico" y, al segundo, a partir de su propuesta de los "programas

de investigacin". Deber detenerse especialmente en estos prrafos, en la presentacin de las ideas fundamentales de la epistemologa de Thomas Kuhn que pueden ser vistas como una ruptura respecto a las posiciones anteriores, enfocadas en la bsqueda del mtodo cientfico (monismo metodolgico) y en el anlisis de la estructura interna de las teoras cientficas para interesarse por la historia de la ciencia y el proceso de gnesis de las teoras cientficas. Las nociones centrales de "paradigma", "ciencia normal", "revolucin cientfica", propias de esta posicin, muestran un quiebre y una transformacin de la concepcin de la epistemologa centrada en el anlisis

lgico y en las estructuras formales de las teoras. Luego de la presentacin de la epistemologa de Kuhn y el sealamiento de algunas de las crticas de las que ha sido objeto, se seala un panorama del complejo entramado de teoras en las que se mencionan distintas estrategias para el anlisis de las teoras cientficas. Debemos advertir que estos temas no estn incluidos como temas del programa y no son de lectura obligatoria. A pesar de ello, como en otros casos, recomendamos su lectura para tener una comprensin cabal de las posiciones presentadas.

Actividades de aprendizaje
ACTIVIDAD 1. LA CONCEPCIN
HEREDADA DE LA CIENCIA Uno de los puntos ms polmicos de la posicin de Popper, en especial respecto al monismo metodolgico y a la propuesta de considerar "cientfico" un mismo mtodo para todo tipo de ciencias, aun en las ciencias sociales, es el estricto criterio de demarcacin que propone entre "ciencia" y "pseudociencia" a partir de considerar el criterio falsacionista como el nico vlido para legitimar el conocimiento

cientfico. En sus anlisis de las teoras cientficas, haciendo lugar con esto a un debate todava vigente, Popper lleg a la conclusin de que dos teoras consagradas en el campo de las ciencias sociales (el Marxismo y el Psicoanlisis freudiano) eran suficientemente flexibles como para acomodarse a cualquier situacin histrica o de comportamiento humano, dando la falsa sensacin de que explicaban los hechos. Popper advierte que una teora que parece explicarlo todo en realidad no explica nada. Segn Popper, una teora con una capacidad explicativa genuina hace predicciones "arriesgadas", que excluyen la mayor parte de posibles resultados. El xito en la prediccin es impactante slo hasta

donde el fracaso sea una verdadera posibilidad. Popper compara esta situacin con la forma, completamente diferente, en que la comunidad cientfica plante la comprobacin de la Teora General de la Relatividad de Einstein. La teora predeca que la luz, al propagarse, debera sentir la accin de la gravedad como cualquier forma de energa y, en consecuencia, su trayectoria debera curvarse al pasar cerca de un cuerpo de masa elevada, como es el caso del Sol. En consecuencia, un observador que apuntase su telescopio a una estrella, la vera en posiciones distintas si lo rayos de luz pasaban cerca del sol que si ste no se encontraba en su camino. En 1919,

el astrnomo britnico Arthur Stanley Eddington (1882-1944) llev a cabo este experimento durante un eclipse y comprob que los resultados estaban de acuerdo con las predicciones de Einstein y, en consecuencia, la Teora General de la Relatividad fue aceptada. Lo verdaderamente importante, en trminos de Popper, no era el hecho de que la observacin de Eddington implicara la validez de la Teora General de la Relatividad, sino que la Teora General de la Relatividad era susceptible de ser comprobada. Exista un procedimiento bien establecido, de acuerdo con el cual poda demostrarse que una teora dada era falsa. Popper emple esta posibilidad de

"falibilidad", como caracterstica esencial que serva para separar las creencias, explicaciones y teoras en dos categoras: cientficas y pseudocientficas. Siguiendo estas ideas y profundizando en la polmica, el fsico y filsofo argentino Mario Bunge (n. 1919) extrema la tesis central del monismo metodolgico en el siguiente artculo y, en este caso, critica especialmente al Psicoanlisis al ponerlo a la altura de la magia o la supersticin. A. Lea el artculo de Bunge, a continuacin: Una pseudociencia es un montn de macanas que se vende como ciencia.

Ejemplos: alquimia, astrologa, caracterologa, comunismo cientfico, creacionismo cientfico, grafologa, ovniloga, parapsicologa y psicoanlisis. Una pseudociencia se reconoce por poseer al menos un par de las caractersticas siguientes: - Invoca entes inmateriales o sobrenaturales inaccesibles al examen emprico, tales como fuerza vital, alma, superego, creacin divina, destino, memoria colectiva y necesidad histrica. - Es crdula: no somete sus especulaciones a prueba alguna. Por ejemplo, no hay laboratorios homeopticos ni psicoanalticos. Correccin: en la Universidad Duke

existi en un tiempo el laboratorio parapsicolgico de J. B. Rhine; y en la de Pars existi el laboratorio homeoptico del doctor Benveniste. Pero ambos fueron clausurados cuando se descubri que haban cometido fraudes. - Es dogmtica: no cambia sus principios cuando fallan ni como resultado de nuevos hallazgos. No busca novedades, sino que queda atada a un cuerpo de creencias. Cuando cambia lo hace slo en detalles y como resultado de disensiones dentro de la grey. -Rechaza la crtica, matayuyos normal en la actividad cientfica, alegando que est motivada por

dogmatismo o por resistencia psicolgica. Recurre pues al argumento ad hominem en lugar del argumento honesto. - No encuentra ni utiliza leyes generales. Los cientficos, en cambio, buscan o usan leyes generales. - Sus principios son incompatibles con algunos de los principios ms seguros de la ciencia. Por ejemplo, la telequinesis contradice el principio de conservacin de la energa. Y el concepto de memoria colectiva contradice la perogrullada de que slo un cerebro individual puede recordar. - No interacta con ninguna ciencia propiamente dicha. En particular, ni

psicoanalistas ni parapsiclogos tienen tratos con la psicologa experimental o con la neurociencia. A primera vista, la astrologa es la excepcin, ya que emplea datos astronmicos para confeccionar horscopos. Pero toma sin dar nada a cambio. Las ciencias propiamente dichas forman un sistema de componentes interdependientes. - Es fcil: no requiere un largo aprendizaje. El motivo es que no se funda sobre un cuerpo de conocimientos autnticos. Por ejemplo, quien pretenda investigar los mecanismos neurales del olvido o del placer tendr que empezar por estudiar neurobiologa y psicologa,

dedicando varios aos a trabajos de laboratorio. En cambio, cualquiera puede recitar el dogma de que el olvido es efecto de la represin, o de que la bsqueda del placer obedece al principio del placer. Buscar conocimiento nuevo no es lo mismo que repetir o siquiera inventar frmulas huecas. - Slo le interesa lo que pueda tener uso prctico: no busca la verdad desinteresada. Ni admite ignorar algo: tiene explicaciones para todo. Pero sus procedimientos y recetas son ineficaces por no fundarse sobre conocimientos autnticos. Al igual que la magia, tiene aspiraciones tcnicas infundadas.

- Se mantiene al margen de la comunidad cientfica. Es decir, sus cultores no publican en revistas cientficas ni participan de seminarios ni de congresos abiertos a la comunidad cientfica. Los cientficos, en cambio, someten sus ideas a la crtica de sus pares: someten sus artculos a publicaciones cientficas y presentan sus resultados en seminarios, conferencias y congresos. Veamos en un ejemplo cmo obran los cientficos cuando abordan problemas que tambin interesan a los pseudocientficos. En 1998 los psicobilogos J. S. Morris, A. Ohman y R. J. Dolan publicaron en la clebre

revista Nature un trabajo sobre aprendizaje emocional consciente e inconsciente en la amgdala humana. Ya que este artculo trata de emociones conscientes e inconscientes, parecera que debiera interesar a los psicoanalistas. Pero no les interesa porque los autores estudiaron el cerebro, mientras que los analistas se ocupan del alma: no sabran qu hacer con cerebros, ajenos o propios, en un laboratorio de psicobiologa. Pues bien, la amgdala cerebral es un rgano diminuto pero evolutivamente muy antiguo, que siente emociones bsicas tales como el miedo y la furia. Dada la importancia de estas emociones en la

vida social, es fcil imaginar los trastornos de conducta que sufre una persona con una amgdala anormal, ya sea atrofiada o hipertrfica. Si lo primero, no reconocer signos peligrosos. Si lo segundo, ser propensa a la violencia. La actividad de la amgdala cerebral puede registrarse mediante un escner PET. Este aparato permite detectar objetivamente las emociones de un sujeto en cada lado de su amgdala. Sin embargo, tal actividad emocional puede no aflorar a la conciencia. O sea, una persona puede estar asustada o enojada sin advertirlo. Cmo se sabe? Agregando un test psicolgico a la observacin neurobiolgica. Por

ejemplo, si a un sujeto normal se le muestra brevemente una cara enojada e inmediatamente despus una cara sin expresin, informar que vio la segunda pero no la primera. Represin? Los cientficos citados no se contentaron con bautizar el fenmeno. Repitieron el experimento, pero ahora asociaron la cara enojada con un estmulo negativo: un intenso y molesto ruido blanco, es decir, no significativo. En este caso, la amgdala fue activada por la imagen visual, aun cuando el sujeto no recordara haberla visto. O sea que la amgdala cerebral sabe algo que ignora el rgano de la conciencia (cualquiera que ste sea).

En principio, con el mtodo que acabo de describir escuetamente se podra medir la intensidad de una emocin. Por ejemplo, se podra medir la intensidad del odio que, segn Freud, un varn siente por su padre. Sin embargo, antes de proceder a tal medicin habra que establecer la existencia del complejo de Edipo. Pero ste no existe, como lo mostraron las extensas investigaciones de campo del profesor Arthur P. Wolf condensadas en su grueso tomo Sexual Attraction and Childhood Association (Stanford University Press, 1995). Las pseudociencias son como las pesadillas: se desvanecen cuando se

las examina a la luz de la ciencia. Pero mientras tanto infectan la cultura y algunas de ellas son de gran provecho pecuniario para sus cultores. Por ejemplo, un psicoanalista latinoamericano puede ganar en un da lo que su compatriota cientfico gana en un mes. Lo que refuta el refrn: "no es oro todo lo que reluce".
[29]

B. En base a lo estudiado sobre el tema: a. Determine cules son los objetivos del Positivismo lgico, y por qu se denomina de esta manera a esta corriente de pensamiento. b. Mencione qu dos tipos de objetos

pueden ser nicamente objeto de investigacin y de conocimiento cientfico para el Positivismo lgico, y seale cules no seran objetos de conocimiento para ellos. c. Responda: Cmo es posible demarcar el campo entre la ciencia y la pseudociencia segn Popper? Qu mtodo utiliz l para separarlas? Cules son las caractersticas que tendra que cumplir una teora para que sea cientfica, segn el mismo autor? d. Explique brevemente cmo se decide cundo una teora es aceptable (evitamos el trmino "verdadera" porque justamente es lo que se discute en distintas posiciones) para los positivistas lgicos, para Popper y para

Kuhn. e. Responda: Qu supone que podra oponer a las afirmaciones de Bunge un partidario del pluralismo metodolgico? f. En los ltimos prrafos del artculo de Bunge se afirma: "Las pseudociencias son como las pesadillas: se desvanecen cuando se las examina a la luz de la ciencia. Pero mientras tanto infectan la cultura". Considera pertinente la extrapolacin de un concepto de la biologa, "infeccin", al anlisis de las sociedades? Por qu? ACTIVIDAD 2. CONCEPCIN EPISTEMOLGICA DEL POSITIVISMO LGICO. KUHN, TRES PENSADORES EN UNO A. Lea el siguiente artculo de Bunge:

Todos los universitarios han odo hablar de Thomas S. Kuhn (19221996). Parecera que no se puede pasar por culto sin citarlo. De hecho, Kuhn es el ms citado, aunque no necesariamente el ms ledo, de todos los autores no literarios. Hasta la fecha su libro ms conocido ha vendido ms de un milln de ejemplares en veinte lenguas. Sin embargo, pocos saben que Kuhn no fue uno sino trino, como dira un telogo cristiano. Y lo peor es que el ms influyente de los tres no es el que el propio Thomas hubiera querido ser, o sea, un historiador de la ciencia venerado por sus pares como lo fue,

por ejemplo, George Sarton en su tiempo. En efecto, el Kuhn popular es el de los paradigmas y desplazamientos de tales, o revoluciones cientficas. stas eran las ideas centrales (aunque oscuras) de su libro La estructura de las revoluciones cientficas, que en 1962 le otorg fama de la noche al da. Un ao despus lo vi ocupar el centro de la primera reunin de historiadores de la ciencia, en Filadelfia. En 1965, en Londres, volvi a atraer la atencin en el simposio dedicado a Popper, y ello por dos motivos. Uno de stos fue que Margaret Masterman, una filsofa desconocida, expuso una ponencia clara y combativa en la que

mostraba que Kuhn haba metido por lo menos dos docenas de conceptos distintos en la bolsa "paradigma". Entre ellos figuraban los de cosmovisin, modelo a imitar y programa de investigacin. Kuhn aprendi esta leccin. Unos aos despus, cuando vino a hablar a mi universidad sobre los orgenes de la teora cuntica, un asistente le pregunt algo sobre los paradigmas, y l lo par en seco: "Estoy harto de eso. Ahora estoy en otra cosa". Controversia con Karl Popper: Al terminar su conferencia le pregunt cul sera su prximo proyecto y me contest que pensaba estudiar la tesis de Mary Hesse, de que las teoras

cientficas son modelos visualizables, como el modelo atmico de Rutherford-Bohr. Tom no tena idea de que las teoras son sistemas de hiptesis, ni de que la teora cuntica moderna no alienta los modelos visuales, porque se ocupa de cosas que carecen de forma propia. El otro motivo por el cual Kuhn descoll en aquel memorable simposio de 1965 fue la resonante controversia que sostuvo con Karl Popper. El contraste entre ambos era fsico, psicolgico y filosfico. Kuhn era un gigantn, hablaba fuerte y fumaba un enorme habano. En cambio, Popper era menudo, hablaba bajito y odiaba el tabaco. La incompatibilidad

filosfica entre ambos no era menos obvia, pese a que Karl intent minimizarla. Mientras Popper era racionalista, Kuhn sostena la tesis irracionalista de que los cambios de teora son tan irracionales como las conversiones religiosas. Sin embargo, paradjicamente, ambos concordaban en que no hace falta justificar la adopcin de una teora; en particular, los datos favorables no seran importantes. Pero volvamos a mi tesis. Mi tesis es que hubo tres Thomas S. Kuhn en una misma persona: el historiador, el filsofo y el socilogo de la ciencia. El primero fue ignorado o fuertemente criticado por sus colegas y no form escuela.

El segundo logr la popularidad que sabemos. Y el tercero, aunque igualmente popular, slo existi en la imaginacin de ciertos socilogos de la ciencia: que lo consideran, junto con su amigo Paul K. Feyerabend, como el cofundador o al menos padrino de la nueva escuela en ese campo. Esta escuela niega la existencia de verdades objetivas y afirma que las ideas, e incluso los hechos, son construcciones o convenciones de grupos o comunidades de investigadores. Se llaman a s mismos constructivistas (por oposicin a realistas) y relativistas (por negar la existencia de verdades universales,

independientes de las circunstancias sociales). Lo curioso es que, aunque Kuhn sostuviera que la sociedad cambia de teoras cientficas como de modas sartoriales, sus trabajos histricos son tan internalistas como los tradicionales. O sea, no practic como profesional lo que predic en su libro ms popular. No menos curioso es que este libro fuera publicado originariamente como el ltimo fascculo de la Encyclopedia of Unified Science, de orientacin positivista. Esto es curioso porque Kuhn era netamente antipositivista. En efecto, no conceda mayor valor a los datos empricos y crea ms en la analoga que en la induccin

(generalizacin a partir de datos empricos). Pero volvamos al constructivismo-relativismo. Contradicciones reveladoras: Hace unos aos, un periodista de Scientific American entrevist a Kuhn y le pregunt si crea que, cada vez que cambia la cosmovisin dominante, tambin cambia el propio mundo. "Por supuesto!", contest Tom con su vozarrn. Segunda pregunta: "Cree que el mundo que lo rodea existe independientemente de usted?" Respuesta: "Por supuesto!". Esta contradiccin muestra a las claras la ingenuidad filosfica de Tom. Hacia el final de su vida, particularmente en una conferencia que pronunci en

Harvard en 1991, Kuhn se distanci explcitamente de los constructivistas, que niegan la existencia autnoma del mundo. Aunque sigui admitiendo (como toda persona razonable) que la poltica desempea un rol en la vida cientfica, neg que ste fuese el principal. Desgraciadamente, Tom no dijo cules son las motivaciones de los investigadores bsicos. El gran Robert K. Merton lo dijo y con razn: son la curiosidad y el deseo de ganar prestigio. Quienes buscan poder se dedican a los negocios o a la poltica. Consejo a los admiradores del triple Kuhn: Decdanse a cul de ellos venerar, porque no slo son diferentes, sino que no armonizan

entre s. A menos, claro est, que estn dispuestos a reconocer que tampoco esta trinidad es inteligible. Mario Bunge (2000, diciembre 4). La Nacin. B. Segn lo estudiado sobre el tema: a. Caracterice esquemticamente el concepto de "progreso" en la ciencia segn el Positivismo lgico, Popper, Kuhn y Lakatos. b. Indique, de acuerdo con la lectura de los dos artculos de Bunge, a qu corriente epistemolgica cree usted que pertenece l? Justifique. c. Determine a qu epistemlogo realiza crticas, y cules son.

d. En relacin con la siguiente expresin de dicho artculo, responda las preguntas a continuacin:
[... ] pocos saben que Kuhn no fue uno sino trino, como dira un telogo cristiano. Y lo peor es que el ms influyente de los tres no es el que el propio Thomas hubiera querido ser, o sea, un historiador de la ciencia venerado por sus pares [...]. En efecto, el Kuhn popular es el de los paradigmas y desplazamientos de tales, o revoluciones cientficas. Estas eran las ideas centrales (aunque oscuras) de su libro La estructura de las revoluciones cientficas, que en 1962 le otorg fama de la noche al da [...]

Explique por qu afirma Bunge que Kuhn no fue uno sino trino? d.1. Por qu usa la frase de un telogo? d.2. Cul sera la relacin que quiere mostrar Bunge entre Kuhn y la religin? Con qu objetivo intenta mostrar esa

relacin? Segn lo estudiado en la unidad, elabore una crtica breve a lo planteado por Bunge. C. La falacia del espantapjaros o del hombre de paja Esta falacia est citada en el libro de la bibliografa, se refiere al recurso que consiste en "inventar" un oponente ms fcil de destruir que el oponente real. El nombre viene del uso de muecos de paja para entrenarse los soldados antes de un combate. Es bastante comn encontrar este tipo de recursos en las polmicas entre partidarios de posiciones rivales. Considera que respecto a Freud, a Marx a Kuhn, Bunge est construyendo un espantapjaros? Explique su

posicin. Actividad 3. Thomas Kuhn y la visin del ojo del conejo A. Lea el siguiente prrafo de Thomas Kuhn en La estructura de las revoluciones cientficas:[30]

Desde la atalaya de la historiografa contempornea, el historiador de la ciencia puede sentirse tentado a proclamar que cuando cambian los paradigmas, el mundo mismo cambia con ellos. Guiados por un nuevo paradigma, los cientficos

adoptan nuevos instrumentos y buscan en lugares nuevos. Lo que es todava ms importante, los cientficos ven cosas nuevas y diferentes al mirar con instrumentos conocidos y en lugares en los que ya haban buscado antes [...] los cambios de paradigmas hacen que los cientficos vean el mundo de investigacin que les es propio, de manera diferente [...] Lo que antes de la revolucin eran patos en el mundo del cientfico, se convierten en conejos despus.

En este prrafo Kuhn alude a la figura pato-conejo para aludir a la visin paradigmtica. Lo que desde un paradigma se ve como "pato" en el otro puede verse como "conejo". Dicho de otro modo, viendo lo mismo, ven cosas distintas. Podemos decir, entonces, que respecto a teoras rivales como pueden haber sido el geocentrismo (Tolomeo) o el heliocentrismo (Coprnico), algunos

vieron "patos" (la Tierra est en el centro del Universo) donde otros vieron "conejos" despus (el Sol est en el centro de nuestro sistema planetario). Hasta el siglo XVII, el paradigma o marco cientfico era el de Aristteles, completado por la Astronoma de Tolomeo, que presentaba el Universo dividido en dos partes de naturaleza diferente, con la Tierra en el centro: la imperfecta, corruptible y cambiante que llegaba hasta la esfera de la Luna; y la perfecta, inmutable e incorruptible, sin movimiento alguno, que se extenda ms all de la esfera lunar. Pero fueron las interpretaciones que Galileo hizo, elaboradas por Kepler despus, y de las suyas propias empleando el telescopio,

las que destruyeron ese paradigma imponiendo la nueva visin que se ha llamado "copernicana". Un caso parecido de cambio revolucionario es el introducido por la Teora de la evolucin de Charles Darwin respecto a las teoras creacionistas anteriores, en el campo de la Biologa. B. En base a lo estudiado sobre la posicin de Kuhn: a. Explique cul es el papel que cumple la historia de la ciencia en la epistemologa de este pensador, y en qu se diferencia de las epistemologas anteriores. b. Identifique en las dos modalidades de produccin cientfica de Kuhn (modo normal y modo extraordinario, y en cada

uno de sus elementos), el cambio de la concepcin "creacionista" a la "evolucionista", y/o de la "geocntrica" a la "heliocntrica". c. Intente definir mediante sinnimos el concepto de "paradigma" segn este autor. d. Explique brevemente por qu podra decirse que Kuhn afirma que dos observadores que contemplan los mismos hechos desde supuestos diferentes ven cosas diferentes, ven "mundos" diferentes. e. En virtud de la consigna anterior, explique qu significa que las teoras son "inconmensurables". MATERIALES DE LECTURA I

PERSPECTIVA FALSACIONISTA.

LA SUPERVIVENCIA DEL MS APTO. VIDA Y MUERTE DE UNA


HIPTESIS Segn el epistemlogo Gregorio Klimovsky, la concepcin hipottica de la ciencia supone admitir lisa y llanamente que los enunciados cientficos son aceptados a ttulo de hiptesis, de conjeturas, y no de enunciados justificados ya que no ofrecen garantas para su veificacin. Esta conclusin puede parecer pesimista pero tambin es posible concebirla como una honesta y modesta posicin al admitirse que en las ciencias fcticas algunas de las teoras ms consagradas han terminado siendo reemplazadas por otras consideradas ms eficaces o

abarcativas. Supongamos que frente a un problema tenemos una hiptesis que trata de resolverlo pero all no podemos dar por terminada la investigacin puesto que el conocimiento cientfico no concluye en la mera formulacin de hiptesis. Proponer hiptesis no es sinnimo de obtener conocimiento. Cmo sigue entonces la investigacin? Parece razonable preguntarnos qu se deduce de esta hiptesis? Esto nos lleva a la formulacin de otras hiptesis y a enunciados observacionales que nos permitan controlar la hiptesis por medio de la experiencia y puede ocurrir que la hiptesis salga exitosa de la contrastacin emprica. Segn Popper, y es difcil no coincidir con l (afirma

Klimovsky), la operacin de contrastacin exitosa no aumenta en ningn sentido (ni absoluto ni probabilstico) nuestro conocimiento acerca de la verdad de la hiptesis. Si la hiptesis ha resistido todos los intentos de falsacin, se puede admitir que es "fuerte" en el sentido de que ha mostrado su capacidad de supervivencia. Qu pasa cuando hay otras hiptesis rivales que compiten como teoras rivales? En general, no hay ms remedio que investigar cada hiptesis por separado y confiar en que alguna de las hiptesis resulte refutada y otra ha mostrado ser lo suficientemente "fuerte" para sobrevivir. Esto lleva a algunos autores a sealar que el proceso

hipottico-deductivo opera metafricamente de un modo similar a la supervivencia darwiniana en la que se admite la supervivencia del ms apto. Dice Klimovsky: "En esta concepcin del mtodo, la vida de una hiptesis (o de una teora) es dura, trgica, y para cada una de ellas podra escribirse acerca de su nacimiento, pasin y muerte. Ante el problema que intenta resolver, la hiptesis nace, pero luego empieza el terrible proceso de contrastacin por medio del cual se la intenta aniquilar. Ella resiste, pero constantemente es agraviada por nuevos episodios de contrastacin hasta que, finalmente, la refutacin termina con ella en un dramtico

episodio de muerte epistemolgica." (Klimovsky, 2005. 140) MATERIALES DE LECTURA II

PERSPECTIVA FALSACIONISTA
George Soros, millonario falsacionista El financiero y billonario George Soros, hngaro de nacimiento, antiguo alumno de Popper, se sinti tan inspirado por su maestro que bautiz su Fundacin Sociedad Abierta en su honor. Soros gan millones en la bolsa invirtiendo segn las lneas establecidas por Popper. Popper pensaba que deba considerarse que las teoras cientficas ms slidas eran aquellas que estaban dispuestas

a someterse a las pruebas ms severas, sobreviviendo a ellas. Soros aplic este principio a Wall Street. De este modo hizo una fortuna al invertir en bonos de seguros hipotecarios de una compaa de California que haba sido sometida a severas crticas por una crisis inmobiliaria; pens que el hecho de haber sobrevivido era una prueba decisiva de que la compaa era, en el fondo, solvente. La Fundacin Sociedad Abierta era la teora poltica de Popper puesta en prctica, poniendo a prueba el poder transformador de la apertura.
[31]

El xito econmico de George Soros podra ser visto como un resultado exitoso de la adopcin de las ideas de Popper en el campo de los negocios. Como el mismo Popper admite, la contrastacin exitosa no agrega conocimiento acerca de la verdad de la teora, el xito de Soros en los negocios podra deberse a otros factores o variables. ACTIVIDAD 4. LAS PERSPECTIVAS FALSACIONISTAS. LAKATOS Imre Lakatos critica el tipo de falsacionismo que l denomina "ingenuo", sin embargo, su propsito no es atacarlo sino recoger y profundizar la heurstica positiva de aqul, generando una nueva versin del falsacionismo que

denomina "sofisticado", hacindose cargo -en su reconstruccin- de las "refutaciones historiogrficas" que Kuhn opuso al programa de investigacin falsacionista. En el texto de la bibliografa se presenta un caso hipottico de investigacin cientfica donde se veran cuestiones centrales de un programa de investigacin como el que propone Lakatos. En vista a este ejemplo: a. Determine qu constituye el ncleo duro de ese programa. b. Identifique algunas de las hiptesis auxiliares que constituyen el cinturn protector. c. Explique por qu el cientfico incorpora tales hiptesis a medida que

se presentan los fracasos experimentales. ACTIVIDAD 5. ACERCA DE DISTINTAS POSICIONES EPISTEMOLGICAS Determine si los siguientes enunciados, son verdaderos o falsos. Justifique en cada caso. a. "Segn Thomas Kuhn, la verdad constituye una meta inalcanzable a la que nos acercamos en un proceso de creacin de teoras tentativas y eliminacin de errores." b. "Imre Lakatos es un epistemlogo identificado con el Positivismo lgico." c. "Para Karl Popper, en el desarrollo de una ciencia hay que distinguir entre dos perodos: uno de desarrollo normal

y otro de ruptura no acumulativa." d. "Un rasgo comn entre los inductivistas y los falsacionistas es el esfuerzo por encontrar un criterio universal y ahistrico para juzgar los mritos entre teoras rivales." e. "El epistemlogo Rudolf Carnap se identifica con el falsacionismo sofisticado." f. "Para Thomas Kuhn, el cambio de paradigma implica progreso en el sentido de mejoramiento de la teora." g. "Segn Karl Popper, la observacin precede a la teora y ofrece una base concluyente para obtener conocimiento cientfico." h. "En la epistemologa de Thomas Kuhn, los cientficos normales se ocupan

de falsar la teora principal o paradigma." i. "El epistemlogo Thomas Kuhn fue miembro del Crculo de Viena".

Actividad de integracin de las Unidades 4, 5 y 6


En esta actividad presentamos un caso muy analizado en el campo de la Epistemologa y revisaremos contenidos de las Unidades 4 y 5. Lea el siguiente artculo y, luego, responda las preguntas:

Ciencias Fcticas. Invencin y contrastacin de teoras en la investigacin cientfica y modelos de explicacin Un caso de investigacin y elaboracin de teoras cientficas clsico de la espistemologa de las ciencias mdicas es el de Semmelweis en relacin con la fiebre puerperal. Presentemos el caso: Entre 1844 y 1848 en el Hospital General de Viena un mdico hngaro, Ignaz Semmelweis, realiz una serie de trabajos vinculados a las mujeres en estado de postparto. En ese Hospital haba dos Divisiones de

Maternidad. Semmelweis era mdico de la Primera Divisin de Maternidad, y en ella un hecho estaba causando conmocin: una gran proporcin de mujeres que haban dado a luz, contraan una seria enfermedad, la cual con frecuencia era mortal. Esta enfermedad era conocida con el nombre de Fiebre Puerperal o Fiebre Postparto. En 1844, en la Divisin Primera de Maternidad del Hospital General de Viena, sobre un total de 3.157 mujeres en estado de postparto, murieron de esa enfermedad 260, un 8,2%. En 1845 el ndice de muertes en esa Divisin y por esa enfermedad descendi al 6,8%, pero en 1846

ascendi al 11,4%. La preocupacin mayor estaba dada porque en la Divisin Segunda de ese Hospital, en los mismos aos los ndices fueron del 2,3; 2,0 y 2,7 respectivamente. Es decir, mientras que en la Primera Divisin de Maternidad los nmeros de muertes por esta enfermedad eran significativamente altos, en la Divisin Segunda, en cambio, eran significativamente bajos en comparacin. Semmelweis, posteriormente a esta experiencia, da cuenta en un libro de las causas de la fiebre puerperal y las prevenciones que deben tomarse para evitarla, relatando sus esfuerzos para resolver este rompecabezas. Hasta

ese momento, cuando se pensaba en las posibles causas de la fiebre puerperal, una opinin ampliamente aceptada sostena que se deba a "influencias epidmicas" las cuales se describan como "cambios atmosfrico-csmico-telricos" que se expandan por ciudades y pueblos enteros produciendo la fiebre puerperal en mujeres en estado de posparto. Respecto de esta posible causa, Semmelweis sin embargo se preguntaba: cmo era posible que una epidemia semejante pueda haber infestado durante aos la Divisin Primera y haber eludido a la Divisin Segunda? Y cmo era posible que mientras esta epidemia acosara al

Hospital, casi no se registrasen otros casos en la ciudad de Viena y sus alrededores? Porque una epidemia de verdad, como el clera por ejemplo, no sera tan selectiva. Semmelweis en principio indica que una causa posible podra darse porque algunas de las mujeres internadas en la Divisin Primera que vivan lejos del hospital, sufran repentinamente de dolores de parto y daban a luz en la calle. Sin embargo, el porcentaje por muerte de fiebre puerperal en "partos callejeros" era menor que en la Divisin Primera. Otra opinin atribua la causa de mortalidad en la Divisin Primera al hacinamiento; sin embargo,

Semmelweis indicaba que el hacinamiento era mucho mayor en la Divisin Segunda. Luego Semmelweis descart otras conjeturas similares. En 1846 se design una comisin especial para investigar este problema. Esta comisin atribuy la frecuencia de esta enfermedad en la Divisin Primera a las lesiones que se producan en las pacientes por revisiones poco cuidadosas efectuadas por los estudiantes de medicina, todos los cuales realizaban sus prcticas de obstetricia en esta Divisin. Con el objetivo de refutar esta ltima opinin, Semmelweis seala que: a) las lesiones que se

producen naturalmente en un parto son mucho mayores que las que podra haber producido un examen poco cuidadoso de la paciente; b) las parteras que reciban enseanzas en la Divisin Segunda hacan los reconocimientos de sus pacientes de una manera muy similar a los estudiantes de obstetricia, sin embargo, no se producan los mismos efectos; c) cuando, a partir de un informe elevado por esta comisin, se redujo a la mitad el nmero de estudiantes y se limit los reconocimientos hechos a estas pacientes por parte de ellos, la mortalidad, luego de un breve descenso, lleg a sus ndices ms

altos. Tambin se acudi a varias explicaciones psicolgicas. Una de stas indicaba que la Divisin Primera estaba organizada de tal manera que, cuando el sacerdote deba dar la extremauncin a una moribunda, antes de llegar a la enfermera deba pasar por cinco salas. Se sostena que la presencia del sacerdote quien pasaba precedido por un aclito que haca sonar una campanilla, causaba un terror tal en las mujeres de las salas por las que pasaba que las debilitaba y las haca ms propicias a contraer la enfermedad. En cambio, en la Divisin Segunda esto no suceda

porque el sacerdote tena entrada directa a la enfermera. Semmelweis decidi probar esta suposicin. Le solicit al sacerdote que suprimiera el toque de la campanilla y evitara pasar por las salas precedentes a la enfermera para que llegara a la habitacin de la enferma terminal en silencio y sin ser visto. A pesar de todo esto, la mortalidad en la Divisin Primera no descendi. A Semmelweis se le ocurri una nueva idea: en la Divisin Primera, las mujeres estaban acostadas de espaldas, mientras que en la Segunda de costado. Decidi probar si la diferencia de posicin resultaba significativa. Sin embargo, la

mortalidad continu. En 1847 finalmente por casualidad Semmelweis dio con la clave para la solucin del problema. Un colega suyo, Kolletschka, sufri una herida cortante en el dedo con un escalpelo que estaba usando un estudiante suyo en una autopsia. Luego de ese hecho, Kolletschka muri presentando los mismos sntomas que las vctimas de fiebre puerperal. Por ese entonces an no se haba descubierto el papel que cumplan los microorganismos en ese tipo de infecciones. Semmelweis por su parte comprendi que la materia cadavrica que el escalpelo del estudiante haba introducido en la corriente sangunea de Kolletschka

era la causa de la enfermedad y muerte de su colega. Las semejanzas entre las dolencias de Kolletschka y las mujeres en estado de postparto le permitieron a Semmelweis llegar a la conclusin de que sus pacientes haban muerto por un envenenamiento de la sangre del mismo tipo. l, sus colegas y los estudiantes de medicina haban sido los portadores de la materia infecciosa, ya que todos ellos solan llegar a las salas de parto para realizar el reconocimiento de las parturientas inmediatamente despus de realizar disecciones de cadveres en la sala de autopsias y slo habindose lavado las manos de un modo muy superficial, de tal manera

que incluso stas conservaban a menudo un caracterstico olor a suciedad. Nuevamente, Semmelweis puso a prueba esta posibilidad. Si la suposicin era correcta, entonces se podra prevenir la fiebre puerperal destruyendo con medios qumicos el material infeccioso que portaban los mdicos y estudiantes en sus manos por el contacto con la materia cadavrica. Para probar esto dict una orden que exiga a todos los estudiantes de medicina que se lavaran las manos con una solucin de cal clorurada antes de realizar cualquier reconocimiento a una enferma. Luego de esto, la mortalidad

por fiebre puerperal comenz a decrecer, y en el ao 1848 descendi hasta un 1,27% en la Divisin Primera frente a un 1,33% en la Divisin Segunda. Esta hiptesis es apoyada adems por el hecho de que en la Divisin Segunda la mortalidad fuera ms baja, ya que all las parturientas eran atendidas por parteras quienes no tenan entre sus prcticas realizar disecciones de cadveres. Asimismo esta hiptesis explicaba tambin el caso de que en los "partos callejeros" la mortalidad fuera tambin menor respecto a la Divisin Primera, ya que al ingresar en el hospital habiendo ya dado a luz, casi nunca se

haca un reconocimiento de la paciente evitando de este modo la posibilidad de infectarse. Tambin esta hiptesis daba cuenta del hecho de que todos los recin nacidos que contrajeron la fiebre puerperal lo haban hecho porque sus madres haban contrado la enfermedad durante el parto; porque en ese caso la infeccin se le poda transmitir al nio antes de su nacimiento a travs de la corriente sangunea comn de madre a hijo, mientras que resultaba imposible esto cuando la madre estaba sana. Posteriormente, a partir de una serie de experiencias realizadas, Semmelweis pudo incluso ampliar su

hiptesis: en una oportunidad, l y sus colaboradores, despus de haberse desinfectado cuidadosamente las manos, examinaron primero a una mujer en estado de parto que sufra de cncer cervical ulcerado y, a continuacin, y slo habindose lavado superficialmente las manos, examinaron a otras doce mujeres de la misma sala. Once de las doce pacientes murieron de fiebre puerperal. De esta manera Semmelweis lleg a la conclusin de que la fiebre puerperal poda ser producida no slo por materia cadavrica, sino tambin por materia en estado de putrefaccin proveniente de organismos vivos.

[32]

a. Cul es el problema? b. Cules son las seis hiptesis que han sido descartadas? c. Reconstruya cules son los argumentos y contraargumentos de esas hiptesis. d. Cules son sus consecuencias observacionales? e. Cul es la primera hiptesis confirmada y qu argumentos la apoyaron? f. Es concluyente la verdad de esa hiptesis? Justifique. g. Cul es la segunda hiptesis confirmada que extrae Semmelweis? Qu argumentos la apoyaron? h. Utiliz algn mtodo para la

elaboracin/descubrimiento de las hiptesis? i. Qu mtodo utiliz para la justificacin de las hiptesis tanto de las refutadas como de la aceptada? j. Cmo se valora el caso confirmatorio de una hiptesis en una posicin refutacionista como la de Popper o en una posicin confirmacionista como la de Carnap y la de Hempel? k. Qu es posible explicar mediante la primera hiptesis confirmada?, y mediante la segunda hiptesis confirmada? Construya el esquema de explicacin explanans/explanandum determinando si se trata de una explicacin nomolgico-deductiva o estadstico-inductiva.

l. Podra predecir otros hechos con esta teora? Por qu?

Temas de la Unidad 7.1. Poltica cientfica 7.1.1 Caractersticas generales 7.1.2. La planificacin de la poltica cientfica. Debates contemporneos 7.1.3. La formacin del investigador 7.1.4. Etapas de la poltica cientfica argentina 7.1.4. Institutos de investigacin y Universidad 7.2. Ciencia, tecnologa y sociedad 7.2.1. Ciencia, tecnologa y tica: la responsabilidad social del cientfico y el tecnlogo 7.2.2. Ciencia y tica: debates en torno a la neutralidad valorativa de la ciencia

7.2.3. La tica de la investigacin cientfica 7.2.4. Ciencia bsica, ciencia aplicada y tecnologa. Cientificismo y anticientificismo Bibliografa obligatoria Unidad 7 en la Gua de Estudio de IPC, producida por UBA XXI y editada por Eudeba, a partir de la edicin de 2010. Contratti, M. B. (2010). Documento de Ctedra: Poltica cientfica: problemas y perspectivas. En esta Gua de Estudio de IPC, Buenos Aires: Eudeba; y en el Campus virtual de UBA XXI. Contratti, M. B. (2010). Documento de Ctedra: tica y ciencia. En esta Gua de Estudio de ipc, Buenos

Aires: Eudeba; y en el Campus virtual de UBA XXI. Presentacin de la Unidad En esta Unidad 7 enfocaremos el problema de la ciencia desde un punto de vista distinto al adoptado en las unidades anteriores, en las que se consideraba a la ciencia exclusivamente como una forma de conocimiento, la ms relevante, por cierto, de nuestro mundo occidental. En esta unidad analizaremos a la ciencia como un fenmeno social. Es preciso tener en cuenta que la consideracin de la ciencia como un modo de conocimiento y la ciencia como fenmeno social sealan aspectos distintos pero interdependientes, de modo que en el desarrollo de los temas

se mostrarn vnculos entre uno y otro modo de enfocar el estudio de la ciencia. En tanto institucin pblica, la ciencia guarda varios tipos de relaciones con el resto de la sociedad: poltica, econmica, cultural, etc. En esta parte del programa veremos a la ciencia desde dos perspectivas: una es la perspectiva poltica desde la cual el fenmeno tecno-cientfico se muestra como un asunto de inters pblico, por lo tanto, un objeto ms de las polticas pblicas que un gobierno debe implementar en todas las reas de su incumbencia. El segundo enfoque sobre la ciencia que desarrollaremos hace hincapi en el aspecto tico que presenta

la ciencia, que es un aspecto ineludible de todo fenmeno social. En relacin con el primer punto se trabaja el texto poltica cientfica: problemas y perspectivas, Documento de Ctedra de Mara Beatriz Contratti. Como se dijo ms arriba, la ciencia y la tecnologa constituyen un rea de inters de los Estados, por eso los gobiernos respectivos deben disear polticas destinadas a organizar las actividades que giran alrededor de esos dos procesos. No pueden llevar a cabo esta tarea unilateralmente pues su complejidad e importancia para la organizacin y crecimiento de un pas excede en mucho los propsitos de un grupo poltico. Por lo general, la

planificacin de una poltica cientfica y tecnolgica, que es el documento ms general y exhaustivo de los principios, propsitos y actividades que se llevarn a cabo en todo lo que concierna al establecimiento, ejecucin y promocin de dicha poltica, se realiza en forma conjunta con la comunidad educativa, los distintos sectores econmicos y las restantes fuerzas polticas, aunque sea de modo indirecto, adems de atender a las lneas que en la materia se ofrecen en otros pases con los cuales mantiene relaciones de cooperacin. Es por esto que cada una de las cuestiones involucradas en la planificacin genera debates tericos y posiciones encontradas. En el texto mencionado se

intenta destacar este hecho, pues as se pone de relieve que el diseo y ejecucin de una poltica cientfica no es producto de decisiones errticas o azarosas; por el contrario, requiere un fundamento terico afianzado y un arduo trabajo de coordinacin y centralizacin entre actores y sectores. De acuerdo con este espritu, las cuestiones que se trabajarn en los apartados del Documento son las siguientes: - Introduccin y conceptos fundamentales. Aqu se desarrollan las nociones bsicas y generales de lo que es una poltica cientfica. Se plantea el carcter histrico que sta tiene y la opinin de Mario Albornoz, profesor de

filosofa, investigador y encargado de la gestin en organismos pblicos y privados en el pas y el extranjero, sobre los diferentes criterios para establecer el momento en que dicha poltica surge. - La planificacin de la poltica cientfica. En este punto se analizan los factores que concurren en el planeamiento de la poltica de la ciencia y la tecnologa y las discusiones centrales en torno a la naturaleza, alcance e importancia de cada uno. La distincin (o no distincin) de la ciencia bsica y la tecnologa y sus implicaciones en la asignacin de recursos destaca una diferencia entre las posiciones del fsico y filsofo

argentino Mario Bunge y el investigador espaol en agroqumica Eduardo Primo Yfera. Asimismo, se mencionan los planteos en torno a la polmica cuestin de la importacin de patentes del doctor en economa, Daniel Chudnosvsky, y del economista Andrs Lpez. - La formacin del investigador. El tema de este apartado recae sobre la formacin de los recursos humanos, el presupuesto que debe asignarse a este fin, la dotacin de becarios, etc., cuestin central en la planificacin de una poltica cientfica. Se considerarn las posturas de Hctor Ciapuscio, doctor en filosofa, y de Carlos R. Abeledo, doctor en qumica y ex presidente del CONICET, sobre la evaluacin que

merece el sistema cientfico argentino en relacin con el punto en anlisis. - Caractersticas de la poltica cientfica argentina. Siguiendo el pensamiento de Albornoz, se mencionan las distintas etapas que se han seguido en la Argentina en materia de poltica cientfica, las caractersticas de cada una y principales representantes. Se examina si en nuestro pas se ha alcanzado un nivel acorde con los estndares internacionales actuales en relacin con el papel desempeado por las nuevas tecnologas en la implementacin de polticas. Se considera la informacin aportada al tema por el ya nombrado Abeledo y del economista, ex secretario de Ciencia y

Tcnica y actual rector de la Universidad de Ro Negro, Juan Carlos Del Bello. - La investigacin cientfico-tecnolgica en la Universidad. En este punto, el anlisis se centra en la cuestin del papel que debe cumplir la universidad dentro del sistema de investigacin. Aunque en las ltimas dcadas se han formulado proyectos de avanzada en el campo de la investigacin en los que se contempla, entre otras cosas, la articulacin entre investigacin y empresa, lo cual involucra una reformulacin completa de bases y objetivos en las polticas de la ciencia y la tecnologa; la universidad no ha acompaado estos cambios en forma

regular y orgnica. Las razones sobre ste y otros problemas de la universidad son examinados en este apartado por Ciapuscio, Abeledo y Del Bello. - Conclusiones. Se sintetiza la problemtica que se ha ido desmenuzando a lo largo del artculo y se cierra el mismo con una reflexin de Albornoz sobre la situacin argentina actual en materia de poltica cientfica y tecnolgica. El segundo aspecto que se considerar sobre la ciencia como fenmeno social se refiere a la relacin entre ciencia y tica. El texto que trabaja el tema es tica y Ciencia. La responsabilidad social del cientfico. Crtica de la teora sobre la neutralidad valorativa

de la ciencia y de la actividad cientfica. Polmica entre cientificismo y anticientificismo, Documento de Ctedra de Contratti. La relacin entre ciencia y tica tiene una historia y esta historia se ha ido articulando a travs de los cambios sociales, las distintas concepciones filosficas de la ciencia y los proyectos polticos, entre otras cosas. Es por esto que se han generado entre los estudiosos del tema posiciones encontradas sobre la naturaleza y alcances de la relacin; incluso dentro de cada una de las posturas se puede encontrar una gran diversidad de matices, lo cual hace dificultosa una presentacin unitaria de la cuestin. Pero quiz el factor

fundamental en la dispora de perspectivas sobre la relacin entre ciencia y tica, sean las situaciones paradjicas que crea la cultura cientfica y tecnolgica a la sociedad actual. El rostro dual de la ciencia y la tecnologa siempre ha sido reconocido: por un lado, stas traen progreso y bienestar en casi todos los rdenes de la vida, pero su marcha, desde su surgimiento en la modernidad, es tan arrolladora que impide reestructurar a tiempo la integridad tica necesaria para acompaar esos cambios. Es as que se ven cuestionadas desde la perspectiva tica, en general, por los daos que ocasionan a su paso, o en particular cuando se produce un descubrimiento o

una innovacin por la inestabilidad y conflictos que generan en la vida social. A partir de este planteo general, el Documento se organiza del siguiente modo: - Introduccin. Se anticipan los temas y el enfoque de los mismos que se van a desarrollar en el texto. - Caracteres de la sociedad contempornea. Se realiza una semblanza de la sociedad actual cuyos rasgos ms sobresalientes llevan la impronta de la ciencia y la tecnologa. Esta caracterizacin muestra la conflictividad generada por estos factores en la vida social que hacen imperativa la intervencin de la reflexin tica para en algunos casos

detener, y en otros contener, el poder adquirido por la ciencia y la tecnologa sobre el resto de los sectores sociales. - La tica y sus relaciones con la ciencia y la tecnologa. Se ofrecen algunas nociones bsicas sobre las que se basa la tica y se examina el sentido preciso del concepto de responsabilidad en el marco de la relacin entre tica y ciencia. Tambin se desarrolla la nocin de tica aplicada y se hace referencia a sus diversas ramas. Se plantea la discusin sobre si se puede adjudicar al conocimiento mismo involucrado en las teoras cientficas y tecnolgicas la responsabilidad por los efectos nocivos ocasionados por la ejecucin de los proyectos cientficos y tecnolgicos.

Otra cuestin relacionada al enunciado del apartado es la que se refiere a las evaluaciones de riesgo; se pone en duda el valor que tienen las evaluaciones obtenidas por procedimientos exclusivamente estadsticos al tiempo que se proponen otros modos ms democrticos y, por lo tanto, ms cercanos a una perspectiva tica. - La tica de la investigacin cientfica. Aqu se trabaja la cuestin de la conducta del cientfico en el proceso de investigacin. Este proceso se lleva adelante mediante la aplicacin del prestigiado mtodo cientfico. El planteo central aqu es: el escrupuloso cumplimiento de los pasos del mtodo basta para que la investigacin resulte

ticamente correcta? Algunos dan una respuesta afirmativa a esta cuestin, otros una respuesta negativa. Estas respuestas estn condicionadas por la distincin entre valores cognitivos y valores morales, si slo los primeros cuentan en el accionar del cientfico o tambin juegan los segundos en el curso de su trabajo. La autora del texto toma partido por la segunda opcin, y con el fin de especificar los problemas ticos que pueden surgir en el transcurso de la investigacin se analiza un documento de la National Academy of Science de Estados Unidos: On Being a Scientist: Responsible Conduct in Research. - El cientificismo, el anticientificismo y la tica. Se ha denominado cientificismo

y anticientificismo a dos posturas opuestas respecto de la naturaleza de la ciencia, su valor epistmico y su rol social. Para el cientificista, la ciencia es la nica empresa intelectual que proporciona conocimiento universal y objetivo; al satisfacer los estndares cognitivos ms altos tiene un rol preponderante sobre las dems instituciones sociales. El anticientificista, por el contrario, piensa que la ciencia es una empresa humana entre otras, y por lo tanto, est sujeta a los mismos vaivenes polticos y sociales a que estn sometidas el resto de las instituciones. Mario Bunge es considerado uno de los ms notables representantes de la postura cientificista.

Pero el cientificismo estara de algn modo respaldado si la posicin epistemolgica a la que se considera asociado, el Positivismo lgico, est a su vez totalmente justificada. A travs del texto de Marta I. Gonzlez Garca, Jos A. Lpez Cerezo y Lujn Lpez, Ciencia, tecnologa y sociedad (1996), se pone en duda esa posibilidad. Estos autores enumeran una serie de objeciones epistmicas al Positivismo que parecen desautorizar su pretendida preeminencia sobre todo otro conocimiento. - Ciencia bsica, ciencia aplicada y tecnologa. Otro de los pilares sobre los que se asienta la posicin cientificista es la consideracin de la ciencia bsica

como un proceso separado y diferente en sus objetivos y resultados de la tecnologa: aqulla slo se ocupa de conocer el mundo, sta de transformarlo. La distincin lleva a considerar a la ciencia ticamente neutral y a la tecnologa como ticamente responsable por las consecuencias de las aplicaciones de las teoras cientficas. Nuevamente, Bunge se presenta como uno de los defensores de esta idea junto al filsofo alemn Nicholas Rescher. Evandro Agazzi (filsofo italiano), por su parte, ofrece una concepcin ms matizada del punto en cuestin: ve una vinculacin tan estrecha entre ciencia y tecnologa que las considera como un fenmeno unitario: la tecnociencia, y

esta forma de entender la relacin da cabida al enfoque tico. Por ltimo, se expone el pensamiento del doctor en filosofa e investigador mexicano Len Oliv, quien mantiene una postura sobre la responsabilidad tica de la ciencia que es afn con la sustentada por la autora del Documento. - Reflexiones generales sobre el tema. Aqu se hace hincapi en una concepcin de la ciencia como una institucin integrada al cuerpo social y solidaria con l. Se remarcan los efectos negativos de la ciencia que generan incompatibilidad con la posicin cientificista, al tiempo que se muestran las dificultades conceptuales que tiene que enfrentar una tica de la ciencia por

la diversidad de aspectos que el tema conlleva.

Actividades de aprendizaje
ACTIVIDAD 1. LA PLANIFICACIN
DE LA POLTICA CIENTFICA 1.1. En el Documento Poltica cientfica: problemas y perspectivas se analizan algunas cuestiones que surgen en el proceso de armado de la planificacin de la poltica cientfica. a. Lea nuevamente los puntos del texto que tratan sobre las siguientes cuestiones:

- La distribucin del presupuesto en las reas consideradas prioritarias: investigacin bsica, investigacin aplicada y desarrollo tecnolgico. - La poltica cientfica orientada al desarrollo, en el que se seleccionan las actividades que permiten que el pas alcance niveles de competitividad. b. Cul es el problema en discusin correspondiente a cada uno de los aspectos sealados en a.? c. Cules son las posiciones respectivas? (Si se mencionan autores que las sostengan, nmbrelos; si no, exprese slo la idea) d. Esquematice los argumentos que respaldan a cada una.

1.2. Qu problemtica trae aparejada la transferencia de los resultados de la investigacin a los sectores productivos?

ACTIVIDAD 2. LA FORMACIN DEL


INVESTIGADOR a. A qu aspecto de la planificacin de la I+D alude el ttulo del apartado? b. Relacione las nociones de conocimiento e innovacin. c. Explique en qu consisten el modelo lineal de innovacin y las crticas que l mismo ha recibido. d. Cules son las ventajas del modelo interactivo? ACTIVIDAD 3. CARACTERSTICAS DE LA POLTICA CIENTFICA ARGENTINA

a. Complete el siguiente cuadro sobre las etapas cumplidas por la poltica cientfica argentina de acuerdo con los conceptos vertidos por Albornoz: Caracterssticas Etapas Problemas principales

b. Por qu, segn Albornoz, la Argentina se estanca en la etapa de institucionalizacin de la ciencia en lugar de progresar en lnea con los pases desarrollados? c. Cul es la situacin actual del problema mencionado en b.? ACTIVIDAD 4. INVESTIGACIN E

INNOVACIN EN LA ARGENTINA Lea el caso sobre la produccin de alimentos y luego responda las preguntas que estn a continuacin. Lunes 3 de febrero de 2003

ALTA TECNOLOGA: "MADE IN ARGENTINA" RA EL SECADO


DE ALIMENTOS

EN CHUBUT, UNA PLANTA NICA EN EL MUNDO Permite tratar frutas y hortalizas, y

conservarlas indefinidamente sin necesidad de la cadena de fro - La liofilizacin posibilita aumentar 12 veces el valor de los alimentos - La tcnica fue inventada por incas y vikingos - Invap desarroll una ingeniera original En el gris de la estepa patagnica, el valle inferior del ro Chubut y su corredor de chacras y cipresales esconden un as de espadas para el campo y la industria criollos. La novedad es una planta de liofilizacin (secado por sublimacin) para alimentos. Pertenece a la empresa

Nutripac SA, y su ingeniera, por ahora nica en el planeta, fue desarrollada por la firma nuclear y espacial Invap SE de Ro Negro gracias a crditos del Fondo Nacional de Tecnologa (Fontar) y avales de la provincia de Chubut. La liofilizacin, poco conocida en la mesa local, no altera la estructura fsico-qumica del material original crudo, pero permite su conservacin indefinida sin cadena de fro. A diferencia de lo que pasa en el secado por calor, con la liofilizacin el aspecto, la textura, el sabor y el aroma del alimento crudo no se pierden. Por el contrario, se intensifican. El proyecto tuvo

prioridad nacional y provincial porque supone dos apuestas simultneas: el pas puede mejorar mucho su performance de exportador frutihortcola, pero adems podra vender fbricas de liofilizados "llave en mano" aprovechando que tiene la planta ms avanzada del mundo, una que vale la mitad, consume la mitad de energa y prcticamente no se rompe. Si para un tecnlogo en alimentos liofilizar es extraer ms del 95 por ciento de agua, para un transportista significa llevar diez veces ms mercanca, pero sin unidad frigorfica a cuestas; para un supermercadista, stocks de frutas y hortalizas sin gastos de conservacin,

y para un economista, valor agregado local para nuestras exportaciones. Pero mucho valor agregado: el proceso permite transformar un kilogramo de frutillas frescas, que vale 20 centavos de dlar, en 70 gramos. Slo que cuando se junta un kilogramo de liofilizado (a partir de 13 kilogramos de material crudo), el valor del producto terminado no es, como indican las razones y proporciones, de 2,6 dlares. Es de 30 dlares. Casi 12 veces ms. Aunque la liofilizacin es compleja, su fsica es simple: previamente congelado y trozado, el alimento va a una cmara hermtica donde soporta un alto vaco como slo lo conocen

los astronautas: de los 1025 milibares tpicos a nivel del mar, la presin de aire cae debajo de 1 milibar, lo que supone una zambullida igualmente brutal de la temperatura. El agua est petrificada como hielo, que en alto vaco "sublima", es decir, pasa directamente de slido a gas, evaporndose despacio de los tejidos, que siguen intactos. Por eso, una frutilla liofilizada parece seriamente una frutilla y no una pasa, aunque pesa menos que el telgopor. El gusto, sin embargo, no es de telgopor. El tecnlogo logra que la sublimacin no arrastre los aceites aromticos livianos del alimento. Por ende, sabor y olor no slo

permanecen intactos, sino que se concentran en la superficie del material. El resultado es que una frutilla liofilizada tiene, aunque parezca raro, ms gusto a frutilla que una fresca. Y es crocante. Y lo mismo vale para la carne, la papa, la pera, la zanahoria... "Para poner en cifras el potencial del producto -explica el doctor Jorge Yanovsky, de Nutripac, rarsima mezcla de bilogo molecular y empresario-, digamos que si la Argentina dedicara no las actuales 800 mil hectreas de tierra a frutas y verduras, sino dos millones, y adems exportara todo ese producto liofilizado, estara generando un milln de puestos nuevos de trabajo.

Y nuevos ingresos por 30 mil millones de dlares." La especulacin de Yanovsky impresiona cuando se piensa que todo lo que genera en la Argentina el poroto de soja anda por los seis mil millones de dlares por ao. Con frutas y verduras liofilizadas se lograra el mismo resultado econmico... pero en slo 400.000 hectreas y con 400.000 nuevos puestos de trabajo, mitad en las chacras y mitad en el segmento industrial. Son nmeros que apabullan. El liofilizado surgi de la necesidad, inventado por incas y vikingos que necesitaban comida hipercalrica,

ultraliviana e imputrescible para sus raids militares. Los incas aprovechaban el altiplano, con sus noches glaciales y su insolacin diurna, para transformar la papa en chuo y la carne de llama en charqui, los primeros liofilizados de la historia. Los vikingos, con montaas ms bajas y sol ms oblicuo, liofilizaron el arenque con menos perfeccin. Pero aunque hoy los sobres con sopas, guisos y otros liofilizados son ms cosa de astronautas, montaistas y comandos militares que del comensal comn, hay otro mercado ms interesante: son las firmas alimentarias que descubrieron los liofilizados por su

sabor intenso, su consistencia crocante y su carcter novedoso. Una constante en la antropologa alimentaria muestra que lo que empieza como tecnologa de conservacin la cultura lo transforma en capricho del paladar. Nadie piensa hoy en pimientas, ahumados o fermentacin como modos de evitar la prdida de carnes o lcteos, pero hace milenios su origen fue se. "Ms que en el terreno de la necesidad, en el del gusto hay un mercado de lmites desconocidos, todo a crear y conquistar", resume Yanovsky. Invap: del uranio a las frutillas

La tecnologa de liofilizacin deriva de otra francamente vetusta: la de refrigeracin. Al hacer vaco en una masa de gas, una bomba genera fro. En el caso de una heladera, el fro es lo importante, y en el caso de un liofilizador, un subproducto, pero la idea de base es la misma desde ms o menos 1870. En Gaiman hay otra idea mejor: en lugar de bombear trabajosamente el aire hacia afuera de la cmara de vaco se lo puede aspirar con un chorro hipersnico de vapor. ste pasa rugiendo a cinco veces la velocidad del sonido por una tobera anexa conectada con la cmara mediante un empalme en T. El teorema de Bernouilli garantiza que

en el vapor hipersnico habr zonas de presin bajsima que "chuparn" el aire de la cmara hasta expulsarlo casi enteramente. Slo que sin enormes, costosas y frgiles bombas llenas de piezas mviles y sellos, y con la mitad del gasto de energa. Sencillo, cuesta la mitad, a la medida de un pas rico en gas (sirve para generar vapor), y funciona. La diferencia conceptual entre el liofilizador de Invap y los que hoy funcionan en otros pases es que el aparato nacional no usa bombas de vaco (carsimas y frgiles). Para crear el vaco y las temperaturas subcongelantes necesarias, el sistema de Invap usa chorros de vapor

hipersnicos, una idea que los ingleses probaron sin mayor xito en la dcada del 60, considerada desde entonces una va muerta. Y por qu funcion aqu y no all? Contesta el ingeniero Horacio Bccoli, de Invap: "En Pilcaniyeu aprendimos mucho de ambientes de alto vaco, sustancias que subliman sin atravesar una fase lquida y difusin de gases a travs de superficies porosas. Y lo que aprendimos funciona tanto para el uranio como para las frutillas. La historia mundial de la tecnologa abunda en estos cambios de escenario. Lo bueno es que sucedan tambin aqu". Opina al respecto el ingeniero

Horacio Bccoli: "Si el Invap no hubiera hecho en 1983 la instalacin de enriquecimiento de uranio de Pilcaniyeu, en Ro Negro, lo de Gaiman habra sido difcil. Pero aunque Nutripac y Pilcaniyeu son plantas con finalidades muy distintas, tienen tres asuntos en comn: ambientes de alto vaco, sustancias que subliman sin atravesar una fase lquida y difusin de gases a travs de superficies porosas. Lo que Invap aprendi con el uranio tal vez ahora le permita ganar plata al pas con tomates, frutillas, manzanas... Las vueltas de la tecnologa son notables. Tenemos la mejor herramienta. La ingeniera de Gaiman resulta tan

barata, econmica y durable, comparada con todo lo actual, que en medio ao de funcionamiento esta planta ya gener ms de 30 pedidos de informes en tres continentes". Muchos vienen de gigantes mundiales de la alimentacin. Daniel E. Arias (2003, febrero 3). En Chubut, una planta nica en el mundo. La Nacin. Lunes 4 de agosto de 2003 Materia gris de exportacin: se venden dos plantas de secado de alimentos

MXICO COMPRA ALTA TECNOLOGA ARGENTINA - Se utiliza para extraer hasta el 98% del agua de productos frescos sin alterar su estructura. - La primera de las unidades vendidas se est levantando a 350 Km de Mxico DF. - Dentro de poco se exportar otra ms. Ambas para liofilizar frutillas. Entre 2000 y 2001, la firma nuclear y espacial argentina Invap construy en Gaiman, Chubut, una planta para

secar alimentos por sublimacin, o liofilizarlos, que gasta la mitad de energa y es ms robusta que cualquier otra en el mundo. Ahora, Mxico acaba de comprar dos de esas unidades, pero adems aparecen clientes locales. El pas est por comenzar dos nuevos negocios que renen casi emblemticamente sus capitales de siempre, tierra negra y materia gris... y por bastantes millones de dlares. La primera de las unidades exportadas se est levantando ya en Quertaro, Mxico, zona industrial situada en el valle Central, unos 350 kilmetros al norte del Distrito Federal. En poco tiempo ms, el mismo cliente (Exportadora

de Hortalizas SD, gigante mexicano de la alimentacin) pedir una segunda planta, y dedicar ambas a liofilizar frutillas para la empresa Kellogg's de Estados Unidos. Ambas plantas son de considerable tamao, capaces de rendir hasta 80 toneladas anuales de producto seco. Por otra parte, la empresa rionegrina est a punto de firmar la construccin de una cuarta unidad, esta vez para una cooperativa frutihortcola neuquina. Tras dcadas de vender slo productos frescos, o a lo sumo secados al calor, ahora algunos productores del Alto valle se aprestan a imitar a los mexicanos y poner un pie en el hemisferio norte

con un producto bastante ms exclusivo. Fsica, qumica y matemtica Ms all de la fsica y la qumica, liofilizar es multiplicar dlares. Al eliminar el 98 por ciento o ms del agua del producto fresco sin alterar su estructura, sabor o calidad nutricional, la liofilizacin permite transformar un kilo de frutillas frescas (que vale 20 centavos de dlar) en slo 70 gramos de lo mismo. Slo que cuando se junta un kilo de frutillas liofilizadas (a partir de 13 kilos de material crudo) el valor del producto terminado no es,

como indican las razones y proporciones, de 2,6 dlares, sino de 30. Esto explica que, hasta que se conoci la planta de Gaiman, en 2002, hubiera "frutilleros" tucumanos dispuestos a liofilizar su producto en... China. Y para entrar al mercado ingls! Semejantes triangulaciones se originan, ante todo, en que hoy la demanda mundial de liofilizado supera largamente la oferta. Mientras esto siga as, en el rubro frutillas quien hoy factura un milln de dlares vendiendo producto fresco podra facturar 6,5 millones liofilizando slo la mitad de su

produccin, y 12 millones procesndola toda. Tanta multiplicacin de valor surge de las propiedades del producto terminado: su sabor natural se potencia, se puede almacenar indefinidamente sin cadena de fro, suele pesar entre el 5 y el 10 por ciento de lo que pesa la materia prima fresca, pasa fcilmente las barreras sanitarias y ofrece una gama poco conocida de consistencias y sabores que est originando muchos productos emergentes de consumo masivo, asuntos casi de "cocina ficcin". Por ejemplo, esa barrita de carne tipo snack que uno puede comprar en un quiosco neoyorquino para comer en la calle y que hace

"crunch" en la boca como un turrn. o papas crocantes idnticas a las papas fritas... pero crudas, nutritivamente superiores y totalmente libres de algunos inevitables subproductos cancergenos que originan las frituras. El dueo actual de la planta de Gaiman, Jorge Yanovsky, de Nutripac SA, cuyo prototipo ya ha funcionado tres aos sin problemas, hace el siguiente clculo: "Si la Argentina dedicara, no las actuales 800 mil hectreas a frutas y verduras, sino dos millones, y adems exportara todo ese producto liofilizado, estara generando un milln de puestos nuevos de trabajo, mitad en las chacras y mitad en el segmento

industrial. Y, de yapa, nuevos ingresos por 30 mil millones de dlares". Todo lo que genera en la Argentina el poroto de soja anda por los 6 mil millones de dlares por ao. Con frutas y verduras liofilizadas se lograra el mismo resultado econmico... pero en apenas 400.000 hectreas y con 400.000 nuevos puestos de trabajo. Son nmeros que apabullan: permiten imaginar un futuro agroindustrial ms integrador que el actual, basado casi nicamente en las oleaginosas. Daniel E. Arias (2003, agosto 4). Mxico compra alta tecnologa

argentina. La Nacin. a. Caracterice los conceptos de I+D, innovacin y competitividad (utilice a este fin el texto completo de Poltica cientfica: problemas y perspectivas, especialmente el apartado: "La investigacin cientfico-tecnolgica en la Universidad"). b. Cmo cree que operan los factores mencionados en 1.1.a. en el caso de la liofilizacin de los alimentos? (Aclare si encuentra un nuevo concepto, adems de los mencionados en el Documento) c. Por qu lo narrado en el caso de la liofilizacin de los alimentos difiere de las conclusiones presentadas en el texto sobre poltica cientfica? ACTIVIDAD 5. TICA Y CIENCIA

En el Documento tica y ciencia... de Contratti, se han considerado centralmente dos aspectos de la relacin entre tica y ciencia: uno est referido a la actuacin del cientfico en tanto investigador y a ciertas normas de conducta que el mismo tiene que satisfacer desde el punto de vista deontolgico de su profesin. El segundo aspecto apunta a la responsabilidad del cientfico como productor de conocimientos en la medida en que tales conocimientos son susceptibles de portar los valores que el cientfico o la comunidad de cientficos, en tanto seres humanos que viven en una sociedad determinada y en una poca histrica dada, introducen en ellos. La

presente actividad requiere que aplique los conceptos desarrollados sobre estas dos cuestiones al relato sobre los experimentos nucleares franceses en Argelia. Lea el artculo periodstico y luego resuelva los puntos que estn a continuacin. infobae.com 16/2/10

FRANCIA US CIENTOS DE SOLDADOS EN EXPERIMENTOS NUCLEARES El ejrcito francs expuso

deliberadamente a sus soldados a radiaciones durante una serie de ensayos nucleares atmosfricos realizados en los aos 60 en Argelia, segn un informe militar. Al situar soldados cerca del punto de explosin, el ejrcito buscaba "estudiar los efectos psicolgicos producidos por el arma atmica sobre el hombre", segn indica el primer tomo del "Informe sobre los ensayos nucleares franceses de 1960-1965", clasificado como secreto. El Ministro de la Defensa Herv Morin asegur que se informara "con total transparencia" sobre el nivel de exposicin a las radiaciones despus de cada ensayo.

El informe, escrito por oficiales franceses seguramente en 1998, es decir despus de que Francia dejara de realizar pruebas nucleares (en 1996), hace referencia especialmente al ltimo ensayo atmosfrico realizado en el Sahara argelino, que se llev a cabo el 25 de abril de 1961, cuando Argelia an era una colonia francesa. Poco despus del lanzamiento, se organizaron maniobras para enviar una treintena de soldados al rea contaminada. Algunos soldados se protegieron en trincheras individuales cavadas a unos 800 metros del lugar de impacto mientras que otros se acercaban a dicho punto en camiones 4x4. Estaban

equipados con botas, guantes y mscaras de combate. El objetivo era "estudiar los efectos fisiolgicos y psicolgicos producidos sobre el hombre por el arma atmica, con el fin de obtener los elementos necesarios para la preparacin fsica y la formacin moral del combatiente moderno", explica el informe. Las maniobras deban permitir tambin "realizar un programa de instruccin sobre las medidas prcticas que deben tomar los combatientes para ponerse en guardia, protegerse y descontaminarse", aade el texto. Despus de la explosin, los resultados constatados por el informe sealan "la ausencia de quemaduras

visibles, efectos mecnicos prcticamente nulos, nivel de radiactividad elevado en maniques". "Pareca, segn estos resultados, que a 800 metros del punto cero y afuera de la zona de cada de polvo radioactivo, los combatientes estaran fsicamente aptos para continuar en combate", aade el documento. El texto reconoce sin embargo que "estando a la ofensiva, si la infantera fuese llamada a combatir en una zona contaminada (... ) la vestimenta especial no les proporcionara sino una proteccin relativa y habra que reducir el tiempo de permanencia". Morin afirm que "una sntesis de ese informe fue elaborada para el

Ministerio de la Defensa en enero de 2007", relatando "las experimentaciones tcticas durante" dos pruebas nucleares. El ministro record adems que una ley de indemnizacin de las vctimas de los ensayos fue adoptada en 2009, indicando que haba exigido "un estudio complementario sobre cada tiro y el nivel de exposicin correspondiente". "Las conclusiones sern pblicas", anunci Morin. Francia ha realizado 210 ensayos nucleares desde el primero en el Sahara en 1960 hasta el ltimo de 1996 en Polinesia francesa. Miles de veteranos de los ensayos nucleares, convencidos de haber sido

contaminados por la radiactividad, luchan porque se les reconozcan los perjuicios ocasionados. En otras publicaciones se dan otras precisiones sobre el operativo francs: "Participaron 300 personas, en su mayora reclutas que 20 minutos despus de la explosin salieron de los refugios y a los 35 minutos empezaron la aproximacin a pie al punto cero y llegaron a 700 metros de distancia. Se hizo llegar una patrulla de vehculos todoterreno a 275 metros. Los autores del informe de 1998 reconocen que las autoridades militares de los aos 60 hicieron

manipular sustancias a los miembros de la tropa, pese a conocer los peligros que eso poda entraar. As, por ejemplo autorizaron a los trabajadores a no llevar mscara, lo que significaba que podan inhalar en un da el polvo radiactivo equivalente al "normalmente autorizado en tres meses". En otro artculo se informa sobre la reaccin de los afectados por la contaminacin radiactiva y la respuesta de los Estados responsables. "Existen imgenes del acontecimiento, aunque el montaje y el relato propagandstico d una versin de los hechos completamente diferente. Pierre Messmer lleg vivo a los 91 aos, pero el ministro Gastn

Palewski muri en 1984 como consecuencia de una leucemia, supuestamente provocada por el incidente. Se desconoce el nmero exacto de afectados por este y otros incidentes, y por los ensayos de Reggane, entre soldados y habitantes de la regin, incluyendo tuaregs que no pasaron por un proceso de descontaminacin (unas duchas no muy fiables). Los primeros se organizaron hace unos aos para reclamar una investigacin de las consecuencias de los accidentes, as como indemnizaciones. Las asociaciones de los afectados por los ensayos nucleares, Aven y Moruroa e Tatou (esta ltima rene a las

vctimas de los 193 ensayos que se realizaron en Polinesia tras la finalizacin de los experimentos argelinos en 1967), llevan tiempo reclamando, adems de indemnizaciones, cosas tan elementales como un censo de potenciales afectados, la creacin de un comit de seguimiento de las consecuencias sanitarias y medioambientales de los ensayos nucleares. El pasado mes de julio se anunci la creacin de un comit "verdad y justicia" de apoyo a dichas asociaciones. Pero ni el Estado francs ni el argelino, cmplice de la antigua potencia colonial, estn dispuestos a dar pasos que impliquen

asumir algn tipo de responsabilidad jurdica". Francia us cientos de soldados en experimentos nucleares. (2010, febrero 16). Disponible en <http://www.infobae.com/mundo/5009 0-0-francia-uso-cientos-soldadosexperimentos-nucleares> a. Cules son los objetivos del experimento descrito en el caso? b. Mencione tres objeciones al procedimiento experimental narrado en el caso, desde el punto de vista del concepto de responsabilidad analizado en el apartado "La tica y sus relaciones con la ciencia y la tecnologa", del Documento de Ctedra.

c. Qu norma o normas de la investigacin se ha incumplido especficamente en este caso? d. i) Determine cul o cules de los siguientes agentes deberan haber evaluado la implementacin de experimentos como el descrito antes de su ejecucin: - El gobierno. - La comunidad cientfica. - El o los cientficos que disearon e intervinieron en el experimento exclusivamente. - La sociedad en cuyo pas se realiz la experiencia - Todos los nombrados. ii) Explique la razn de su eleccin. e. identifique en el caso el aspecto

cientfico y el tecnolgico. Suponiendo cul de los aspectos mencionados se encontraran incorporados valores polticos, econmicos y sociales para el cientificista y para el anticientificista? Fundamente su respuesta. f. Qu opinaran sobre lo ocurrido en Argelia, en el supuesto de que lo hicieran, los siguientes autores: Mario Bunge, Nicols Rescher, Evandro Agazzi y Len Oliv? g. Cules son los mecanismos de reparacin con los soldados "cobayos" utilizados por el ejrcito y gobierno francs? Cree usted que son suficientes y adecuados? Por qu?

Introduccin y conceptos fundamentales Se denomina poltica cientfica al conjunto de disposiciones gubernamentales destinadas a organizar

el potencial investigador de un pas y orientarlo en su crecimiento y su aplicacin en otras reas donde los resultados de la investigacin son necesarios para su desarrollo. Mario Albornoz (investigador argentino) caracteriza este proceso como "un mbito de polticas pblicas referido a la produccin de conocimiento, su comercializacin, adaptacin e incorporacin al aparato productivo [...] configura un vasto espacio en el que confluyen aspectos de diversas polticas, como la educativa, la econmica y la industrial. La poltica cientfica y tecnolgica est entramada con todas ellas, lo cual le confiere centralidad, en el marco de las

polticas pblicas, y refuerza su naturaleza poltica" (Albornoz, 1990. 171). A travs de las distintas etapas histricas la poltica cientfica se fue complejizando ms y ms. Mientras a fines del siglo pasado apenas se poda hablar de administracin de las actividades cientficas, hoy tenemos un gran nmero de categoras distintas interviniendo en la conceptualizacin del proceso. Por eso haremos continuas referencias a fenmenos que se han generado en su carcter peculiar especialmente a partir de la segunda mitad del siglo XX, como el "proceso de innovacin", el "cambio tecnolgico" y su impacto en la economa y la vida

social, a las actividades de investigacin y desarrollo (representadas habitualmente por la sigla I+D) que son objeto de la poltica cientfica y a la competitividad, uno de los fines fundamentales de esta poltica dada la existencia de un mundo globalizado. La vigencia institucional de la poltica de la ciencia tiene una historia muy breve; la mayora de los autores que se ocupan del tema acuerdan en que la Segunda Guerra Mundial marca la consolidacin de tal institucin. Adems de las razones de ndole blica que impulsaron el avance de la investigacin en ese momento (entre los productos del inters militar se encuentra el proyecto

Manhattan de la bomba atmica, el radar en la defensa area de Inglaterra, los cohetes balsticos alemanes, etc.), existieron otras razones procedentes de las caractersticas adquiridas en su expansin por la ciencia y la tecnologa: el papel que juegan en el desarrollo econmico de un pas, la creciente tendencia a la aplicacin de las teoras cientficas, los efectos beneficiosos o nocivos de la tecnologa que provocaron el consiguiente inters social por ella y por la ciencia a la cual est asociada y la aparicin del cientfico como un profesional independiente. Albornoz no acuerda con esta consideracin histrica, al menos en el caso latinoamericano, especialmente el

argentino. Segn este autor es "[...] hasta cierto punto, un lugar comn en la bibliografa sobre el tema dar por sentado que la poltica cientfica y tecnolgica constituye un fenmeno que emerge despus de la Segunda Guerra Mundial. Pero esta suposicin no toma en cuenta importantes decisiones adoptadas en pocas muy anteriores, ni la centralidad que lleg a tener el problema cientfico en el discurso poltico, en determinados perodos de la historia argentina [... ] una perspectiva histrica que no se restrinja a las dcadas recientes, permitir ver que la ciencia y la tecnologa se han acercado al centro de la escena poltica, o se han alejado

a zonas perifricas, por razones que han obedecido tanto a los proyectos polticos hegemnicos como a la percepcin de los cientficos respecto de sus propios intereses" (Albornoz, 1990. 172). La planificacin de la poltica cientfica La articulacin de todas las disposiciones decididas por los gobiernos en el rea cientfica y tecnolgica se plasma en planificaciones de largo alcance. stas deben ser el resultado del consenso de todas las fuerzas polticas de un pas, nico modo de alcanzar la estabilidad necesaria para el logro de los objetivos propuestos. En el diseo de los planes de poltica

cientfica se toman en cuenta distintas cuestiones que van de lo general a lo especfico. Dada la diferente generalidad de los aspectos considerados en los planes, los actores intervinientes en su confeccin son tambin distintos, operan en diferentes niveles. En este trabajo comentaremos algunas de esas cuestiones, aquellas que a nuestro juicio muestran en toda su riqueza el debate terico sobre la cuestin de la poltica de la ciencia y que son tambin, frecuentemente, objeto de debate pblico y de inters inmediato para el especialista en este tema, y para el ciudadano. Algunos de los factores a tomar en cuenta en los planes de polticas

cientficas son, esquemticamente, las siguientes: a) el monto total de inversin en I+D; b) la seleccin y valoracin de los objetivos mayores: ciencia bsica, desarrollo industrial, servicios, etc.; c) la seleccin de reas prioritarias cientficas y tecnolgicas, donde no slo se debe tener en cuenta el nivel de desarrollo a alcanzar, sino tambin la formulacin de prospectivas, es decir la previsin de los avances cientficos y tecnolgicos y los cambios sociales; d) la organizacin del sistema de investigacin y la coordinacin interinstitucional e interdisciplinar; e) el control y la evaluacin de los planes implementados en relacin con los resultados de la produccin cientfica y

tecnolgica; f) la aplicacin de los resultados de la investigacin, es decir, la transferencia de informacin hacia los sectores productivos (innovacin); g) la promocin y organizacin de las relaciones cientficas internacionales (Yfera, 1994).[33] Como hemos sealado ms arriba, no vamos a analizar puntualmente todas estas cuestiones si vamos a considerar globalmente ciertos problemas que se presentan al momento de hacer explcitos los planes de poltica cientfica. De todos modos, los aspectos enumerados no slo no funcionan en forma puramente secuencial, sino que adems interaccionan creando vas de ida y vuelta y generando mecanismos de

condicionamiento mutuo. Veamos la primera cuestin referida a la inversin en Investigacin y Desarrollo. Es una tendencia general que todos los gastos de investigacin estn unificados en el rubro I+D. El peso otorgado a la I+D dentro del presupuesto total de un pas corresponde a una decisin poltica cuyo criterio de decisin se basa en las necesidades relativas de todas las reas que debe cubrir el presupuesto y a la funcin que cumplen la ciencia y la tecnologa como variables del desarrollo econmico-social para un Estado determinado. Pero el criterio mencionado no se aplica de modo uniforme en todos los pases. En algunos se suele fijar el monto en

I+D adecundolo al 3% del PBI. Este criterio es menos racional que el mencionado en el prrafo anterior, porque al considerar slo la cantidad de recursos que se van a volcar en investigacin cientfica y tecnolgica se corre el riesgo de que dichos recursos sean malgastados, al no estar guiados por una planificacin ordenada y compatible con otros criterios de crecimiento a nivel nacional. Los comentarios que haremos a continuacin estarn, por lo tanto, orientados por el primero de los criterios mencionados. Con el fin de realizar la distribucin global del monto asignado en ciencia bsica o aplicada y tecnologa, el

estudio del presupuesto debe contemplar, en primer lugar, los gastos de estructura y funcionamiento. Esta estructura est constituida por los centros de investigacin y las universidades. En el presupuesto debe tenerse en cuenta el mantenimiento de los institutos de investigacin, el crecimiento del potencial investigador en la creacin de nuevas entidades y aumento del personal, la dotacin de la investigacin universitaria y la renovacin del instrumental de alto costo. Es criterio corriente que la cantidad asignada para mantenimiento del personal no debe superar el 70% del presupuesto total. En el caso de nuestro

pas, resulta ilustrativo el documento del mdico argentino Bernardo Houssay de 1921, en el cual este investigador se quejaba de que aqu se gastaba un 90% del presupuesto en sueldos y un 10% en laboratorio, mientras en Berln suceda casi lo contrario: 61% en los institutos y 2l% en sueldos. Con el correr de los aos, la situacin no ha variado sustancialmente. En segundo lugar, otro aspecto que debe caer bajo la consideracin presupuestaria est ligado a los objetivos considerados prioritarios. Es decir, las cantidades necesarias para la investigacin bsica, la investigacin aplicada y el desarrollo tecnolgico y los mbitos especficos dentro de cada

una de stas, que se deben calcular en funcin de la importancia que cada una de estas reas representa en el desarrollo del pas. Consideremos con un poco ms de detalle los problemas que surgen en este punto. En la seleccin de prioridades entran en juego diversas cuestiones. Segn lo define Eduardo Primo Yfera (qumico espaol, 1918-2007): "La poltica de I+D debe decidirse en funcin de la poltica socioeconmica general y nunca desconectada de sta, lo que supone que la mayor parte de la comunidad cientfica deber adaptar sus objetivos a los grandes problemas tecnolgicos, econmicos y sociales de su pueblo" (Yfera, 1994. 333). Este

autor compara el modo en que se da este proceso de seleccin en los pases desarrollados y en los de menor desarrollo a la luz de lo que se considera en los pases avanzados el principio fundamental: el objetivo prioritario de la poltica cientfica de un pas debe subordinarse a sus necesidades sociales y econmicas y no perseguir el incremento de investigacin cientfica como un fin en s mismo. Es posible ver la diferencia de enfoque entre pases en desarrollo y desarrollados a travs de la declaracin de principios de las polticas cientficas de Francia y Alemania. Las declaraciones dicen, en el caso alemn, lo siguiente: "El gobierno federal

alemn ha tomado conciencia, y as lo expresa pblicamente, de que el fomento y perfeccionamiento de la capacidad de rendimiento cientfico no apunta tan slo a los fines de la poltica cientfica, sino que es premisa fundamental para las realizacin de todos los dems fines nacionales [...] El gobierno federal actuar de modo que la poltica de investiga cin no incida slo sobre el desarrollo de la ciencia, sino que tendr en cuenta las necesidades nacionales relacionadas con ciertas parcelas de la ciencia y la tecnologa y sus consecuencias en la sociedad" (Yfera, 1994. 334). Por su parte, en el llamado plan Giscard, que preside las investigaciones

de Francia desde los aos setenta, de acuerdo con Yfera, se afirma lo siguiente: "Sern prioritarias las investigaciones de fuerte impacto colectivo y las que, en el sector industrial y agronmico, favorecen el crecimiento de las exportaciones y la independencia nacional en el aprovisionamiento de energa y materias primas" (Yfera, 1994. 335). En cambio, en los pases sin potencial econmico y poltico independiente, segn Yfera, "las prioridades sectoriales son vagas, difuminadas y poco fundamentadas en estudios socioeconmicos serios, y la trascendencia de la actividad de I+D no se valora a la hora de

seleccionarlas" (Yfera, 1994. 334). Y lo que es quiz peor, el resultado de las investigaciones en ciencia bsica practicadas en estos lugares se publica en revistas cientficas internacionales con la triste consecuencia de que es aprovechado por otros pases poseedores de un desarrollo econmico afianzado. En realidad, hace ya bastante tiempo se ha instalado en los medios conectados con las polticas de la ciencia una discusin sobre el monto que se debe destinar a la investigacin bsica en relacin con otras reas. Reproducimos la versin de Yfera de esa polmica segn la cual hay dos posiciones extremas:

Una de esas posiciones expresa lo siguiente: "Hay que dar preferencia a la ciencia bsica libre porque lo dems viene por su cauce natural; la investigacin bsica es la causa primaria de la riqueza". La otra, en cambio, dice: "La ciencia bsica necesaria para el desarrollo tecnolgico est en las revistas a disposicin de todos; los pases menos ricos deben aprovecharla para crear riqueza; los pases ricos son los que pueden estar en cabeza de la investigacin bsica" (Yfera, 1994. 339). Los ejemplos de Estados Unidos y Japn apoyaran la ltima posicin. El primero se sirvi de la ciencia bsica europea

para lograr su potencial industrial y ms tarde logr su liderazgo en materia cientfica. Japn tom tanto la ciencia bsica como import la tecnologa que necesitaba, y ello le sirvi de plataforma de lanzamiento, en primer lugar, de su desarrollo tecnolgico y luego, en los ltimos aos, de su nivel cientfico. El fsico y filsofo argentino Mario Bunge (1997), en cambio, denomina practicismo o anticientificismo a la posicin segn la cual los intereses nacionales, considerados de naturaleza econmica y social pero no cultural y poltica, deben orientar la poltica cientfica de un pas. Rechaza esta perspectiva porque desde ella se abona

el desconocimiento sobre la funcin cultural que tiene la ciencia bsica, esto es la de satisfacer la necesidad de saber y la de promover la independencia intelectual de la poblacin, subordinando la ciencia a los imperativos del desarrollo econmico y la produccin. Adems, con ese desprecio a la ciencia bsica -afirma el autor- se olvida al mismo tiempo que la ciencia es precondicin del desarrollo tcnico, "La ciencia til favorecida en nuestros pases en desmedro de la investigacin bsica, ha resultado intil" (Bunge, 1997. 106). El autor proporciona numerosos ejemplos que muestran el valor de la ciencia para el desarrollo

tecnolgico: "A quienes dudan de los beneficios prcticos a largo alcance de la investigacin bsica, habra que recomendarles la lectura de Pasteur, o de una buena historia de la ciencia o de la tcnica y recordarles los siguientes ejemplos. La industria mecnica no puede prescindir de la mecnica terica, que a su vez naci de la astronoma, ciencia pura si la hay, aunque necesaria para la navegacin de altura y la confeccin de calendarios [...]" (Bunge, 1997. 106). En consecuencia, en los pases subdesarrollados siempre es ventajoso practicar la investigacin bsica, aunque deben darse ciertas condiciones de orden cultural para que ello sea posible.

Tales condiciones, sin embargo, no constituyen una barrera insalvable. Menciona a modo de ilustracin, entre otros, el caso de Florentino Ameghino,[34] quien desarroll una rama de ciencia bsica en un contexto cultural que no era el ms favorable para ese tipo de actividades (Bunge, 1997. 114). No slo se puede hacer buena ciencia bsica en el subdesarrollo, sino que es ms fcil hacerla que hacer buena tcnica. sta exige inversiones en produccin que un pas pobre no tiene. Por otro lado, la innovacin tcnica depende de la demanda, y "A quin se le puede ocurrir disear un nuevo proceso metalrgico en un pas sin industria metalrgica, o una nueva

mquina de escribir en una nacin de analfabetos?" (Bunge, 1997. 117). En cambio, lo nico que se necesita para hacer investigacin terica, del tipo de la matemtica pura, fsica y qumica tericas y ciencias sociales tericas, es "papel, lpiz y bibliografa". Incluso pueden realizarse estudios experimentales siempre y cuando no signifique la utilizacin de equipos costosos. Para Bunge entonces, la libertad de investigacin debe estar necesariamente contemplada en la planificacin de las polticas cientficas. Aunque la mayora de los especialistas estn de acuerdo con el postulado de la libre investigacin, el peso que le otorga

Bunge y los argumentos que ofrece sobre su primaca no son compartidos, en general, en la actualidad. Yfera entiende que en el presupuesto de I+D debe destinarse una parte a la actividad libre de los cientficos y otra al desarrollo tecnolgico. Cul sea la proporcin que le corresponde a la primera depende de criterios valorativos y pragmticos: la creatividad cientfica es intrnsecamente libre; pero hay tambin involucrada una cuestin de decisin poltica: dicha actividad responde a la necesidad de integrar grupos internacionales de prestigio en la materia. En tal caso seala este autor- el nivel profesional alcanzado por los cientficos debe ser el

adecuado al compromiso emprendido, cuestin que conviene cuidar especialmente en los pases en desarrollo donde muchas veces este requisito no se cumple. Veamos ahora otro punto de vista necesario a tener en cuenta en las planificaciones de I+D. Es el caso de la poltica cientfica orientada al desarrollo, en el que se prioriza el fomento de las actividades que permitan al pas alcanzar o sostener los niveles de competitividad necesarios para su sustentabilidad y tambin, como condicin necesaria del desarrollo interno, su participacin en el juego competitivo internacional. La cuestin central que aqu se plantea es la relacin

que debe establecerse entre la investigacin tecnolgica y las lneas de desarrollo econmico y comercial del pas. La atencin en esta etapa se dirige fundamentalmente al potencial investigador de las empresas, a quienes el Estado debe apoyar y complementar, por ejemplo, facilitndoles estudios prospectivos cuando sea necesario. En los pases desarrollados, ms del 60% de la actividad de I+D se realiza en las empresas. En los pases menos competitivos, en cambio, el Estado sigue concentrando las polticas de desarrollo tecnolgico. En estos lugares, el problema radica en la forma en que se obtiene la tecnologa producida en los

centros desarrollados, esto es, por medio de la compra de patentes o bien por acuerdos de cooperacin, aunque estos ltimos en general son de naturaleza cientfica y no tecnolgica. La utilidad que reviste la compra de patentes para lograr un nivel de desarrollo adecuado es otro asunto siempre discutido. Como dice Yfera, en s mismo no es bueno ni malo que haya un dficit en la balanza de pagos por la compra de licencias de tecnologa, pues la calificacin de la compra depende de si est integrada o no a planes de I+D. El problema de la importacin de tecnologa es analizado por el doctor en economa Daniel Chudnovsky y el

economista Andrs Lpez (1996) en el artculo "Poltica tecnolgica en la Argentina: hay algo ms que laissez faire?" La expresin laissez faire hace referencia en general a la poltica del "dejar hacer" en un asunto; en el caso del desarrollo tecnolgico, se refiere a la falta de regulacin por el Estado de la adquisicin de patentes y su consiguiente ausencia de coordinacin con los planes generales de desarrollo. Para los autores mencionados, la presencia del laissez faire en nuestro pas en materia tecnolgica se ha dado en casi todas las etapas histricopolticas y est estrechamente vinculado a la poltica econmica vigente y a la

desatencin crnica de los marcos regulatorios en muchos asuntos nacionales. En la dcada de los 90, no obstante, se adopt la modalidad del laissez faire de una manera ms deliberada en relacin con la poltica de ortodoxia liberal de la economa que, en la mayor parte de los casos, considera a la tecnologa como una variable exgena (es decir, que no juega en forma directa en el proceso de equilibrio econmico) y con otros supuestos vinculados a procesos de modernizacin de la economa. Desde esa perspectiva poltica se pensaba que:
La apertura de la economa a las importaciones estimulara una mejora en la eficiencia del sector productivo a travs de la mayor competencia en el mercado local; al mismo tiempo facilitara el acceso

a maquinaria y equipo de ltima generacin. A su vez, la liberalizacin del rgimen de IED (inversin extranjera directa) promovera la incorporacin y difusin de los conocimientos tcnicos y gerenciales de los inversores extranjeros. As mismo, la desregulacin de los acuerdos de transferencia de tecnologa sera otro estmulo al proceso de modernizacin. (Chudnovsky y Lpez, 1996. 34)

Estos presupuestos por s solos no garantizan el cumplimiento de las metas esperadas. Es preciso poner en marcha polticas de innovacin complejas y continuadas en respuesta a la demanda empresarial de modernizacin tecnolgica. La mayora de estas condiciones de una poltica de desarrollo tecnolgico efectivo a partir de la transferencia de tecnologa no se han cumplido por cuanto "las iniciativas implementadas estn desarticuladas,

carecen de rumbo definido y no generan las sinergias que deberan constituir el propsito central de la poltica tecnolgica" (Chudnovsky y Lpez, 1996. 36). A consecuencia de ello, a pesar de este intento ms coherente y fundamentado, el laissez faire tecnolgico no se diferenci de las polticas errticas del pasado. En un artculo posterior de uno de los autores citados, Daniel Chudnovsky (1999), se da cuenta de la introduccin de cambios en esta poltica. Esos cambios estn contemplados en el Plan Plurianual de Ciencia y Tecnologa 1998-2000. All ya comienza a instrumentarse un concepto, el llamado Sistema Nacional de Innovacin (SNI)

que tendr mucho peso en lo sucesivo en las polticas cientfico-tecnolgicas de la Argentina (y en otros pases tambin). Haremos una referencia a las caractersticas especficas del SNI ms adelante. Por ahora, basta decir que es un concepto que involucra a una gran cantidad de actores de diversa ndole (Estado, empresas, mercado, sistema de investigacin, universidades, mbitos sociales y otros). El plan mencionado inspirado en polticas afines con el SNI deja atrs el fracasado laissez faire. Lo importante a rescatar en la presentacin de este plan es que, a pesar de que se trata slo de la formulacin del plan y considera las condiciones concretas de la ejecucin del mismo, pone en

evidencia la necesidad de desarrollar internamente la investigacin cientficotecnolgica (no depender solamente de la inversin extranjera) y su vinculacin con el sector productivo, para lo cual la articulacin por parte del Estado de los diferentes sectores con polticas adecuadas es un factor imprescindible. Esto, por s mismo, represent en su momento un avance importante. Otro objetivo que debe ser considerado en una planificacin de I+D es la transferencia de los resultados de estos planes a los sectores productivos. Dado que el nivel econmico de las naciones est basado, de una forma cada vez ms decisiva, en el desarrollo de las explotaciones agrcolas y la industria, es

obligacin del Estado aumentar el nivel tecnolgico de dichas reas. Esta cuestin se articula, no slo en torno a la transferencias de conocimientos del Estado al sector productivo privado, sino tambin a la distribucin del financiamiento total de I+D, o sea a la parte que le corresponde realizar al Estado y a la parte que le corresponde a la empresa privada. En general, en los pases desarrollados el Estado participa en la financiacin de la I+D realizado en las industrias. Algunos datos ilustran bien la situacin: en EE.UU. el 70% de la investigacin se realiza en las empresas, pero del gasto global de I+D, el Estado aporta el 50%; en Japn el 75-80% de los gastos totales

de investigacin lo invierte el sector privado y en Suiza casi toda la investigacin la hace la empresa. En los pases de menor desarrollo corresponde al Estado liderar los emprendimientos de I+D. Puede formalizar programas conjuntos entre empresas y universidades o centros de investigacin, o proporcionar incentivos fiscales a la investigacin, subsidios, etc. Un problema aqu es que las empresas transnacionales no realizan labor de I+D en los pases poco avanzados. Por consiguiente, el pas debe importar todas las tecnologas necesarias cuyas consecuencias son, entre otras, el atraso en materia de innovacin y el desequilibrio de la

balanza de pagos. El ltimo aspecto del proceso de planificacin de I+D, estrechamente ligado al anterior, que vamos a considerar aqu, es la produccin comercial de los resultados de la investigacin, es decir la transferencia de la labor de los institutos de investigacin aplicada a la industria. En este nivel se consolida el punto final del avance tecnolgico: la "innovacin" tecnolgica. Aqu se pone en juego otro aspecto de la financiacin que representa grandes inversiones de capital sin garanta de rentabilidad, pues la innovacin producida est sujeta a muchos factores de incertidumbre, como cambios de ciclos econmicos,

desequilibrios financieros, variaciones de la demanda del mercado internacional, etc. El "capital de riesgo" que se requiere, en general, procede de inversores especficos, es decir, de entidades financieras ad hoc. Un caso es la Agence National de Valorisation de la Recherche de Francia, creada para promover el aprovechamiento de la tecnologa producida en los organismos pblicos de investigacin. La formacin del investigador Un aspecto crtico del proceso de I+D es la cantidad y la formacin de cientficos que lo lleva adelante. Lo ideal es que el nmero de investigadores no implique un gasto mayor del 65% del gasto global, porque de lo contrario no

queda nada para invertir en mantenimiento y materiales. Algunas cifras, nuevamente, ilustran la diferencia entre los distintos pases en esta cuestin: en EE.UU. la cantidad de personal entre cientficos e ingenieros asciende a 750.000; en Japn, 370.000; en Francia, 135.000 y en Gran Bretaa, 92.000. Con referencia a la cantidad de habitantes: en Espaa hay 2,21 investigadores por cada 1.000 habitantes activos, en Francia, 5/1.000 y en Alemania, 5,9/1.000. Hay que tener en cuenta, como observa Yfera y es importante para interpretar las cifras, que la escasez de investigadores es a la vez causa y efecto del bajo desarrollo. Al mismo tiempo, el

nmero del personal condiciona la poltica cientfica y sta a su vez determina la cantidad de cientficos que se requieren de acuerdo con los planes de I+D, condicionados, por su parte, a la situacin econmica general. Como vemos, se producen espirales de las cuales slo es posible salir mediante decisiones de alta poltica cuyos principios y planeamientos trascienden los intereses sectoriales hacia el crecimiento de la nacin. Una perspectiva sobre el modo en que es posible encontrar un punto de partida para tomar decisiones es la formulada por Hctor Ciapuscio. Este autor afirma: "Dado que la revolucin cientficotecnolgica actual tiene al

'conocimiento' como su factor principal, la formacin intensiva de los recursos humanos es una estrategia hbil para mejorar la situacin relativa de los pases menos desarrollados" (Ciapuscio, 1990. 257). Ciapuscio apoya su tesis en afirmaciones como la siguiente: "La materia prima estratgica de la industria no es ms el carbn, el petrleo o algn metal raro, es la materia gris. sta, si se la sabe movilizar, transforma todo; no hay ms sector de industria o de punta del pasado, sino empresas que giran sobre la inteligencia y otras que giran sobre los resortes menos poderosos, financieros y materiales" (Ciapuscio,

1994. 257). Una consecuencia de la adopcin de polticas basadas en la premisa de que el saber es el factor fundamental del desarrollo de un pas es la necesidad de estudiar el problema de los recursos humanos (formacin de profesionales, investigadores y becarios) relacionados con las polticas de la ciencia y la tecnologa y los mecanismos que permitan implementar eficazmente los resultados de dichos estudios. En este aspecto, es importante la fecha en que Ciapuscio escribe, pues los datos que ofrece al respecto hoy han perdido vigencia, aunque no podamos decir todava que se hayan revertido totalmente las condiciones de atraso.

Por ejemplo, el autor se refiere al sistema de becarios, y pone a modo de ilustracin del rezago argentino en este campo: en 1989 Brasil, de acuerdo con su Programa de becarios, esperaba contar con 10.000 becarios de este tipo; en tanto la Argentina tena, en ese mismo ao, nada ms que 100 becarios externos. sta y otras cuestiones que hacen al tema de los recursos humanos han cambiado en la actualidad. Existen ejecucin de polticas y proyectos pergeados durante las casi dos dcadas posteriores al texto de Ciapuscio que permiten visualizar mejoras importantes: en 2006 haba ya 3700 becarios slo del CONICET (Abeledo, 2006b). Ciapuscio analiza la opinin segn la

cual la escasez de investigadores en formacin se justifica por nuestra deficiente situacin econmica y la estrecha estructura del sector productivo y de servicios. Bajo estas condiciones, mayor cantidad de becarios significara un gasto intil y gente muy preparada sin lugares de insercin para su realizacin profesional. El autor califica esta perspectiva de "malthusiana"[35], Venezuela, que contaba en 1974 con poderosos recursos derivados del petrleo, implement un ambicioso programa de becas en el exterior. Cuando los beneficios del petrleo se acabaron, el pas qued con una buena infraestructura industrial, tecnolgica y universitaria.

Nosotros, en cambio, afirma, quedamos en total condicin de inferioridad frente a un vecino como Brasil, por ejemplo, que cuenta con una gran poblacin capacitada cientfica y tcnicamente. El problema es que, pese a los datos en contrario, "en nuestro pas se tiene la arraigada ilusin de que disponemos de recursos humanos de alta calidad y relativamente abundantes" (Ciapuscio, 1990. 258). Esto se debe a "la supervivencia del mito de la riqueza nacional y la autocomplacencia de algunos de nuestros dirigentes cuando se refieren al hombre argentino." En realidad, segn el autor, esta idea del hombre argentino proviene de ciertas condiciones del pasado (segunda mitad

del siglo XIX y principios del XX) en las que se destacan la conducta de ciertos sectores sociales que contribuyeron en su momento al crecimiento general en materia poltica, social y econmica del pas, a saber: "el legado de la inmigracin, la vieja escuela pblica y el perodo 'cientificista' de la Universidad" (Ciapuscio, 1990. 259). El autor sostiene tambin que, comparados con otros pases donde se ha producido una verdadera revolucin acadmica, consistente en incluir la investigacin cientfica en el nivel de grado universitario, nosotros estamos en el nivel de pregrado, ciclo de enseanza caracterizado por la enseanza

repetitiva, el libro de texto y la clase "magistral". El nuevo modelo, en cambio, pone el acento en la actitud crtica e independiente del aprendiente, favoreciendo su capacidad de creatividad y cuestionamiento. Especificando y dando forma concreta a la nocin de conocimiento, mencionada por Ciapuscio, como motor de las decisiones de poltica tecnolgica, Carlos Abeledo (doctor en qumica) parte de la nocin de un sistema nacional de innovacin, surgida de los encargados de formular polticas cientficas en los pases industrializados. Dicha nocin, afirma, "proporciona un marco til para la formulacin de polticas

tecnolgicas[36] puesto que hace explcitas las muchas y variadas formas de aportes necesarios para producir una economa innovadora y por lo tanto competitiva en los mercados actuales, cada vez ms globales" (Abeledo, 2006b). Antes de continuar ahondando en las caractersticas de este sistema nacional de innovacin, es preciso hacer algunas aclaraciones respecto del trmino "innovacin". En otro texto del autor que estamos viendo (Abeledo, 2004a), ste dice que el trmino tiene dos significados: uno responde al modelo lineal, segn el cual la relacin entre investigacin y desarrollo tecnolgico sigue la secuencia:

Investigacin bsica investigacin aplicada desarrollo tecnolgico introduccin de nuevos productos al mercado (Abeledo, 2004a) El autor sostiene que este modelo lineal ya no responde a la realidad. Originalmente surgi a partir de innovaciones excepcionales que se gestaron a raz de descubrimientos cientficos de gran importancia y notoriedad, como por ejemplo los casos de la energa nuclear, el caucho y los textiles sintticos, el radar, la computacin digital, los transistores y circuitos integrados, el lser, los antibiticos y la ingeniera gentica. Pero la mayora de las innovaciones no obedece a la secuencia establecida por

el modelo. La inadecuacin del mismo se acenta cuando se intenta aplicar a los procesos de investigacin, innovacin y desarrollo tecnolgico al mbito latinoamericano. Como explica Abeledo: "La mayora de las innovaciones que impulsan el desarrollo tecnolgico no son necesariamente invenciones de gran alcance. Algunas innovaciones son 'radicales' y conducen a la creacin de productos o procesos totalmente nuevos, en tanto que la mayora son 'incrementales' y representan una serie de pequeas mejoras en la forma de hacer las cosas. Las empresas introducen constantemente innovaciones incrementales en sus

productos o procesos para poder mantener su competitividad" (Abeledo, 2004a). A raz de estas comprobaciones, y dada la diversificacin de las economas actuales, se ha formulado otro concepto de innovacin denominado modelo interactivo, no causal, que responde a las variadas relaciones que se establecen entre los principales actores que intervienen en el proceso de innovacin. En este modelo se muestran las interacciones entre los mbitos de innovacin de las empresas y a su vez stas con las fuentes del conocimiento, es decir, las universidades y los organismos de investigacin, que en el caso de nuestro pas, son: CONICET

(Consejo Nacional de Investigaciones Cientficas y Tcnicas), INTA (Instituto Nacional de Tecnologa Agropecuaria), INTI (Instituto Nacional de Tecnologa Industrial), CNEA (Comisin Nacional de Energa Atmica), etc. Siendo un modelo no causal las interacciones pueden darse en paralelo, no linealmente. Es preciso notar tambin que el modelo descrito pone de relieve el papel de las empresas y las demandas del mercado en el proceso de innovacin. En el concepto de "sistema nacional de innovacin" se piensa la innovacin en el segundo de los sentidos, tal como lo muestra la definicin que da Abeledo: "un sistema de interaccin de empresas

(pequeas o grandes) del sector pblico y privado, de universidades y organismos estatales orientado a la produccin de ciencia y tecnologa dentro de fronteras nacionales. La interaccin entre estas unidades puede ser tcnica, comercial, jurdica, social o financiera siempre y cuando el objetivo de la interaccin sea el desarrollo, la proteccin, el financiamiento o la reglamentacin de ciencia y tecnologa nuevas" (Abeledo, 2006b). Este ambicioso programa de innovacin, plasmado en proyectos y polticas de las instituciones correspondientes, exigi prestar mucha atencin al tema de los becarios y personal de investigacin.

Desde 1990, fecha del artculo de Ciapuscio, hasta la actualidad, en todos los mbitos conectados con la implementacin de las polticas de I+D e innovacin se ha tomado conciencia de los cambios que era necesario introducir, tanto en la formulacin de polticas como en el nivel operativo, para superar el atraso en materia de recursos humanos denunciado por Ciapuscio. Como vimos, el dato del nmero de becarios proporcionado por Abeledo en 2006, contrasta fuertemente con el ofrecido por Ciapuscio. Veremos ms adelante si la enseanza universitaria tambin se ha puesto a tono frente a los desafos planteados a la educacin por la ciencia y la tecnologa

mundiales. Caractersticas de la poltica cientfica argentina Albornoz sostiene respecto de las caractersticas de la poltica cientfica en la Argentina "la hiptesis de que, a lo largo del proceso de organizacin nacional y durante el presente siglo, hasta nuestros das, la poltica cientfica se fue desplazando desde el centro hacia la periferia de la escena poltica pero, inversamente, se especific 'hacia adentro' del sistema, lo cual alent tendencias autonomizantes y corporativas." (Albornoz, 1990a. 173). Entonces, segn esta afirmacin, la poltica cientfica fue perdiendo esa centralidad no por

integrarse a la accin de otros factores importantes para el desarrollo nacional, sino ms bien por encerrarse en s misma y separarse de esos otros mbitos del progreso econmico y social. No obstante, desde su ciuda-dela continu ejerciendo su papel rector en los procesos polticos de la ciencia y la tecnologa. La distincin de las diferentes etapas de la historia de la poltica cientfica argentina que realiza Albornoz explica claramente la afirmacin antedicha, la ciencia tuvo un lugar central al comienzo del proceso de institucionalizacin de la poltica cientfica. En la segunda mitad del siglo XIX, en el perodo de organizacin nacional, se

despleg un gran inters por la ciencia expresado en el marco de un proyecto poltico articulador del sistema econmico, industrial, educativo y de la consolidacin de la institucin cientfica y su insercin en la escena internacional. La figura que se destaca en este emprendimiento es la de Domingo Faustino Sarmiento, entusiasta admirador del modelo tecnolgico e industrialista norteamericano en materia de educacin e investigacin. A pesar de este entusiasmo e inters poltico por la ciencia en este perodo, no se logr alcanzar un contenido cientfico alto. Es decir, hubo grandes dificultades para concretar las aspiraciones enunciadas en discursos y

proyectos. Inversamente, en el perodo siguiente, o sea en el siglo XX, en la etapa que Albornoz denomina de institucionalizacin de la ciencia, "La actividad cientfica se intensific, mejor su calidad y se expandi a un conjunto ms amplio de actores. Las polticas para la ciencia se hicieron ms especficas y apropiadas. De este modo el proceso adquiri un fuerte tono en lo cientfico pero perdi paulatinamente su centralidad poltica. La poltica cientfica se fue convirtiendo en un asunto de cientficos" (Albornoz, 1990a. 185). En esta etapa la figura sobresaliente es la de Bernardo Houssay, un cientfico con inquietudes por los temas de

poltica cientfica y esa preocupacin se reflejaba en todos sus escritos. Conoca bien todas las variables y problemas del proceso de institucionalizacin de la ciencia, salarios, institutos, becas, etc., por eso pudo desarrollar normativas respecto de esas cuestiones: los requisitos que deben cumplir los institutos para funcionar adecuadamente, cantidad y carcter de las becas, etc. Albornoz considera a Houssay uno de los representantes ms notorios del positivismo argentino. De todas las citas que el primero toma del segundo, la que mejor refleja esa tendencia del ilustre bilogo, a nuestro entender, es la siguiente: "[...] Por lo tanto la ciencia es bienhechora. Acorta o suprime las

distancias, permite distribuir la riqueza, disminuye el esfuerzo material, facilita la supresin de todas las esclavitudes, asegura una vida sana y ms larga, con ms confort; da las bases para mejorar el bienestar general, distribuir las ventajas y disminuir las desigualdades excesivas" (Albornoz, 1990a. 187). Todo este optimismo respecto del poder de la ciencia es trasladado por Houssay a sus planteos de poltica cientfica. En la segunda posguerra se consolida definitivamente la institucionalizacin de la ciencia, ya considerada en concomitancia con la tecnologa y el desarrollo econmico. Marc el comienzo de un segundo ciclo dentro de

este perodo de institucionalizacin caracterizado por la preponderancia del paradigma cientfico, de la "gran ciencia", especialmente de la ciencia fsica. En el plano tecnolgico, en los aos 60 aproximadamente, en el resto de Amrica Latina tambin comienzan a tomar fuerza las teoras del desarrollo centradas en la investigacin aplicada y en la problemtica de la transferencia de tecnologa. Estas cuestiones vistas preponderantemente desde el lado poltico, se analizaron bajo la oposicin "centro-periferia" en la que se puso el acento en las relaciones polticas de dominacin ejercidas por los pases centrales en detrimento de los

perifricos en virtud de su poder econmico, poltico y militar posibilitado por el desarrollo tecnolgico. Un rasgo propio de este perodo es la falta de claridad respecto de los vnculos entre ciencia y cambio tecnolgico, explicable en razn de la estrecha relacin que el desarrollo tecnolgico tiene con la vida econmica. Este problema no resuelto se present tanto en el plano conceptual como en el organizativo. El modelo descrito comienza a mostrar su agotamiento en la dcada que va de los 70 a los 80. El centro de este cambio est constituido por las llamadas "nuevas tecnologas", es decir, la

tecnologa de las comunicaciones, de la informacin y la ingeniera gentica, cuya caracterstica principal es el acercamiento que establecen entre la investigacin bsica, el desarrollo tecnolgico y el aparato productivo. En nuestro pas el reacomodamiento que se debe operar en el plano de las polticas a raz de las nuevas formas de conocimiento y difusin de estas tecnologas sofisticadas, esto es, la revisin de la racionalidad de las prioridades, la distribucin de recursos, el tipo de institucin requerida y el perfil adecuado del profesional, tard mucho en producirse. Recin avanzada la dcada del 90 y estos primeros aos del siglo XXI, se vieron los frutos del

trabajo de polticos, tcnicos e investigadores plasmados en proyectos y prospectivas que buscan armonizar el sistema argentino de I+D al concierto internacional y propiciar de ese modo el tan ansiado despegue poltico, social y econmico del pas que le otorgue al mismo tiempo un lugar destacado en el progreso de las naciones. El ejemplo ms destacado en este sentido es el Plan Estratgico Nacional de Ciencia, Tecnologa e Innovacin "Bicentenario" 2006/2010. La lectura de sus metas y objetivos muestra a las claras que en el pas se ha asumido la necesidad de actualizar los planes de acuerdo con los conceptos vertidos en este Documento:

(i) alcanzar al final del perodo una inversin consolidada en CTI del 1% del PIB, con una participacin paritaria estatal y privada, y un total de investigadores equivalentes a tiempo completo de 2,3%o de la PEA; (ii) desarrollar capacidades, recursos humanos y acciones de investigacin e innovacin en nueve "reas-problema- oportunidad" y en diecinueve "reas temticas prioritarias" (disciplinarias y tecnolgicas), en ambos casos en coordinacin con los ministerios sectoriales; (iii) desarrollar el Sistema de Informacin de Ciencia y Tecnologa Argentino (SICyTAR); (iv) evaluar externamente las instituciones nacionales del sistema nacional de CTI e impulsar programas estratgicos de fortalecimiento institucional y cambio estructural; (v) desarrollar y consolidar la federalizacin de la CTI a nivel de provincias y regiones; (vi) promover actividades de divulgacin y alfabetizacin en ciencia y tecnologa y, (vii) fortalecer los mecanismos de coordinacin del sistema y de programacin a corto, mediano y largo plazo. (Del Bello y Abeledo, 2007)

La investigacin cientfico-tecnolgica

en la Universidad En relacin con las exigencias que plantea la nueva orientacin en materia de investigacin, Ciapuscio, en su trabajo comentado ms arriba, seala las deficiencias que padece el sistema de investigacin en nuestro pas, y recomienda una rpida reconversin del mismo si queremos evitar una situacin de atraso irrecuperable frente a la velocidad del cambio tecnolgico. Entre muchas carencias, seala que "ramas enteras de las nuevas ingenieras y de las tecnologas estn desiertas" (Ciapuscio, 1990. 259). Podra esperarse que hoy las cosas hayan mejorado notablemente si se llevaran a la prctica los proyectos

establecidos, los cuales, como el denominado "Bicentenario", estn orientados a promover la innovacin. Para que esto ocurra, las instituciones de investigacin tendran que presentar un nmero alto de investigaciones dedicadas a solucionar los problemas cientfico-tcnicos ms urgentes del pas, es decir, las que ponen en marcha a las nuevas tecnologas. Pero las conclusiones de los analistas de polticas pblicas en materia de I+D e innovacin no son positivas: el CONICET ha alterado sus objetivos iniciales que apuntaban a apoyar las investigaciones de los docentes que trabajaban en las universidades y otras instituciones de investigacin cientfica

y tcnica. En la dcada del 70 se modific el estatuto de la carrera de investigador y los investigadores pasaron a tener una relacin directa con el CONICET, de modo que el organismo dedic ms a atender la carrera de investigador de su propio personal abandonando el objetivo de formacin de investigadores en relacin con las necesidades del pas. Abeledo afirma que hay una diferencia sustancial con lo que sucede en los pases industrializados:
La comparacin de los datos estadsticos con los de pases de mayor desarrollo muestra, adems de las diferencias cuantitativas de la cantidad de investigadores en relacin con la PEA (poblacin econmicamente activa), una distribucin con una muy baja proporcin de investigadores en empresas. En los sistemas de innovacin de mejor

desempeo, la proporcin de investigadores en empresas es comparable a la que se desempea en universidades y en institutos pblicos. Un aumento en las tasas de formacin de nuevos investigadores debera estar acompaado por un aumento en la apertura de nuevas plazas, especialmente en las empresas privadas. (Abeledo, 2006b)

Albornoz, por su parte, en su artculo "Universidad y nuevas tecnologas" atribuye a la Universidad la funcin de "crear, atesorar y difundir el conocimiento cientfico, tanto a travs de la formacin de profesionales, como de la prestacin de servicios a la sociedad" (Albornoz, 1990b. 263). Dicho en otras palabras, debe asumir en todos los niveles el fenmeno de las nuevas tecnologas. Pero este ideal es por el momento difcil de alcanzar para los pases

latinoamericanos: el cambio tecnolgico y la respectiva adaptacin del sistema educativo comenzaron en los pases industrializados y desde all se siguen produciendo ondas expansivas de innovaciones que abren con los pases de la periferia brechas tecnolgicas y problemas en la divisin internacional del trabajo. Otra consecuencia para los pases en vas de desarrollo por esta falta de protagonismo en la produccin de tecnologas es la dificultad para constituirse en culturas tecnolgicas; existe ms bien un dualismo tecnolgico provocado por la presencia de empresas de tecnologa de punta, por lo general pertenecientes a empresas

multinacionales, y otras retrasadas y obsoletas. Tambin en este orden pueden mencionarse la infrautilizacin de la capacidad productiva, la falta de incentivos en la inversin local y la ausencia de alternativas tecnolgicas viables. La Universidad refleja estas tensiones polticas, econmicas y sociales dando lugar a falsas dicotomas como "masividad" versus "excelencia", o a una situacin de incapacidad para adaptar sus estructuras a la incertidumbre del cambio tecnolgico (qu carreras deben priorizarse, cul es la duracin real de las carreras?). La Universidad de Buenos Aires es la institucin argentina que, pese a sus

deficiencias estructurales, cuenta con una importante tradicin de investigacin creativa en ciencia, con la mayor cantidad de investigadores y con el prestigio conferido por los Premios Nobel a sus graduados (B. Houssay, medicina, 1947; L. F. Leloir, qumica, 1970 y C. Milstein, medicina, 1984). No ha avanzado, sin embargo, en relacin con las urgencias planteadas por las nuevas tecnologas. Para alcanzar el nivel adecuado, Albornoz entiende que es necesario entender la investigacin bsica como investigacin estratgica, de acuerdo con la proximidad que hoy tiene la ciencia bsica con la industria. En el momento en que Albornoz publica su texto, existe

una escasa cantidad de investigadores dedicados a disciplinas estratgicas bsicas (biotecnologa, biologa celular y molecular, microbiologa y gentica) o a las aplicadas (microbiologa aplicada, inmunologa, etc.). Ha cambiado esta situacin en la actualidad? Como dijimos antes, ha cambiado en los proyectos pero no en los hechos, o no lo suficiente. Un caso ilustrativo: en 2007 el Ministerio de Economa lanz el Plan Productivo Nacional. En l se destaca un concepto clave para la cuestin del desarrollo nacional que estamos examinando: el concepto de competitividad. Transcribimos las afirmaciones de Juan C. Del Bello y C. Abeledo sobre el

significado de este concepto:


La competitividad es entendida como el incremento sistemtico del nivel de productividad de la economa y la transformacin de la estructura productiva, a nivel de sectores, cadenas de valor y conglomerados productivos, que combinen un alto grado de innovacin y que no sean islas de modernidad. El Plan afirma: "El reto de la poltica productiva para la Argentina es, entonces, promover un cambio que transforme la estructura productiva a favor de actividades que incorporen ms innovacin y que contribuyan al desarrollo de un tejido productivo ms denso, en forma consistente con los ejes rectores de aumento de las exportaciones, la inversin y la competitividad." En cuanto a los lineamientos sectoriales se promueven las actividades de "alto impacto potencial": biotecnologa, nanotecnologa y energas renovables, los "sectores innovadores": agroalimentos, farmo y agroqumica, TICs, tecnologa nuclear, aeroespacial y satelital, bienes de capital y servicios de alto valor agregado (turismo, industrias culturales, diseo, etc.) y los

"sectores tradicionales". El documento explicita la necesidad de la "coordinacin interministerial", especficamente del Plan Estratgico Nacional de Ciencia, Tecnologa e Innovacin, el Plan Estratgico Territorial y el Plan Nacional de Inversin Pblica. (Del Bello y Abeledo, 2007)

Lamentablemente, si bien la finalidad que el plan enuncia es la correcta, falla en la especificacin de cuestiones bsicas que debera contemplar. No establece, por ejemplo, "la institucionalidad necesaria para asegurar una buena gobernanza del Plan", es decir, no indica mediante qu mecanismos se van a relacionar las actividades de investigacin con el sector productivo y con el de financiamiento. Como hemos sealado a propsito del funcionamiento del CONICET, los

institutos de investigacin no se han adecuado a los lineamientos establecidos por los planes y proyectos gubernamentales. Dicen los autores:
[Los organismos estatales] Padecen de baja autonoma respecto de la administracin central, cuentan con rgidos sistemas de personal e inflexibles regmenes de remuneraciones, no tienen flexibilidad y agilidad para generar recursos propios y disponer de ellos, as como de su patrimonio. En fin, son instituciones pensadas para el enfoque superado de la "transferencia de tecnologa", que supone una situacin de generacin y oferta de tecnologas por parte del Estado, y un sector empresarial pasivo "demandante". El enfoque de los sistemas nacionales de innovacin ha superado esa visin, pero las instituciones continan funcionando con el viejo enfoque. Los "involucrados/actores" (stakeholders) no tiene participacin en los gobiernos de las instituciones, o la participacin es de naturaleza corporativa (representantes del sector privado en el Directorio del CONICET sin

empresas, por ejemplo). Mientras que otros pases de la regin han reestructurado instituciones o generado nuevas, con un enfoque basado en la asociatividad y en el desarrollo de redes, los institutos estatales argentinos permanecen congelados en el tiempo. (Del Bello y Abeledo, 2007)

Otra condicin que puede favorecer el crecimiento en materia de investigacin, segn Albornoz y los dems autores que estamos viendo, es la integracin de la investigacin con la estructura docente, es decir, integrar la investigacin en la Universidad y proyectarla realmente en los planes de estudio. Hacer esto significara adecuar la Universidad, institucin rgida por naturaleza, al dinamismo del cambio tecnolgico, es decir, tornarla flexible y cambiante, significa pedirle que "no

ensee tanto conocimientos objetivos, como que entrene para la discusin de alternativas, opciones, riesgos y ganancias potenciales", que reconozca que "el cmo que transfiere el conocimiento se ha convertido en algo ms importante que el propio conocimiento transferido" (Albornoz, 1990b. 271). Conclusiones Hasta 1990, la Argentina, y Amrica Latina en general, presentaban un atraso radical respecto de los pases industrializados en materia de polticas educativas orientadas al desarrollo y el crecimiento socio-econmico. Durante la dcada de los 90 se comienza el intento de recuperar el terreno perdido

pero con polticas equivocadas, tal como lo sealan Chudnovsky y Lpez (1996), por eso no se produjo el cambio estructural esperado. En la dcada siguiente, que llega a nuestros das, se intenta dar nuevo impulso a la investigacin cientfica y tecnolgica orientada a la innovacin, ya no dependiendo de la inversin extranjera como en la dcada anterior, sino fortaleciendo la infraestructura existente mediante la creacin de organismos adecuados y planes conceptual y tcnicamente a la altura de las exigencias internacionales. El contenido del Proyecto de Plan Nacional de Ciencia, Tecnologa e Innovacin Productiva de 2003, 2004 y 2005,

elaborado por la Secretara de Ciencia, Tecnologa e Innovacin Productiva, corrobora lo que acabamos de decir. Sin embargo, los cambios que tales propuestas de avanzada introducen no se han concretado en las acciones correspondientes. Como se ha expresado a lo largo del presente documento, la inercia de los organismos de investigacin, el academicismo universitario nacional, la desvinculacin de ambos con las necesidades sociales y las empresas, a lo que se suma la no intervencin del Estado en estas cuestiones, impiden realizar las modificaciones necesarias. Para finalizar, se transcribe una reflexin de Mario Albornoz que reitera

todo lo dicho hasta aqu de forma muy contundente:


Hoy, en cierta medida, en ciencia y tecnologa vivimos del capital acumulado en las dcadas de los cincuenta y los sesenta, ya que ellas fueron prdigas en esfuerzos y resultados. En aquellos aos se consolid la capacidad cientfica del pas, se crearon sus principales instituciones (como el CONICET, el INTA, el INTI, la CNEA) y las universidades nacionales alcanzaron un alto nivel en materia de investigacin y se constituyeron en los principales ncleos impulsores de la investigacin cientfica.

El panorama actual es un reflejo empobrecido del pasado esplendor. Buenos ejemplos actuales, como los de la empresa INVAP (que es una sociedad del Estado perteneciente a Ro Negro) y la Comisin Nacional de Actividades Espaciales (CONAE), son excelentes casos de capacidades tecnolgicas,

aunque lamentablemente aisladas, en un escenario con un perfil productivo de escaso valor agregado. El modelo neoliberal, pero tambin la incapacidad puesta de manifiesto por los sectores ms progresistas para generar un nuevo pensamiento movilizador, carece de motivos para formular polticas que tiendan a aprovechar al mximo las capacidades disponibles. De este modo, un pas que dcadas atrs pudo producir premios Nobel y desarrollar tecnologa propia, hoy no puede sostener el esfuerzo cientficotecnolgico que requiere el mundo de fin de siglo (Albornoz, 2003. 2). Bibliografa Abeledo, C. (2004a). La investigacin

en la Universidad: Creacin de conocimiento o desarrollo tecnolgico? En Universidad, Sociedad y Produccin. (J. C. Pugliese, Ed.). Buenos Aires. Abeledo, C. (2006b). Las necesidades de recursos humanos para el desarrollo del sistema nacional de innovacin. Albornoz, M. (1990a). Consideraciones histricas sobre la poltica cientfica y tecnolgica en la Argentina. En Albornoz, M. y Kreimer, P. Ciencia y tecnologa: estrategias y polticas de largo plazo. Buenos Aires: Eudeba. Albornoz, M. (1990b). Universidad y nuevas tecnologas. En Albornoz, M. y Kreimer, P. Albornoz, M. Ciencia y tecnologa en

la Argentina: capacidades sin sustento, 2003. <http://www.escenariosalternativos.org> Bunge, M. (1997). Ciencia, tcnica y desarrollo. Buenos Aires: Sudamericana. Chudnovsky, D. (1999). Polticas de ciencia y tecnologa y el Sistema Nacional de Innovacin en la Argentina. Revista de la CEPAL. N 67. Chudnovsky, D. y Lpez, A. (1996). Poltica tecnolgica en la Argentina: hay algo ms que laissez faire? Redes, revista de estudios sociales de la ciencia (vol. III). Universidad Nacional de Quilmes. Ciapuscio, H. (1990). Formacin intensiva de recursos humanos: una

necesidad imperiosa. En Albornoz, M. y Kraimer, P. Ciapuscio, H. (1994). El fuego de Prometeo: tecnologa y sociedad. Buenos Aires: Eudeba. Del Bello, J. C. y Abeledo, C. (julio de 2007). Reflexiones sobre cuestiones pendientes de la Agenda de Poltica Pblica en Ciencia, Tecnologa e Innovacin de Argentina. Primer Congreso Argentino sobre Estudios Sociales de la Ciencia y la Tecnologa. Universidad Nacional de Quilmes. Yfera, E. P. (1994). Introduccin a la investigacin cientfica y tencnolgica. Madrid: Alianza.

LA RESPONSABILIDAD SOCIAL DEL CIENTFICO. CRTICA DE LA TEORA SOBRE LA NATURALIDAD VALORATIVA DE LA CIENCIA Y DE LA ACTIVIDAD CIENTFICA. POLMICA ENTRE CIENTIFICISMO Y ANTICIENTIFICISMO
Mara Beatriz Contratti

Introduccin La relacin entre tica y ciencia deriva principalmente de la imperiosa necesidad de la sociedad contempornea de poner en orden o contener los resultados de las investigaciones

cientficas que se hacen tangibles para la mayor parte de los habitantes del planeta a travs de la tecnologa. Desde luego, la tica no es el nico modo a travs del cual puede llevarse a cabo esa tarea de ordenamiento. Tambin lo hacen la jurisprudencia, la poltica y la religin, por ejemplo. Pero el fenmeno de la ciencia y tecnologa ha desbordado las posibilidades de esos tres mbitos que se han ocupado tradicionalmente de generar o mantener orden social. Por qu esto parece ser as, se ir viendo a medida que se avance en el desarrollo de los temas. Una de las primeras tareas que se abordarn aqu, entonces, ser la de esbozar brevemente los rasgos de la

sociedad actual que, precisamente al estar caracterizada por la impronta de la ciencia y la tecnologa, se suele denominar sociedad del conocimiento o sociedad de la informacin. En segundo lugar, se har una breve sntesis de lo que es la tica, en tanto marco normativo de la conducta humana y en tanto disciplina filosfica, para referirnos despus a la tica aplicada, terreno al cual pertenece en parte el tema de este texto. Estas nociones permitirn incursionar en la cuestin especfica que nos ocupa: los aspectos ticos de la ciencia y/o de la tecnologa. Sobre todo, se intentar hacer hincapi en el modo de reflexin que opera en este campo, con el propsito de poner

en evidencia los problemas o cuestionamientos que ms comnmente se formulan a la ciencia y la tecnologa desde diversos sectores sociales. Se ver que es el concepto de responsabilidad el que predomina en los juicios ticos que tienen por objeto a la tecnociencia. Una vez aclaradas estas cuestiones bsicas preliminares, es posible examinar uno de los aspectos de la tica de la ciencia: la investigacin cientfica. En sta se pone de relieve la conducta del investigador a la luz de la correccin o incorreccin de sus acciones en relacin con la produccin de conocimiento. Como todo trabajo o profesin, la tarea del investigador debe

ajustarse a ciertos estndares establecidos, algunos generales y otros peculiares a su campo, para que el resultado de su tarea sea el adecuado. Se analizarn algunos de esos estndares o reglas de conducta. Pronto se ver que la tica de la ciencia no puede reducirse a juzgar la tarea del cientfico como si de eso slo dependiera la obtencin de resultados confiables de las investigaciones, entendiendo por resultado confiable teoras ciertas y objetivas o altamente probables. La conducta del cientfico ajustada a normas ticas es imprescindible en una investigacin seria, pero el problema de la tica de la ciencia es ms complejo y difcil de

tratar. Incluye el anlisis y crtica del conocimiento mismo surgido de las investigaciones, aun de aquellas que respetan todos los protocolos requeridos por la labor cientfica. El anlisis epistmico de la investigacin y las teoras cientficas mostrar que la pretensin de obtener conocimiento altamente confiable, objetivo, es una empresa mal encaminada. El conocimiento, desde este anlisis, resulta ser ms bien "construido" que "obtenido". Las conclusiones de esta forma de ver el conocimiento de la ciencia tienen importantes implicancias en el tema que nos ocupa. Esto lleva, finalmente, a la distincin entre cientificismo y anticientificismo,

trminos estos desde los que se ha planteado la polmica entre dos posturas opuestas: quienes piensan que la ciencia es neutral respecto a valores, es decir, es conocimiento "obtenido" acerca del mundo por procedimientos confiables, no contaminado con intereses o tendencias personales, polticas o ideolgicas (cientificismo), y quienes creen que la ciencia es conocimiento "construido", lo que hace inevitable que todos esos sesgos e intereses confluyan en su produccin (anticientificismo). Es preciso aclarar que la postura cientificista, que proporciona una imagen de la ciencia como conocimiento imparcial y objetivo, tiene an, a pesar de que los recientes estudios sobre la

ciencia han mostrado que esa imagen es inadecuada, una influencia muy grande en las capas dirigentes de la sociedad, lo que conlleva una unin ntima entre ciencia y poder, o conocimiento y poder. Como cierre, se intentar brindar un panorama del mundo actual que recoja todos los aspectos de la relacin ciencia-tica que se han ido comentando. Se dejar claro que cuando se habla de ciencia se quiere decir en realidad "tecnociencia" en todos los casos, pues una y otra, a diferencia de lo que piensa el cientificista, son, en la prctica, difciles de distinguir y, a los efectos de considerar las consecuencias de la aplicacin del conocimiento cientfico y tecnolgico en el mundo

social y natural, son indistinguibles. Caracteres de la sociedad contempornea "Saber es poder", el famoso aforismo enunciado por el filsofo ingls Francis Bacon[37] en el siglo XVII, parece encontrar en la sociedad actual un significado pleno. Efectivamente, en la segunda mitad del siglo pasado se caracteriz a la sociedad contempornea como la "sociedad de la informacin" y "sociedad del conocimiento" justamente porque la informacin y el conocimiento constituyen los recursos fundamentales que nutren su cultura y resultan condicin de su mantenimiento y desarrollo. El alto valor adquirido por el saber en el presente se relaciona con

la difusin de las llamadas "tecnologas de la informacin", por un lado, y, por otro, con la preponderancia adquirida por el conocimiento cientfico sobre cualquier otro tipo de conocimiento. Las tecnologas de la informacin presiden el desarrollo tecnolgico en general puesto que toda la tcnica actual depende para su funcionamiento en forma directa o indirecta de la primera. A su vez, la tecnologa de la informacin ha sido posible gracias a la comprensin por parte de la ciencia de los mecanismos que permiten a los seres vivos en general interactuar con su medio, de los procesos cognitivos estudiados por la psicologa, de los conocimientos matemticos avanzados y

del inters en transformar los dispositivos mecnicos en automticos en beneficio de la produccin industrial.[38] Es fcil advertir que la complejidad y profusin del panorama que presenta la cultura cientfica y tecnolgica contempornea hace que los fenmenos como la ciencia y la tecnologa no se dejen apresar en definiciones o caracterizaciones simples. Una afirmacin como la siguiente: "a la ciencia slo le interesa ofrecer conocimiento del mundo", resulta demasiado estrecha si tomamos en cuenta las diversas motivaciones que impulsan a los cientficos a realizar su trabajo y no considerar que el

conocimiento sobre el mundo es algo tan problemtico y discutido que hace poco plausible dicho ofrecimiento. De igual modo, sostener, como es habitual, que la tecnologa tiene como objetivo no conocer el mundo, sino "slo transformarlo", es realizar un recorte sumamente arbitrario en la compleja red formada por propsitos, saberes, historia, relaciones sociales y otras cuestiones que son inherentes al proceso tecnolgico. Se hace necesario, pues, proporcionar una caracterizacin de la ciencia y la tecnologa ms ajustada a la diversidad y complejidad de su papel en el mundo contemporneo, y esta necesidad no se debe slo a razones de ndole

conceptual, sino sobre todo a esclarecer las relaciones entre la ciencia y la tecnologa con la sociedad. Es obvio que ellas han sido creadas y desarrolladas por el hombre para hacer cada vez ms confortable y satisfactoria su vida, pero este propsito natural y original parece desvirtuarse en la actualidad cuando la ciencia o la tecnologa, cada una por razones a veces muy diferentes y otras veces semejantes, se independizan de la sociedad a la que pertenecen y cobran un poder autnomo mediante el cual dejan de servir a la humanidad para convertirse ms bien en sus amos despticos. Cmo y por qu ha sucedido esto est ya insinuado en la Introduccin y se profundizar en el

resto del documento. Esta cuestin del poder adquirido por la ciencia y la tecnologa sobre el resto de las fuerzas sociales, poder que en ltima instancia tiene una raz poltica, ha sido denunciada y analizada por muchos especialistas en cuestiones sociopolticas, filsofos de la ciencia y de la tecnologa, cientficos, tecnlogos y analistas provenientes de muchas otras reas. En este artculo se destacar el aspecto tico de esas relaciones. Es sta una cuestin que requiere realizar distinciones y precisiones porque el rtulo "los problemas sociales y ticos de la ciencia y la tecnologa" admite diversas interpretaciones y enfoques que son muy discutidas o, al menos,

discutibles. La tica y sus relaciones con la ciencia y la tecnologa Antes de hablar sobre las relaciones entre ciencia, tecnologa y sociedad es preciso aclarar qu se entiende por tica en general y especialmente en el marco de los estudios sociales de la ciencia. En trminos muy abarcadores, la tica es una disciplina filosfica que se ocupa de la justificacin racional de las normas morales que regulan la conducta humana individual y social. Las normas morales nos dicen qu es correcto hacer y qu no lo es, tanto en relacin con nosotros mismos como en relacin con los dems. Dichas normas se expresan en juicios morales que, si se consideran en

conjunto, suelen revelar aspectos importantes de la idiosincrasia de grupos o pueblos, como podra ser el ideal de vida buena que una comunidad comparte. En relacin con la accin humana, entonces, en dichos juicios se articulan conceptos como bueno, malo, correcto, incorrecto, justo, injusto, deber, obligacin, prohibicin y otros que aluden a las creencias que tiene la gente sobre la conducta propia y ajena y lo que est corporizado en las instituciones sobre lo que es moralmente aceptable. El concepto vinculado con las cuestiones ticas de la ciencia y la tecnologa que ms peso tiene actualmente, dada la influencia que tienen la ciencia y la tecnologa sobre el

destino de las sociedades y los hombres, es el concepto de responsabilidad. Como sucede con la gran mayora de los conceptos filosficos, el significado de la nocin de responsabilidad es problemtico. Aqu se tomarn en cuenta slo aquellos sentidos que sean relevantes a las cuestiones de la tica de la ciencia. En primer lugar, habra que distinguir un sentido causal y un sentido normativo de responsabilidad: la falta de lluvias es causa de la sequa y, por lo tanto, de la ruina de las cosechas, pero la falta de lluvias no es moralmente responsable de esa catstrofe. Pero si alguien arroj alguna sustancia perjudicial a los cultivos, entonces ese alguien s es responsable moral y/o

legalmente de su ruina.[39] Parece, pues, que un elemento de intencionalidad es necesario para adjudicar responsabilidad por la consecuencia de una accin, por eso en el lenguaje del derecho se dice que quien causa dao a otro tiene la obligacin de "responder" o es "responsable" o tiene "responsabilidad" por el dao sufrido por la otra persona. Desde luego, es preciso realizar posteriores especificaciones respecto de las condiciones psicofsicas que debe reunir una persona para ser considerada responsable de sus actos, pero sta es una cuestin que urge dilucidar ms desde el sentido jurdico que desde el moral. Es necesario adems aclarar que

estos dos tipos de evaluaciones del comportamiento humano no siempre coinciden sobre las distinciones de los actos (correctos o incorrectos) y su correspondiente nocin de responsabilidad. Un ejemplo: desde el punto de vista jurdico no es imputable mentir a un allegado por piedad o conmiseracin, pero desde un punto de vista moral puede crear un dilema, y desde la perspectiva tico-filosfica, es posible plantearse si la regla que dice "no mentir" admite excepciones. De acuerdo con estas consideraciones, cabra preguntarse por qu y en qu medida se puede hablar de la responsabilidad moral de la ciencia y la tcnica. Si, como se ha dicho, los

sujetos de la responsabilidad son las personas o los individuos, son los cientficos o tecnlogos quienes pueden ser imputados en ese sentido. Pero la responsabilidad moral no es slo atribuible a los individuos, tambin es lcito plantear la existencia de la responsabilidad colectiva. Justamente, los problemas socio-ambientales ocasionados por la ciencia y la tecnologa actual han llevado a considerar el concepto de responsabilidad ms all de los individuos que intervienen circunstancialmente en la produccin de un fenmeno dado; los desastres ecolgicos producidos por derrame de petrleo, las transfusiones realizadas con sangre

contaminada con el VIH y muchos otros casos conocidos ilustran la cuestin de la responsabilidad colectiva. Es indudable que el concepto plantea muchos interrogantes difciles de solucionar,[40] pero su empleo viene exigido por las peculiares situaciones conflictivas de la vida contempornea. Por otra parte, a raz de los diversos perjuicios en la salud y el medioambiente derivados de la implementacin de las teoras cientficas y tecnolgicas, es que ha nacido en los ltimos tiempos una disciplina filosfica denominada "tica aplicada". En principio, la tica aplicada sera, valga la redundancia, la aplicacin de la tica terica, es decir, aquella que analiza

filosficamente las normas morales, a casos particulares. Cmo se realiza tal aplicacin, o sea, los mtodos y procedimientos, si los hay, que permitiran "bajar" al campo de los hechos concretos los principios tericos, es materia de ardua discusin. Lo que interesa en este texto es mostrar cmo el uso generalizado de la tica aplicada est sealando la ntima relacin que hay entre ciencia, tecnologa y tica, puesto que son esos dos primeros fenmenos tpicamente contemporneos los que han elevado la necesidad de apelar a la tica para resguardar los derechos a la vida de los habitantes de la Tierra expuestos a diversos peligros por el accionar de

aquellas. Precisamente, el tema tratado en este artculo se encuadrara en una reflexin general sobre cuestiones de tica aplicada a la ciencia y la tecnologa en tanto productores de conocimiento y las prcticas que ello implica. Existen dentro del campo de la tica aplicada otras disciplinas que se ocupan de los problemas especficos ocasionados por la ciencia y la tecnologa en ciertos sectores del mundo natural y social. As se puede hablar de "tica del medioambiente", "tica mdica" o "biotica", "tica aplicada al tratamiento de los animales (y plantas)", etc. Todos estos mbitos tienen a la investigacin cientfica o al conocimiento cientfico y tecnolgico

como marco de referencia, pero no cuestionan directamente el conocimiento mismo o el accionar de la ciencia y la tecnologa como instituciones generadoras de saber, como se hace aqu, sino sobre todo a los conflictos sociales y morales que provoca su aplicacin. Es preciso aclarar, por otro lado, que ambas perspectivas, la que considera las consecuencias de la aplicacin del conocimiento y la que se interesa por el conocimiento mismo, interaccionan en forma continua en los estudios de tica aplicada. Otra distincin aplicable a la cuestin de la responsabilidad moral de la ciencia y la tecnologa (a las personas o al colectivo) es la establecida entre

responsabilidad retrospectiva y responsabilidad prospectiva. La primera se refiere a los resultados de las acciones ya realizadas, por ejemplo, el haber permitido que se arrojaran los residuos industriales en el Riachuelo, y la segunda, a la exigencia de prever los posibles resultados de las acciones. El segundo sentido, del que se pueden encontrar infinidad de ejemplos, tiene un inters especial pues plantea a la ciencia y la tecnologa la exigencia de conservar el planeta en buenas condiciones para las generaciones venideras. Sin embargo, no todos acuerdan en que el "conocimiento" cientfico y tecnolgico pueda ser en parte responsable de los daos

medioambientales, mdicos, etc. que ocurren en el mundo actual. Desde un punto de vista por dems optimista, se suele afirmar que la ciencia y la tecnologa estn ms all de posibles objeciones de naturaleza tica porque son buenas en s mismas, lo cual quedara probado por las mltiples formas de progreso con que han beneficiado a la humanidad. Efectivamente, da a da sorprenden al mundo nuevos artefactos y procesos innovadores que salvan vidas, evitan los efectos, o al menos los atenan, de las catstrofes naturales, incrementan la posibilidad de la comunicacin entre las personas y, en fin, proporcionan todos los medios para lograr el bienestar

material, psicolgico y espiritual que necesitan los seres humanos. Y esto es, sin duda, cierto. Pero desde otras perspectivas se muestran los efectos adversos e irreversibles que acompaan a los beneficios. Muchas veces las mismas tcnicas y productos que permiten salvar vidas ocasionan otros malestares y dolencias, y no pocas veces esos malestares y dolencias terminan en daos irreversibles: la represa que proporciona la energa elctrica tan necesaria en un mundo superpoblado tambin provoca la prdida de valiosas especies naturales y altera el microclima de la regin donde se construy, perturbando de ese modo el sabiamente aceptado modo de vida de las

poblaciones cercanas, al punto de causar su extincin por la ruptura del equilibrio ecolgico, la comodidad que brindan los medios de transporte modernos se ve disminuida en su valor por la contaminacin que produce, y como stos hay gran cantidad de otros ejemplos que muestran los efectos nocivos que en forma directa o potencial puede acarrear la implementacin de las distintas tecnologas. Un caso que ha resultado paradigmtico en aos anteriores en los estudios de ciencia y tecnologa es el de la energa nuclear. En este mbito se ve claramente el rostro dual de la ciencia y la tecnologa. Si fuera posible dominar los procesos que rigen la desintegracin o

fisin del tomo de modo que su utilizacin fuese segura, los beneficios seran cuantiosos. Pero la leccin que dej la catstrofe de Chernobyl[41] fue suficiente para saber que est lejos de alcanzarse este ideal, y que los peligros que entraa la manipulacin de la energa nuclear por ahora se consideran inevitables, como tambin lo atestigua el problema de los residuos nucleares. La decisin de poner en marcha una central nuclear exige una evaluacin rigurosa de los riesgos que implica. Pero, es posible realizar una evaluacin tal? Hay algn procedimiento establecido desde el cual se pueda determinar con una precisin aceptable el riesgo potencial que supone el funcionamiento

de una planta nuclear? Las evaluaciones de riesgo se realizan en general por procedimientos estadsticos, pero como la estadstica opera sobre variables preestablecidas es bueno preguntarse hasta qu punto ese mtodo resulta totalmente satisfactorio para los habitantes de una regin donde se encuentra una planta nuclear. La estadstica nos da alguna informacin sobre la correlacin entre ciertas variables seleccionadas por el evaluador, pero bien analizadas las cuestiones referidas a las evaluaciones de riesgo van mucho ms all de los porcentajes que proporciona la estadstica. Lo que la gente quiere saber para aceptar o tener cierta seguridad

sobre una determinada tecnologa, tiene menos relacin con nmeros que con modos de vida, esperanzas, temores, creencias enraizadas en la tradicin y perspectivas sobre el futuro. La consideracin de estos factores por parte de quienes dirigen el aspecto poltico y social de la ciencia y la tecnologa significara adoptar un modelo de evaluacin en el que se considere el riesgo, no como algo abstracto y determinable en forma cuantitativa o tcnica solamente, sino pensndolo desde una dimensin integralmente humana. En otras palabras, este modo de encarar el control de la ciencia y la tecnologa implica asumir sobre todo una perspectiva tica y no

meramente tcnica en dicho control. Silvio O. Funtowics y Jerome R. Ravetz, filsofos de la ciencia contemporneos, argentino e ingls respectivamente, hablan en este sentido de una "comunidad de evaluadores extendida" que es requerida cuando la evaluacin, como en el caso de la complejidad del proyecto de construccin de una represa, "no depende de manera esencial de la diversidad de disciplinas cientficas relevantes, sino que, ms bien, consiste en la multiplicidad de perspectivas legtimas desde las que es posible contemplar el problema en su totalidad" (1997. 156). La posibilidad de concretar este tipo de evaluaciones se sostiene, segn los autores, en lo

siguiente: "En las sociedades modernas, incluyendo tanto las ricas como las pobres, hay un gran nmero de gente comn que puede leer, escribir, votar y debatir. La democratizacin de la vida poltica es algo usual hoy por hoy; sus riesgos se aceptan como un pequeo precio a pagar a cambio de sus beneficios" (1997. 159). Estos beneficios resultan de "el logro de un sistema que, a pesar de sus ineficacias, es el medio ms eficaz de evitar desastres originados por el prolongado silenciamiento de las crticas. Experiencias recientes han mostrado que tal presencia crtica es tan importante para nuestros problemas ambientales y tecnolgicos

como lo es para la sociedad" (1997. 159). La tica de la investigacin cientfica Es preciso convenir, por lo pronto, que el proceso de produccin de conocimiento cientfico y tecnolgico involucra cuestiones ticas que comprometen al cientfico en tanto ser humano que tiene intereses, porta valores personales, incurre en parcialidades y errores y otros rasgos conductuales comunes a todos. Algunas normas de conducta exigidas a los cientficos, en tanto individuos, son las exigibles a cualquier profesional o trabajador, como la honestidad, la veracidad, etctera, y otras estn asociadas a la puntillosa observacin de

los pasos del mtodo cientfico y las condiciones de su aplicacin. A raz de este nexo entre regla metodolgica y regla moral, que sera exclusivo del mbito de la ciencia, muchos piensan que ms que hablar de cdigos de conducta o normas a las que el cientfico debe ajustarse, es suficiente poner de relieve los valores que son propios de las actividades inherentes a la investigacin. Estos valores seran de diversos tipos: cognitivos, metodolgicos y prcticos o morales, y el tenerlos en cuenta y darles cumplimiento en el proceso de investigacin dara forma por s mismo a la responsabilidad tica del investigador. Otros, en cambio, piensan

que slo los valores o reglas cognitivas y metodolgicas son de posible consideracin en la ciencia, puesto que sta se ocupa de conocer el mundo nicamente y no realiza acciones que apunten a cambiar ese mundo.[42] Los valores prcticos o morales quedaran fuera de toda consideracin. Pero esta perspectiva resulta bastante simplificadora y supone la aceptacin de criterios que han sido muy objetados, como la distincin tajante entre ciencia y tecnologa. Es, pues, pertinente hablar de una tica de la investigacin cientfica en la que se entienda al investigador cientfico como una "persona" que investiga, sujeto por lo tanto a una constelacin de restricciones

de orden moral que quedara formulada en cdigos deontolgicos como en cualquier otra actividad profesional. En un documento de la National Academy of Science de Estados Unidos: On Being A Scientist: Responsible Conduct in Research,[43] documento redactado y supervisado por academias y comits cientficos norteamericanos, se examina el problema de la investigacin cientfica en la actualidad y los errores que se pueden cometer en la investigacin daando gravemente no slo el resultado de la misma, sino tambin la confianza que la sociedad puede otorgar a la ciencia. Una primera cosa que los autores del documento aclaran es que cuando se

habla del cientfico, no slo se alude a un individuo particular, sino tambin a una comunidad de cientficos, sin la cual la ciencia actual en su complejidad y entrecruzamientos de campos y disciplinas no sera posible. Esta comunidad cientfica tiene que enfrentar en forma constante numerosos problemas: el crecimiento de la ciencia ha sido muy grande y rpido, y esto ha creado dificultades con el financiamiento, con los medios para alcanzar las metas y nuevas y problemticas relaciones con la sociedad de la que forma parte. El peso que la ciencia y la tecnologa tienen sobre la sociedad contempornea ha despertado el inters de los no

cientficos en evaluar las afirmaciones de los cientficos, ya que su bienestar hoy depende en alto grado de la actividad de los ltimos. Entre las cuestiones que ataen a la responsabilidad personal del investigador relacionadas con el cumplimiento de las normas institucionales explcitas o implcitas, o de las que se desprenden de las prescripciones metodolgicas, se encuentran las siguientes: Errores cometidos en el transcurso de la investigacin: pueden ser "errores honestos" que se producen inevitablemente por razones circunstan-ciales (falta de tiempo o

recursos financieros); en la medida en que sean detectados en publicaciones o artculos, deben ser reconocidos por su autor, lo cual seguramente ser considerado con tolerancia por la comunidad cientfica. Otro tipo de error posible se puede deber a la "negligencia" del investigador; el error surge por precipitacin, falta de cuidado, inatencin, etc. En este caso, la reaccin de los pares ser ms severa, y las consecuencias que pueden tener los errores debidos a la negligencia son ms difciles de revertir que los primeros. Pero los errores ms graves son los que involucran

"engao" por parte del cientfico: inventar datos o resultados o falsificarlos y el plagio constituyen los ms conocidos. Segn los autores del documento On Being a Scientist, stos no slo socavan el progreso de la ciencia, sino el conjunto completo de valores sobre los cuales se sustenta la misma. Las consecuencias de tales acciones no es asunto interno solamente, pues su correccin no se dirime exclusivamente en el mbito de la comunidad cientfica, sino tambin en tribunales externos. La adjudicacin de crditos o recompensas:[44] segn este documento de la National Academy

of Science, en el sistema estndar de la ciencia la adjudicacin del crdito por el mrito de los trabajos realizados es reconocido en tres lugares: en la lista de autores, en el reconocimiento por las contribuciones de otros y en la lista o referencias de las citas. En estos lugares pueden surgir conflictos por una inadecuada atribucin del crdito o reconocimiento. Varias son las consecuencias cuando las citas no se realizan como es debido: perjudican tanto al investigador en su prestigio y su carrera como al sistema de publicaciones cientficas. Los nombres de los

autores de un proyecto de investigacin, el primero de los lugares mencionados, debe preestablecerse al desarrollo de la investigacin, para no crear conflictos posteriores. La contribucin de cada miembro suele determinarse de acuerdo con el rango que ostenta: graduado, estudiante, etc., pero una cuestin importante es que estn mencionados "todos los colaboradores". El tratamiento de las tcnicas experimentales: las observaciones realizadas por los cientficos a travs de las tcnicas experimentales estandarizadas

permite la verificacin independiente de los datos. En la medida en que el cientfico usa esas tcnicas, los resultados de su trabajo pueden ser reproducidos por otros cientficos y esto favorece su confiabilidad. Pero tanto los mtodos como el conocimiento obtenido mediante ellos no son infalibles, deben someterse a continua revisin y control, de lo contrario se puede caer en fatales errores. El escepticismo es por esta razn una caracterstica de la ciencia presente en todos los momentos de la investigacin. La publicacin y la apertura: la

ciencia no es una experiencia individual sino conocimiento compartido basado en la comprensin comn de algunos aspectos del mundo fsico o social. Para garantizar la confiabilidad de este conocimiento la ciencia cuenta con ciertas convenciones sobre la forma en que deben difundirse los resultados de la investigacin. La principal es la publicacin de esos resultados. Se pueden originar conflictos sobre la prioridad en los descubrimientos, ya que el primero que publica es el que obtiene el crdito, no el primero en descubrir algo nuevo. Quien publica es reconocido por sus pares a travs

de la cita en otras publicaciones que usan sus hallazgos, pero debe cumplir una condicin bsica: su trabajo tiene que haber pasado el control de calidad efectuado por sus colegas. De lo contrario, puede caer en errores o dar lugar a interpretaciones equivocadas. Conflictos de intereses: el cientfico tiene que emplear su propio juicio para interpretar datos, para encontrar problemas de investigacin y cundo concluir un experimento. Para ello se necesita desarrollar ciertas habilidades que se pueden aprender, pero no totalmente. Cuenta mucho la creatividad y la intuicin, que son

condiciones personales. En estos casos los juicios empleados por los cientficos comportan valores. Algunos de estos son de naturaleza epistmica, como la consistencia, la precisin en las predicciones, la capacidad de unificacin de observaciones diferentes, la simplicidad y la elegancia. Otros son valores personales, religiosos, filosficos, culturales, polticos o econmicos, que forman el juicio del cientfico de manera fundamental. Un punto importante desarrollado por los autores del Documento es el de la influencia positiva o negativa de los valores en el desempeo del cientfico en

tanto tal. Piensan que a veces favorecen la investigacin y otras la entorpecen; un ejemplo de lo primero es el deseo de realizar una buena investigacin, as como aceptar los estndares de honestidad y objetividad propios de la ciencia y un ejemplo de lo segundo es cuando desde el campo de la eugenesia (aplicacin de las leyes biolgicas de la herencia al perfeccionamiento de la especie humana), se usan las tcnicas de la ciencia para intentar mostrar la inferioridad de ciertas razas. Los autores creen que cuando la investigacin corre peligro de distorsin por la influencia de los

valores de los cientficos, los mecanismos correctivos de la ciencia, como el control emprico de las afirmaciones y el consenso de la comunidad cientfica, ponen lmites a esa posibilidad. Adems, la formacin del cientfico debe ser lo suficientemente amplia a fin de que pueda tomar conciencia de las suposiciones y creencias que subyacen a las elecciones y decisiones que debe enfrentar en su trabajo y as, en lo posible, neutralizarlos cuando afectan negativamente los objetivos de la ciencia. Esa formacin debe incluir el aprendizaje de cuestiones no cientficas, como las que

proporcionan las ciencias sociales, la religin y la tica. Las consideraciones que se acaban de formular se relacionan con dos cuestiones que posiblemente sean las que ms influyen en los problemas ticos de la ciencia: el aislamiento del cientfico del resto de la sociedad y la percepcin exclusivamente tcnica de los conflictos que suscita su actividad. Pero por s solas, estas consideraciones no agotan la cuestin de la responsabilidad de la ciencia, razn por la cual deben ser ajustadas y profundizadas. Si bien la tica de la investigacin tal como la expone la National Academy of

Science destaca aspectos importantes de la conducta responsable de los cientficos, es de lejos insuficiente para comprender los aspectos ticos de la ciencia. La razn de esta insuficiencia es que mide la adecuacin moral de la conducta del cientfico sobre el trasfondo de "la ciencia" como una institucin autosuficiente e indiscutiblemente certera en todos sus presupuestos. Bastara entonces con dar al cientfico una formacin amplia que le permita reflexionar sobre sus inclinaciones e intereses, como recomendaba antes el documento mencionado, para lograr un conocimiento objetivo y neutral respecto de valores ticos. Sin embargo, en

primera instancia, se puede decir que este enfoque es muy defectuoso porque supone que tal conocimiento es posible, logrando solamente la "deshumanizacin" del investigador. Si bien es cierto que es necesario despojar a la investigacin de elementos espurios, como ciertos intereses o inclinaciones, no es algo indiscutido que todos los intereses o inclinaciones sean negativos en la labor cientfica. Quienes sostienen a ultranza la idea de que el conocimiento cientfico es, y debe ser, formalmente racional, objetivo y ticamente neutral, es decir, no atravesado por intereses o valoraciones de ningn tipo, han sido denominados "cientificistas". Una de las estrategias

del anticientificista para desactivar las bases de la posicin cientificista es apelar al argumento de la naturaleza problemtica del conocimiento. Se ver a continuacin cules son las caractersticas de estos dos enfoques, sus proponentes, los problemas conceptuales, histricos y polticos que entraan, as como las alternativas de la disputa en la que estn involucrados. El cientificismo, el anticientificismo y la tica Se considera al cientificismo como un punto de vista sobre la naturaleza de la ciencia, la tecnologa y sus relaciones con el resto de la sociedad caracterizado por considerar a la ciencia, no slo independiente del resto

de la sociedad, sino tambin su rectora. La preeminencia de la misma sobre los dems mbitos sociales se supone derivada de su capacidad de obtener y formular un conocimiento de la realidad absolutamente racional y objetivo. Las teoras cientficas, como resultado de ese proceso altamente preciso y seguro, proporcionaran una descripcin y explicacin de los fenmenos reales totalmente cierta y confiable. En los estudios epistemolgicos sobre la ciencia, fueron las corrientes denominadas positivismo y empirismo lgico[45] las que trasmitieron esta imagen del conocimiento cientfico. Sin embargo, desde otras perspectivas epistemolgicas se fueron planteando

objeciones a esa imagen y sus implicancias, de modo que el positivismo o empirismo lgico fueron perdiendo vigencia y terminaron siendo superados por otras perspectivas que, lejos de idealizar el conocimiento cientfico, mostraron sus races histricas y humanas. Estos cambios en el enfoque sobre la naturaleza de la ciencia provinieron tambin desde otros estudios filosficos, desde la epistemologa, tales como la historia de la ciencia y la filosofa de la tecnologa, y desde otros mbitos no filosficos como la sociologa y la antropologa. Se puede decir que hoy da una caracterizacin ajustada de lo que es la ciencia no puede prescindir de alguno

de estos distintos puntos de vista. Los estudios sobre ciencia, tecnologa y sociedad, justamente, representan un enfoque integral del fenmeno tecnocientfico.[46] Es preciso aclarar, no obstante, que si bien el positivismo lgico ha sido reemplazado en su presentacin primitiva por otras concepciones que son opuestas en sus presupuestos fundamentales, los aportes que aquellos realizaron al anlisis de la ciencia fueron tan importantes y fecundos que constituyen la condicin necesaria de toda la reflexin posterior. El control emprico de las hiptesis y las cuestiones lgicas que ste implica, entre muchas otras, que fueron muy

elaboradas y estudiadas por los miembros de aquellas corrientes, deben ser tenidos en cuenta en cualquier enfoque epistemolgico o metodolgico que se precie de tal, aunque se discrepe con los positivistas sobre su rol y alcance. Continuando con el tema del cientificismo, se puede decir que uno de sus ms famosos exponentes es el fsico y filsofo argentino Mario Bunge (n. 1919) que se encuentra trabajando en Canad desde 1966. Desde su punto de vista, la ciencia es el nico modo de saber que proporciona una comprensin fundamentada de la realidad, por eso est totalmente justificado su papel central en la cultura contempornea.

Pero no solamente la ciencia predomina sobre cualquier otro aspecto del sistema cultural sino que la ciencia misma es un modelo de tica. Afirma: "La ciencia es un modelo de produccin con una modalidad tica bien precisa: no puede haber ciencia deshonesta, ciencia en bsqueda deliberada del error, o que eluda la crtica, o que suprima la verdad. La bsqueda de la verdad objetiva impone una recta conducta [...]" (Bunge, 1996. 54). Y ms an: "De aqu la posibilidad de moralizar por la ciencia; esta posibilidad puede realizarse si se adapta el cdigo de conducta deseable en la vida diaria al cdigo de conducta deseable en el campo de la ciencia" (Bunge, 1996.

56). Las palabras de Bunge son suficientemente elocuentes como para no dejar dudas sobre el modo de pensamiento caratulado como cientificista. Profundizando las crticas y objeciones ya sealadas recibidas por el cientificismo, se podra decir, en primer lugar, que la influencia que ha tenido la imagen de la ciencia construida por el Positivismo y sus defensores ha sido y es muy grande. Como sera absurdo pensar que los tericos fundadores del Positivismo, y otros filsofos que adhieren a esa corriente, hayan obrado de acuerdo con oscuros propsitos, resulta claro que es en el terreno poltico desde donde se ha logrado

instalar la idea de autonoma y "santidad" de la ciencia11 de modo de sacar partido de su poder potencial. Es preciso aclarar que se entiende aqu el trmino "poltica" en sentido amplio, referido descripcin de los lazos polticos, econmicos y culturales y las teoras cientficas. Ver Gonzlez Garca, M., Lpez Cerezo, J. y Lujn Lpez, J. (1996). 11 La expresin es de Vandana Shiva (1993). al modo en que una sociedad y sus instituciones gestionan sus recursos, proyectos, mbitos de poder, etc., y no significando las actividades partidarias del sistema poltico formal. Al hacer de la ciencia no una institucin ms en interaccin con otros sistemas sociales, sino un

santuario encerrado en una torre de marfil desde la cual slo se emite verdades indiscutidas, se ha impedido al resto de la sociedad participar en un dilogo con la comunidad cientfica sobre la conveniencia de proseguir investigaciones propuestas o de conservar teoras que se han mostrado perjudiciales, aun cuando las actividades de esta comunidad ponen en riesgo sus intereses y hasta su supervivencia. Habra que preguntarse, entonces, si el conocimiento cientfico contiene los caracteres que el cientificista dice que tiene y que justificaran su posicin. Si se encuentra que no los tiene, o son imposibles de definir claramente,

entonces la posicin anticientificista, cuya tesis es opuesta a la del cientificista en la medida en que considera a la ciencia como conocimiento construido, provisorio y negociable, histrica y culturalmente contextualizada y ticamente comprometida junto con la tecnologa, encontrara razones de peso para rechazar la posicin cientificista, que, como se ha dicho, pretende que el conocimiento cientfico tiene preponderancia sobre todo tipo de conocimiento y por eso es ajena a todo requerimiento de orden social y tico. En Ciencia, tecnologa y sociedad (Gonzlez Garca, Lpez Cerezo y Lujn Lpez. 1996) se ha sistematizado el

conjunto de objeciones epistmicas que comnmente se formula contra la idea de ciencia como conocimiento universalmente vlido, racional y objetivo que es la que sustenta la postura cientificista; esas objeciones son las siguientes: La fragilidad del conocimiento inductivo: remite al problema ya planteado por David Hume (filsofo britnico, 1711-1776) en el siglo XVIII, segn el cual ningn nmero finito de enunciados singulares puede justificar concluyentemente un enunciado universal. Ninguna de las dos versiones de la induccin, ni la verificacionista ni la confirmatoria,

se salva de la condena de este argumento. Las observaciones son limitadas y las enunciados de la ciencia son generales, abiertos, de modo que el conocimiento que brindan no es totalmente racional (en el sentido de tener un fundamento absoluto), sino slo "razonable", dependiente ms del acuerdo intersubjetivo que de la certeza objetiva. Tampoco la refutacin resulta un proceso concluyente, aunque el esquema lgico correspondiente lo sea, pues en ciencias fcticas las hiptesis no se ponen a prueba aisladamente sino en "red" con otras creencias e hiptesis auxiliares, lo cual hace

imposible determinar su falsedad. Tomas Kuhn (1922-1996), socilogo e historiador de la ciencia, asest un golpe definitivo a la concepcin del conocimiento emprico tradicional al introducir nociones impensables hasta ese momento en el hallazgo y evaluacin de ese conocimiento: a travs del concepto de paradigma postul que una amalgama de factores tecnolgicos, pedaggicos, psicolgicos, sociolgicos, considerados no epistmicos por la tradicin empirista, era la responsable de las teoras aceptadas. La historia interna de la ciencia se muestra as insuficiente

para explicar el modo en que la ciencia debe resolver sus problemas. Relativismos: a partir de Kuhn se desarrollan con intensidad los estudios sociolgicos de la ciencia. Con la entrada de la sociologa, la frontera entre ciencia y sociedad parece desdibujarse. La ciencia no es ya un dominio privilegiado donde se produce el conocimiento, sino que ste surge de las mismas fuentes que alimentan cualquier creencia comn. La calificacin de buena o mala ciencia ya no depende slo de criterios epistmicos sino tambin de intereses y convenciones sociales.

Michael Mulkay (n. 1936), socilogo ingls que trabaja actualmente en la Universidad de York, por ejemplo sostiene que la ciencia toma muchas veces del mbito cultural externo ciertos trminos o conceptos cuando sus recursos interpretativos fracasan o que tambin es receptora de las demandas de intereses de grupo, y esto le permite afirmar que "el contenido de la ciencia se afecta por factores sociales y culturales originados fuera de la ciencia" (Mulkay, 1994. 347). Por lo tanto, el valor cognitivo de una teora cientfica resultara en aras de su justificacin relativa a los

contextos particulares, o sea, a los criterios correspondientes a aquellos sectores externos, o bien carecera por completo de justificacin. La carga terica de la observacin: esta cuestin alude a que lo que se ve o percibe depende tanto de las impresiones sensibles como del conocimiento previo, las expectativas, los prejuicios y el estado interno general del observador. Este modo de concebir la observacin complica la explicacin del cambio terico (cuando una teora es sustituida por otra que le es incompatible) o bien la eleccin entre teoras rivales

incompatibles. El concepto de paradigma de Kuhn expresa en forma radical este problema, pues cientficos que trabajan bajo paradigmas diferentes tambin ven el mundo de modo diferente.[47] La cuestin de la carga terica profundiza el problema de la relatividad. Infradeterminacin: es un problema ocasionado por la posibilidad de establecer teoras alternativas incompatibles para explicar un mismo conjunto de fenmenos. Otra manera de expresar esto es afirmar que la evidencia emprica es insuficiente para determinar la solucin de un problema dado. La

infradeterminacin se relaciona con el problema de la carga terica y la inconmensurabilidad. Clausura metodolgica: a raz de los problemas anteriormente mencionados se produce un vaco epistmico entre la evidencia emprica y las teoras respectivas. Como no hay recursos de ese orden para llenar esa brecha se recurre a factores no epistmicos, que, segn los autores, pueden ser"-Factores tcnico-instrumentales, como tradiciones tcnicas y disponibilidades instrumentales, que canalizan el tipo de resultados obtenidos y la interpretacin de los mismos; -

Factores sociales, es decir, factores econmicos, profesionales, polticos o ideolgicos (el 'contexto social', en pocas palabras), que tienen un papel crucial en la interpretacin de los resultados tericos y la promocin selectiva de ciertas tecnologas" (Gonzlez Garca, Lpez Cerezo y Lujn Lpez, 1996. 46). De este modo, parece clara la relacin del contenido de las teoras con los factores externos considerados no epistmicos por la perspectiva tradicional. En lugar de considerar este modo de clausura como un

procedimiento espurio, se piensa que favorece la consecucin de los objetivos de la ciencia y disminuye la incertidumbre ocasionada por la infradeterminacin. Son importantes en la medida en que permiten tomar decisiones sobre asuntos de inters general. Este conjunto de restricciones al alcance epistmico de las afirmaciones del conocimiento cientfico es objeto de amplio consenso entre los estudiosos de la ciencia y ofrece, ms all de lo que pueda implicar desde el punto de vista estrictamente lgico o epistemolgico de las teoras, una imagen de la ciencia como una actividad humana ms, lejos de la idealizada perspectiva del

cientificista. Y esto no es un rasgo secundario, externo a la ciencia misma, sino una condicin necesaria en la produccin de su saber. Las cuestiones puntualizadas ms arriba estn apuntando a una concepcin del conocimiento cientfico como algo construido, no dado, sus resultados son consensuados, no impuestos por "la realidad"; los cientficos, que son seres humanos, de algn modo deciden con qu cosas han de tratar (ya sean tericas o empricas), no las "encuentran" aqu y all. Este modo de concebir el mundo de la ciencia no significa que sus conceptos y objetos sean libres invenciones. Hay lmites respecto de la posibilidad de construccin del conocimiento. Como

dicen los autores: "La naturaleza, realidad, mundo externo, o como queramos llamarlo, es interpretable de diversas maneras (en este aspecto reside el componente social del conocimiento) pero no de cualquier manera (de aqu su componente lgico y emprico).[...] No decidimos, en un sentido fuerte, cuales son los hechos del mundo, aunque s asumamos o alcancemos un consenso acerca de cmo describirlo o acerca de cmo manipularlos dados ciertos presupuestos y ciertos fines "extracientficos" (Gonzlez Garca, Lpez Cerezo y Lujn Lpez, 1996. 51). Esto significa decirle "no" al mantenimiento de una postura positivista

ya sin fundamento, pero tambin "no" a la anticiencia. La idea es desenmascarar a la ciencia en su pretensin hegemnica injustificada para acercarla a la sociedad y ponerla a su servicio, es decir, invertir la direccin que actualmente se da entre ambas. Ciencia bsica, ciencia aplicada, tcnica y tecnologa En los tpicos que hacen a la tica de la ciencia, tratados hasta el momento, no se ha hecho ninguna alusin a las posibles diferencias entre ciencia y tecnologa. Ahora es preciso referirse al tema porque otro de los factores que favorecen el mantenimiento de una idea de ciencia desvinculada de la sociedad y productora de conocimiento universal

y necesario, tal como la ve el cientificista, es justamente la diferenciacin que se suele establecer entre ciencia o ciencia aplicada y tecnologa.[48] La distincin da pie muchas veces a la creencia de que la ciencia es valorativa o ticamente neutral porque slo se ocupa de producir conocimiento, el cual es indiferente respecto a valores o normas morales, y que es la tecnologa, en tanto aplicacin de ese conocimiento, la que tiene propsitos o fines que pueden ser intrnsecamente malos. Este argumento general del cientificista suele tener mucho peso en la sociedad por la propia consideracin de los cientficos respecto de su actividad; segn Stewart

Richards, cientfico ingls contemporneo dedicado al estudio de la epistemologa y la historia de la ciencia, "Muchos cientficos practicantes, que ignoran o desprecian la filosofa de la ciencia, se afe-rran a una concepcin idealizada de su profesin y propagan una opinin de la 'verdad cientfica' que implica absoluta certeza, objetividad y desprendimiento. Tal punto de vista podra sostenerse con conocimiento total de que muchos tipos de ciencia pueden practicarse solamente en virtud del apoyo financiero proporcionado por los gobiernos o las compaas industriales con fines que frecuentemente son poco claros, y casi siempre dirigidos por

intereses polticos o econmicos" (Richards, 1987. 172). La distincin entre ciencia y tecnologa suele deberse, adems, a que la gente tiene mayormente contacto directo con tcnicos o tecnlogos y no con los cientficos, quienes permanecen a salvo aislados en gabinetes o laboratorios, y es a aquellos a quienes atribuyen toda la responsabilidad (causal y moral) por los daos y perjuicios en el medioambiente social y natural. Es momento de preguntarse qu razonabilidad tiene la afirmacin sobre la distincin tajante entre ciencia y tecnologa, como si fueran dos empresas con propsitos cognitivos, sociales, polticos y econmicos tan diferentes

que, en tanto objetos de una evaluacin tica, segn los cientificistas, deben ser tratadas como mbitos independientes. Uno de los autores internacionalmente conocido que apoya la distincin es Mario Bunge, filsofo al que ya se ha hecho referencia. Bunge distingue entre ciencia bsica, ciencia aplicada y tecnologa. Las dos primeras tienen como objetivo la produccin de conocimiento aplicando el mtodo cientfico, pero mientras la primera "trabaja en los problemas que le interesan (por motivos puramente cognoscitivos), el investigador aplicado estudia solamente problemas de posible inters social" (Bunge, 1997. 42). Por eso, mientras la investigacin

bsica no se puede planear, la aplicada s. En la ciencia bsica el cientfico debe ser absolutamente libre en la eleccin de los temas a investigar, y en el tiempo y los procedimientos que utiliza. Si no se entorpece este modo de trabajo del cientfico bsico, el mismo producir resultados beneficiosos a la ciencia aplicada, a la tcnica y a la sociedad en forma automtica. La tcnica se diferencia de las dos formas de ciencia porque mientras stas buscan las leyes que explican la realidad, el propsito de la primera es controlar la realidad empleando principalmente el conocimiento y el mtodo cientfico, pero tambin otros tipos de conocimientos. Se encarga de

solucionar problemas prcticos, no cognoscitivos. La diferencia en los resultados de la investigacin bsica/aplicada y la tcnica tambin es inequvoca: en la primera el producto final es conocimiento y en la segunda es un artefacto o plan de accin con valor prctico. El cuadro trazado por Bunge no deja duda acerca de la concepcin del autor sobre la ciencia, la tcnica y sus relaciones. Y explica por qu sostiene la neutralidad valorativa de la ciencia y no de la tecnologa: la primera es totalmente independiente de cualquier inters externo u objetivo que trascienda el puro conocimiento, en cambio la segunda produce efectos reales en el

mundo. Una de sus conclusiones sobre el tema de la tica, la ciencia y la tcnica, es la siguiente: "A diferencia de la ciencia bsica o pura, que es intrnsecamente valiosa o, en el peor de los casos, carente de valor, la tecnologa puede ser valiosa o disvaliosa, segn sean los fines a los cuales sirve. Por consiguiente es preciso someter la tecnologa a controles morales y sociales" (Bunge, 1996. 125).[49] En la misma vena que Bunge, Nicolas Rescher (n. 1928), filsofo alemn radicado en EE.UU., en su obra Razn y valores en la Era cientficotecnolgica afirma que "Las ciencias de la Naturaleza y la tecnologa van

juntas como piezas de un mismo cuerpo. Ninguna puede ir lejos sin la otra" (1999, 100). Esta unidad, sin embargo, se debe al sofisticado desarrollo cientfico-tecnolgico actual y no a la funcin intrnseca de cada una. En este ltimo sentido, considera que el cometido de la ciencia es puramente cognitivo: "la representacin y racionalizacin 'desinteresadas' del hecho objetivo" (1999, 108). Mientras el conocimiento cientfico es slo descriptivo, el conocimiento tecnolgico presenta una vertiente cognitiva y otra normativa: la primera corresponde al "saber cmo hacer" y la segunda al "saber si" hacer algo concreto es razonable dadas las circunstancias. El

saber hacer es especficamente tcnico, pero generalmente est enraizado en el conocimiento cientfico. Por ser la ciencia descripcin (y explicacin) de lo que es, considerarla "antitica" respecto de los valores e intereses humanos es "irracional e inapropiado". "Las cuestiones normativas de valor, importancia, legitimidad y similares, quedan simplemente 'fuera del tema' en este proyecto" (1999, 111). Otros autores encuentran ms compleja la cuestin de la distincin entre ciencia y tecnologa y la consiguiente atribucin de responsabilidad moral. Evandro Agazzi (1934), filsofo italiano, considera que ciencia y tecnologa conforman un fenmeno de la

modernidad que no tiene precedentes en pocas anteriores. La constitucin de la tecnologa, afirma, es una consecuencia de la ciencia moderna; es esta ciencia la que proporciona los fundamentos tericos de la accin eficaz. Por eso la tecnologa no es la forma moderna de la tcnica sino una rama de la tcnica, la que se puede entender como "ciencia aplicada". La tcnica actual sigue el camino de la tcnica antigua: la fabricacin de dispositivos tiles descubiertos empricamente, sin usar, ni necesitar, conocimiento cientfico. Una diferencia entre tcnica y tecnologa que tiene relacin directa con el tema de la responsabilidad moral es el vnculo que resulta establecido, por parte de cada

una, entre la humanidad y el mundo artificial. El surgimiento de la tcnica de algn modo produjo una bifurcacin en el mundo humano al producir lo artificial, pero esto signific una domesticacin del mundo natural para satisfacer las necesidades de la naturaleza humana; adems, el crecimiento de ese mundo artificial fue lento y fragmentario, permitiendo as la integracin paulatina de los artefactos al ambiente natural y al contexto de las condiciones humanas existentes. En el caso de la tecnologa, la naturaleza no se domestica sino ms bien es sustituida por los artefactos, y el mundo artificial crece con una rapidez, una amplitud y una complejidad tal que le confiere las

caractersticas de una autonoma creciente. Aunque la produccin de cada tecnologa es local, su impacto y consecuencias, que son imprevisibles, inmediatamente se globalizan. El modo en que cada uno de estos procesos, el tcnico y el tecnolgico, de acuerdo con Agazzi, se relaciona con el tema de la tica, se da a travs de la necesidad de regulacin requerida por el mundo artificial que generan. En el caso de la tcnica, se supona que sta se pona al servicio del hombre adaptando la naturaleza a sus necesidades, pero se lo haca "obedeciendo a la naturaleza", motivo por el cual la necesidad de regulacin estaba implcita en las "reglas del

oficio". Pero en el caso de la tecnologa, el mundo artificial es creado por los seres humanos en forma independiente de la naturaleza, por eso necesita ser regulado por los propios seres humanos en forma de normas que aseguren la armona y sabidura que antes se confiaba a la naturaleza. La caracterstica de la ciencia y la tecnologa consistente en conllevar la posibilidad de su uso incontrolado hace que las mismas no contengan las directivas de su propio uso. El campo de la tica constituye el campo apropiado desde donde se fijen las directivas para su ejercicio positivo. Es preciso tener en cuenta que para Agazzi la posibilidad de distincin entre

ciencia y tecnologa es a lo sumo de carcter conceptual, pero no existe entre ellas ninguna distincin real, de ah la pertinencia del trmino "tecnociencia" que refiere a un fenmeno unitario. No obstante, para el autor, la cuestin de la responsabilidad exigida a la tecnociencia no debe entenderse como la de un asunto externo a la propia ciencia o como la posible existencia de un tribunal moral que juzgue sus producciones. Es al cientfico individual, en tanto investigador, a quien compete el control responsable de la tecnociencia, pues guindose por los principios de la moralidad, puede evaluar cada etapa de la investigacin de modo que el resultado final pueda ser

visto por la humanidad como un beneficio y no como una amenaza. Sin embargo, el enfoque de Agazzi puede ser cuestionado. La formacin tica de los cientficos es necesaria, y tambin lo es seguir las normas establecidas para la regulacin de la actividad cientfica. Pero de ninguna manera es suficiente. Los peligros inherentes a la tecnociencia, como dice el propio Agazzi, son incontrolables, y los conflictos actuales y potenciales que genera exceden los lmites personales de responsabilidad y pide el control ticopoltico y social. ste sera el modo ms adecuado para que la ciencia y la tecnologa logren la confianza y la cooperacin de la sociedad toda en el

cumplimiento de los objetivos de la ciencia, que no seran otros, de modo mediato, que los de la propia sociedad. Un ejemplo de armonizacin de los intereses sociales y los cientficos se ha visto en el presente documento en el tema de la evaluacin de riesgo de las tecnologas. Len Oliv (n. 1950), filsofo mexicano, rechaza la idea de que la neutralidad valorativa sea una caracterstica de la ciencia, porque tanto ella como la tecnologa estn presentes en el desarrollo de una tecnologa dada. Tanto en una como en otra hay que aplicar lo que cientficos y tecnlogos, as como instituciones ecologistas, han llamado el "principio de precaucin",

que exhorta a suspender la implementacin de determinadas tecnologas si existen bases razonables, "aunque no se tenga evidencia conclusiva de que existen relaciones causales" (Oliv, 2000. 66), de que una innovacin puede provocar daos en el medioambiente. A pesar de que es difcil establecer cules son las bases razonables cuando intervienen actores de cuo muy distinto como empresarios y funcionarios de Estado en la puesta en marcha de una determinada tecnologa, la nica solucin justa es la discusin y participacin de todos los afectados por esa tecnologa, incluso la comunidad ms amplia.

Si parece difcil la aplicacin del principio de precaucin en el campo de la tecnologa por la diversidad de actores intervinientes en las decisiones y/o evaluaciones, ms difcil resulta pensar que un cientfico, antes de obtener la evidencia conclusiva, sobre bases slo "razonables", deba, por razones ticas solamente, suspender un determinado desarrollo. Sin embargo, como documenta Oliv, ese caso existi: Mario Molina, Premio Nobel de Qumica (1995) junto a su colega Sherwood Rowland, enfrentaron lo que Molina llam "un problema de tica superior", a principios de la dcada del 70, cuando sospecharon que los clorofluorocarburos (CFC), presentes en

muchos productos industriales (aerosoles, aire acondicionado, etc.), provocaban la destruccin de la capa de ozono de la atmsfera terrestre. No es necesario reproducir aqu la historia de lo sucedido; lo importante es el nfasis de Oliv sobre el hecho de que las consecuencias dainas de la sustancia podan ser detectadas en el "contenido" mismo de la creencia cientfica. Por eso afirma tajantemente: "No es cierto que los nicos problemas morales de la ciencia y la tecnologa los constituya el uso posterior (bueno o malo) que se haga de los conocimientos" (Oliv, 2000. 75). Reflexiones generales sobre el tema Los beneficios que la ciencia y la

tecnologa proporcionan a la sociedad son cuantiosos, pero tambin estn rodeados de riesgos. Aqu se ha defendido la idea de que la responsabilidad por los riesgos actuales y potenciales de la empresa tecnocientfica abarca diversos aspectos de las actividades de investigacin y tambin los resultados de la misma. Cientficos y tecnlogos comparten esa responsabilidad en la medida en que ciencia y tecnologa no constituyen mbitos separados con su propia lgica, sino son extremos de un continuo que los incluye. Antes de la explosin tecnolgica del siglo XX, la ciencia guardaba cierta distancia del resto de la sociedad, pero

con el surgimiento de tecnologas de uso cotidiano, la gente comienza a tomar conciencia de la participacin en los riesgos que tiene la ciencia. El cuestionamiento por los "descubrimientos" cientficos se fue extendiendo y llev a que muchos sectores sociales tomaran cartas en el asunto. Los comits de tica en hospitales y centros de salud, los acuerdos internacionales sobre el cuidado de los recursos disponibles en el planeta, como el protocolo de Kyoto de 1997 y la recientemente finalizada Cumbre de Copenhague sobre la reduccin de gases que causan el efecto invernadero, y otros acuerdos regionales sobre el uso cauteloso de los bienes

naturales compartidos, son ejemplos de la bsqueda de lmites a las actividades de la ciencia y la tecnologa. Aun as los controles polticos y sociales son muy resistidos, y lo son en nombre de una ciencia que muchas veces se autocalifica como autnoma y reclama independencia completa de los dems sectores sociales. Ya se vio que algunas posiciones filosficas sobre la ciencia respaldan tal pretensin argumentando que la ciencia slo produce conocimiento objetivo universal-mente vlido, que por eso mismo es neutralmente valorativo, no pudindosele imputar ningn tipo de responsabilidad moral. Pero esto no es otra cosa que el enfoque que hemos

llamado "cientificista" que es cerrado y determinante respecto del papel jerrquico y hegemnico que otorga a la ciencia y los valores que porta sobre otros mbitos de inters social. Se ha tratado de mostrar aqu la insuficiencia de esta perspectiva, ya sea desde el punto de vista epistemolgico, tico o social. El conocimiento cientfico no tiene la precisin ni la objetividad que se crea al comienzo, tampoco est libre de las influencias externas en la eleccin de problemas y la justificacin de las soluciones. Adems, como hija de su tiempo, puede estar sujeta a la direccin que le marcan las fuentes de financiacin o los fines polticos. En el terreno de los efectos y

resultados de las investigaciones, la falta de responsabilidad ha sido y es frecuente: la sofisticacin de los armamentos es cada vez mayor, los experimentos con personas no informadas en lugares del Tercer Mundo an ocurren, siempre aparecen sustancias nuevas que son contaminantes, etc. La solucin propuesta consiste en que los proyectos de investigacin y de desarrollo tecnolgico sean puestos a consideracin de los interesados o afectados por ellos. A muchos puede parecerles al menos utpica este tipo de sugerencia, pero no hay soluciones indiscutibles, salvo en lo que hacen al mantenimiento del respeto y dignidad de

las personas. Adems, una ciencia no contaminada por los intereses econmicos, polticos y sociales no deja tampoco de ser una idea utpica. La dimensin tica la atraviesa, como lo hace con todo quehacer humano. Es preciso aclarar que al tratar el tema de tica de la ciencia y sus aplicaciones derivadas, que es objeto de enfoques controvertidos e interpretaciones contrapuestas, se incurre en simplificaciones. Algunas veces ocurre involuntariamente, pero la mayora de las veces es necesario realizar recortes con el fin de lograr concretar algunas ideas, aunque con ello se cercene la profundidad que tiene este tema. Queda pues, como tarea siempre pendiente,

rescatar la riqueza y complejidad que le es inherente al estudio de un fenmeno tpicamente humano como es el de la ciencia y su relacin con la tica. Bibliografa Agazzi, E. (1997). Equivalence or Separation Betwen Science and Technology? En From Technique to Technology: The Role of Modern Science. Society for Philosophy & Technology. Vol. 4, n 2. vila, H. (2002). Ciencia y tica. En Temas de pensamiento cientfico. Buenos Aires: Eudeba. Bonilla, A. (2003). La tica de la responsabilidad de Hans Jonas en su discusin contempornea. (Conferencia sin editar). Pontificia Universidade

Catolica Do Rio Grande Do Sul. Brasil. Bunge, M. (1996). tica, Ciencia y tcnica. Buenos Aires: Sudamericana. Bunge, M. (1997). Ciencia, tcnica y Desarrollo. Buenos Aires: Sudamericana. Ciapuscio, H. (1994). El fuego de Prometeo. Buenos Aires: Eudeba. Echeverra, J. (1998). Filosofa de la ciencia. Madrid: Akal. Funtowicz, S. y Ravetz, R. J. (1997). Problemas ambientales, ciencia postnormal y comunidades de evaluadores extendidas. En Gonzlez Garca, M. I., Lpez Cerezo, J. A. y Lujn Lpez, J. L. Ciencia, tecnologa y sociedad. Buenos Aires: Ariel. Gonzlez Garca, M. I., Lpez Cerezo, J.

A. y Lujn Lpez, J. L. (1996). Ciencia, tecnologa y sociedad. Madrid: Tecnos. Guariglia, O. (1996). Moralidad. tica universalista y sujeto moral. Buenos Aires: Fondo de Cultura Econmica. Laudan, L. (1984). Science and Values. Berkeley: Univ. of California Press. Mulkay, M. (1994). La ciencia y el contexto social. En La explicacin social del conocimiento. Mxico: UNAM. National Academy of Sciences. (1995). On Being a Scientist: Responsible Conduct in Research. Washington DC. Oliv, L. (2000). Ciencia y tecnologa. En Los linderos de la tica. Madrid: Siglo Veintiuno. Rescher, N. (1999). Razn y valores en

la Era cientfico-tecnolgica. Barcelona: Paids. Richards, S. (1987). Filosofa y sociologa de la ciencia. Mxico: Siglo Veintiuno. Shiva, V. (1993). Colonialism and the Evolution of Masculinist Forestry. En Harding, S. (ed.). The "Racial" Economy of Science. Indiana University Press.

Notas
[1]1

Samaja, J. (1993). Epistemologa y Metodologa (329). Buenos Aires: Eudeba. [2]2. Recomendamos para el conocimiento de la obra de Lewis Carroll la edicin de: Eduardo Stilman (comp. y trad.). (1998). Lewis Carroll. Los libros de Alicia. Buenos Aires: Ediciones de la Flor y Best Ediciones, con prlogo de Jorge Luis Borges. En la misma, se encuentra la edicin crtica de las Aventuras de Alicia en el pas de las maravillas, A travs del espejo y lo que Alicia encontr all, La avispa con peluca, La caza del Snark y una

cantidad importante de cartas y fotografas. [3]3. Las citas de la obra de Lewis Carroll, Alicia en el pas de las maravillas, corresponden a la edicin de Alianza, Buenos Aires, 1990. [4]4. En 1856, a pedido del editor de sus primeros poemas, Charles Dodgson debe elegir un seudnimo por lo que propone varios: 1) Edgar Cuthwellis, por transposicin de Charles Lutwidge, 2) Edgar U. C. Westhill, 3) Louis Carroll, derivado de Lutwidge=Ludovic y Charles=Carolus, 4) Lewis Carroll. El editor escogi la ltima opcin. En adelante har una tajante separacin entre las obras publicadas por Dodgson, las publicadas por Carroll y otras

totalmente annimas. [5]1 Eco, U. (1993). El nombre de la rosa. Barcelona: RBA Editores.
[6]2 Bertelloni, F. (1997). Para leer El nombre de la rosa de
Umberto Eco. Sus temas histricos, filosficos y polticos.

Buenos Aires: Oficina de publicaciones del C.B.C. [7]3 Eco, U. (1993, 198). [8]4 Borges, J. L. (1969). El Golem. En El otro, el mismo. Buenos Aires: Emec. [9]5 Morris, C. (1985). Fundamentos de la teora de los signos. Barcelona: Paids. [10]1 Pitcher, G. (1964). The Philosophy of Wittgenstein. Englewood Cliffs: Prentice-Hall. [11]2 Devereux, G. y Weiner, F. (1950). The Ocupation Status of Nurse. En

American Sociological Review. N 5 (vol. 15). [12]3 Weininger, O. Sexo y Carcter (286). Buenos Aires: Losada. [13]4 Tapia, A. El rbol de la retrica. [en lnea]. [consulta: 26 de julio de 2010]. <http://elarboldelaretorica.blogspot.com/2 argumentativos.html> [14]1 Mostern, J. (2000). Conceptos y teoras en la ciencia (208-209). Madrid: Alianza. [15]2 El acertijo "MU" extrado de Hofstadter, D. R. (1979). Gdel, Escher, Bach: an Eternal Goleen Braid (34-41 y 260-261). N.Y.C.: Basic Books. [16]3 Borges, J. L. (1979). El libro de

arena. En El libro de arena. Madrid: Alianza-Emec. [17]4 Martnez, G. (2001). Borges y la matemtica. Buenos Aires: Eudeba. [18]5 Martnez, G. y Pieiro, G. (2009). Gdel para todos. Buenos Aires: Seix Barral. [19]1 Peri Rossi, C. (1999). El amor es una droga dura. Barcelona: Seix Barral. [20]2 "La Esfericidad de la Tierra 2: De la Antigedad a Coln". (2009, enero 6). Odisea csmica. Blog de actualizacin astronmica y espacial. [en lnea]. [consulta: 10 de junio de 2010] <http://www.odiseacosmica.com> [21]1 Elster, J. (2000). El cambio

tecnolgico. Barcelona: Gedisa. [22]2 Elster, J. (1991). Tuercas y tornillos (67-68). Barcelona: Gedisa. [23]1 Se entiende por monismo metodolgico a la corriente epistemolgica que sostiene que existe un nico mtodo aplicable a las ciencias tanto naturales como sociales, afirmando de esta manera el monopolio del mtodo hipottico-deductivo, o del dialctico. En este sentido, toda posicin monista es reduccionista, ya sea que reduzca la cuestin del mtodo al hipotticodeductivo, o ya sea que la reduzca al dialctico. En este sentido, a ambos tipos de ciencias fcticas, o bien se aplica el mtodo hipottico-deductivo, o bien se aplica el dialctico, reduciendo

ulteriormente toda otra posibilidad. (Asti Vera y Ambrosini, 2009.189) [24]2 Se entiende por pluralismo metodolgico a aquella corriente epistemolgica que sostiene que para cada tipo de ciencias fcticas, es decir, para las naturales y para las sociales, existe un mtodo propio de justificacin de hiptesis o teoras para cada una de ellas; en el primer caso, podr ser el inductivista o el hipottico-deductivo; en el segundo, ser el intencional, el gentico, etc. (Ibdem) [25]3 Tradicionalmente se denominan como "filsofos analticos" a aquellos cuya actividad principal consiste en argumentar a favor o en contra de distintas posiciones filosficas, incluso

pensando a la argumentacin como la principal actividad de los filsofos. Este tipo de actitud filosfica estaba encarnada en los filsofos sociales como Dewey-, los matemticos o los cientficos naturales -como Carnal y Reichenbach. Segn Rorty, actualmente el panorama ha cambiado, y esta actitud la detentan los abogados, y lo que definira entonces a la filosofa analtica hoy sera un "estilo", una "aptitud filosfica" consistente en ser capaz de discernir en cualquier argumento "lagunas lgicas", y de producir un buen argumento en favor de cualquier posicin. Frente a esta corriente filosfica encontramos a los llamados "filsofos continentales", como

Heidegger, Nietzsche, Derrida, Foucault, Gadamer, etc., cuyo objetivo, sostiene Rorty, consiste en la bsqueda de una reconstruccin histrica exhaustiva, de una redescripcin del pasado adecuada para distintos fines y, en este sentido, Rorty los llama "crticos de la cultura". (Comesaa, 1998. 114115) [26]4 Asti Vera, C. y Ambrosini, C. (2009. 218-220). [27]5 Comprese con el mismo concepto y el ejemplo ya citado en este Documento. [28]6 Phillip Melanchthon, nacido como Phillip Schwartzerd, fue un reformista germano, colaborador de

Martn Lutero, y principal jefe del luteranismo tras la muerte de su fundador. Fue uno de los lderes de la Reforma Luterana, aunque su intencin era lograr un entendimiento entre protestantes y catlicos. Su obra y lecciones han tenido una gran influencia en la educacin, tal es as que se convirti en el autor ms ledo de su tiempo, y algunas de sus obras fueron textos de estudio en las escuelas hasta 1800. [29]1 Biblioteca Escptica. [en lnea]. [consulta: 26 de julio de 2010]. <http://bibliotecaesceptica.word

son-laspseudocienciasmario-bunge/#more-1743> [30]2 Kuhn, T. (1986). La estructura de las revoluciones cientficas. Mxico: Fondo de Cultura Econmica. [31]3 Edmonds, D. J. y Eidinow, J. A. (2001). El atizador de Wittgenstein. Una jugada incompleta (222). Barcelona: Pennsula. [32]4 Semmelweis, I. (1988). Etiologa, concepto y profilaxis de la fiebre puerperal. En El desafo de la epidemiologa, problemas y lecturas seleccionadas. Organizacin Panamericana de la Salud.

[33]1

Para deslindar los distintos aspectos de la planificacin de la poltica cientfica, nos basaremos sobre todo en el captulo mencionado de E. P. Yfera. [34]2 Florentino Ameghino (1854-1911), cientfico argentino fundador de la paleontologa sudamericana a fines del siglo XIX. [35]3 El adjetivo "malthusiana" alude a la prediccin de Malthus, T. R. (17661834), economista y demgrafo britnico. Segn esa prediccin, puesto que la poblacin tiende a crecer en progresin geomtrica y los alimentos en progresin aritmtica, llegar un da en que la poblacin superar los medios de subsistencia de no mediar obstculos

preventivos y represivos. Si se entiende el factor "poblacin" como poblacin de investigadores y el factor "medios de subsistencia" en calidad de recursos financieros, la prediccin resultante sera que si el aumento de investigadores crece geomtricamente y los recursos financieros lo hacen en proporcin aritmtica, la cantidad de investigadores superar el volumen de los recursos. Pero el caso ya mencionado de Venezuela contara como un contraejemplo de esa prediccin. [36]4 El destacado es del autor. [37]1 Bacon, F. (1961). Novum Organum, Buenos Aires: Losada. [38]2 Esta referencia a las fuentes diversas de las cuales surge la teora y

la tecnologa de la informacin puede servir como ejemplo de lo que se sostendr ms adelante sobre la construccin del conocimiento, tanto el cientfico como el tcnico. [39]3 Los distintos aspectos del significado del concepto de responsabilidad sealados en este artculo son tratados por Bonilla, A. (2003). [40]4 El concepto de responsabilidad colectiva genera muchos cuestionamientos, por ejemplo: cmo se puede adjudicar responsabilidad a un grupo por daos que provocaron unos pocos, sin violar los principios de la libertad individual?, se puede atribuir intencionalidad al grupo, considerado

distinto de sus miembros, para atribuirle responsabilidad?, etc. [41]5 La explosin ocurri en un reactor de la planta nuclear de Chernobyl, ex Unin Sovitica, el 26 de abril de 1986. [42]6 Larry Laudan, por ejemplo, afirma que la filosofa de la ciencia slo se puede ocupar de valores epistmicos o cognitivos, pero no debe preguntarse por el deber ser de los cientficos. [43]7 Washington DC: National Academy of Science, 1995. [44]8 Generalmente se cree que la recompensa de la que gozan los cientficos es de naturaleza intelectual u honorfica, en tanto la del tecnlogo es ms bien econmica, pues los inventos tienen un valor comercial del que carece

la produccin cientfica. [45]9 El origen del Positivismo lgico se remonta a la constitucin del Crculo de Viena en la Universidad de Viena en 1922. Sus representantes principales son L. Wittgenstein y M. Schlick. Sostienen una forma de empirismo estricto: las proposiciones cientficas pueden ser verificadas en forma completa por la experiencia. El Empirismo lgico que contina esta corriente, representado principalmente por R. Carnap, C. Hempel y H. Reichembach, encarna un empirismo ms moderado: la experiencia slo puede mostrar que las proposiciones cientficas son probables. [46]10 A partir de la obra de Kuhn (1962) surge una profusin de trabajos

sobre la ciencia sealando especialmente su relacin con la sociedad actual; constituyen un conjunto heterogneo, pero suelen agruparse bajo la sigla CTS (Ciencia, Tecnologa y Sociedad). Se ha distinguido entre la tradicin europea de Science and Technology Studies y la tradicin americana: STS (Science, Technology and Society). Aunque comparten presupuestos generales sobre la dimensin social de la ciencia y la tecnologa, la primera enfatiza el origen de las teoras cientficas, es decir, en la ciencia como proceso. La segunda se centra en la descripcin de los lazos polticos, econmicos y culturales y las teoras cientficas. Ver Gonzlez Garca,

M., Lpez Cerezo, J. y Lujn Lpez, J. (1996). 11 La expresin es de Vandana Shiva (1993). [47]12 La tesis de la incomensurabilidad entre paradigmas y el relativismo que implica fue radical en La estructura de las revoluciones cientficas (Kuhn, 1962), pero luego, ya en la Posdata de La estructura... y en las obras posteriores, fue debilitando ese relativismo haciendo hincapi en la inconmensurabilidad lingstica. [48]13 Ciapuscio (1994) desarrolla un panorama completo de las teoras sobre las diferencias entre ciencia y tecnologa. [49]14 Es preciso aclarar que para Bunge, el cientfico s es moralmente

responsable pues se puede prestar a cualquier tipo de conducta corrupta o a alterar el correcto proceso de investigacin cediendo a presiones externas. Ver Bunge (1996), captulo V.

S-ar putea să vă placă și